Download as pdf or txt
Download as pdf or txt
You are on page 1of 128

Vel Tech High Tech Dr.

Rangarajan
Dr.Sakunthala Engineering College

Avadi, Chennai - 600062.

DEPARTMENT OF MATHEMATICS

Transforms and Partial


Differential Equations(TPDE)
MA8353
by
K.A. Niranjan Kumar
MA8353 TRANSFORMS AND PARTIAL DIFFERENTIAL EQUATIONS LTPC
4 0 0 4
UNIT I PARTIAL DIFFERENTIAL EQUATIONS(P.D.E.) 12

Formation of partial differential equations – Singular integrals - Solutions of standard types of first
order partial differential equations - Lagrange’s linear equation - Linear partial differential equations of
second and higher order with constant coefficients of both homogeneous and non-homogeneous types.

UNIT II FOURIER SERIES(F.S.) 12

Dirichlet’s conditions – General Fourier series – Odd and even functions – Half range sine series – Half
range cosine series – Complex form of Fourier series – Parseval’s identity – Harmonic analysis.

UNIT III APPLICATIONS OF P.D.E.(A.P.D.E.) 12

Classification of PDE – Method of separation of variables - Fourier Series Solutions of one


dimensional wave equation – One dimensional equation of heat conduction – Steady state solution of
two dimensional equation of heat conduction.

UNIT IV FOURIER TRANSFORMS(F.T.) 12

Statement of Fourier integral theorem - Fourier transform pair - Fourier sine and cosine transforms -
Properties - Transforms of simple functions - Convolution theorem - Parseval’s identity.

UNIT V Z-TRANSFORMS AND DIFFERENCE EQUATIONS(Z.T.) 12

Z−transforms - Elementary properties – Inverse Z−transform (using partial fraction and residues) –
Initial and final value theorems - Convolution theorem - Formation of difference equations – Solution
of difference equations using Z− transform.

TOTAL : 60 PERIODS
OUTCOMES: Upon successful completion of the course, students should be able to:

• Understand how to solve the given standard partial differential equations.

• Solve differential equations using Fourier series analysis which plays a vital role in engineering
applications.

• Appreciate the physical significance of Fourier series techniques in solving one and two dimensional
heat flow problems and one dimensional wave equations.

• Understand the mathematical principles on transforms and partial differential equations would provide
them the ability to formulate and solve some of the physical problems of engineering.

• Use the effective mathematical tools for the solutions of partial differential equations by using Z
transform techniques for discrete time systems.
Contents

1 Partial Differential Equations(P.D.E.) 1


1.1 Part-A . . . . . . . . . . . . . . . . . . . . . . . . . . . . . . . . . . . . . . . . . . . . 1
1.2 Part-B . . . . . . . . . . . . . . . . . . . . . . . . . . . . . . . . . . . . . . . . . . . . 6
1.2.1 Examples of (Number of arbitrary constants > Number of independent variables) 6
1.2.2 Examples of (Number of arbitrary functions > 1 ) . . . . . . . . . . . . . . . . 7
1.2.3 Solution of standard types of first order partial differential equations . . . . . 9
1.2.3.1 Type II: z = px + qy + f (p, q) (Clairaut’s Form) . . . . . . . . . . . . 9
1.2.4 Examples of Lagrange’s Linear Equations by Method of multipliers . . . . . . 12
1.2.5 Examples of Homogeneous Linear P.D.E. . . . . . . . . . . . . . . . . . . . . . 17
1.3 Assignment I[Partial Differential Equations] . . . . . . . . . . . . . . . . . . . . . . . 21

2 Fourier Series(F.S.) 23
2.1 Part-A . . . . . . . . . . . . . . . . . . . . . . . . . . . . . . . . . . . . . . . . . . . . 23
2.2 Part-B . . . . . . . . . . . . . . . . . . . . . . . . . . . . . . . . . . . . . . . . . . . . 31
2.2.1 Examples under (0, 2π) . . . . . . . . . . . . . . . . . . . . . . . . . . . . . . . 31
2.2.2 Examples under (−π, π) . . . . . . . . . . . . . . . . . . . . . . . . . . . . . . 38
2.2.3 Half range Fourier series in (0, π) . . . . . . . . . . . . . . . . . . . . . . . . . 42
2.2.4 Examples under (0, 2ℓ) . . . . . . . . . . . . . . . . . . . . . . . . . . . . . . . 43
2.2.5 Examples under (−ℓ, ℓ) . . . . . . . . . . . . . . . . . . . . . . . . . . . . . . . 43
2.2.6 Examples under (0, ℓ) . . . . . . . . . . . . . . . . . . . . . . . . . . . . . . . . 43
2.2.7 Complex form of Fourier Series . . . . . . . . . . . . . . . . . . . . . . . . . . 43
2.2.8 Harmonic Analysis . . . . . . . . . . . . . . . . . . . . . . . . . . . . . . . . . 44
2.2.8.1 Examples under π form(Radian form) . . . . . . . . . . . . . . . . . 44
2.2.8.2 Examples under θ form(Degree form) . . . . . . . . . . . . . . . . .

44
2.2.8.3 Examples under T form: (θ = 2πx/T ) . . . . . . . . . . . . . . . . . 45
2.2.8.4 Problems under ℓ form: (2ℓ = Number of data) . . . . . . . . . . . . 46
2.3 Assignment II[Fourier series] . . . . . . . . . . . . . . . . . . . . . . . . . . . . . . . . 47

3 Applications of P.D.E.(A.P.D.E.) 49
3.1 Part-A . . . . . . . . . . . . . . . . . . . . . . . . . . . . . . . . . . . . . . . . . . . . 49
3.2 Part-B . . . . . . . . . . . . . . . . . . . . . . . . . . . . . . . . . . . . . . . . . . . . 52
3.2.1 One dimensional wave equation . . . . . . . . . . . . . . . . . . . . . . . . . . 52
3.2.1.1 Zero Velocity Problems . . . . . . . . . . . . . . . . . . . . . . . . . 52
4

3.2.1.2 [Non - Zero Velocity Problem] . . . . . . . . . . . . . . . . . . . . . . 60


3.2.2 One Dimensional heat flow equation . . . . . . . . . . . . . . . . . . . . . . . 65
3.2.2.1 Steady state conditions and zero boundary conditions . . . . . . . . . 65
3.2.2.2 Steady state conditions and non-zero boundary conditions . . . . . . 68
3.2.3 Two dimensional heat flow equations . . . . . . . . . . . . . . . . . . . . . . . 71
3.2.3.1 Examples of two dimensional finite plates . . . . . . . . . . . . . . . 71
3.2.3.2 Examples of two dimensional infinite plates . . . . . . . . . . . . . . 78
3.3 Assignment III[Applications of Partial Differential Equations] . . . . . . . . . . . . . 83

4 Fourier Transforms(F.T.) 85
4.1 Part-A . . . . . . . . . . . . . . . . . . . . . . . . . . . . . . . . . . . . . . . . . . . . 85
4.2 Part-B . . . . . . . . . . . . . . . . . . . . . . . . . . . . . . . . . . . . . . . . . . . . 92
4.2.1 Examples under Fourier Transform Pair . . . . . . . . . . . . . . . . . . . . . 92
4.3 Parseval’s Identity for Fourier transform: . . . . . . . . . . . . . . . . . . . . . . . . . 96
4.3.1 Examples under Fourier Sine & Cosine Transform: . . . . . . . . . . . . . . . . 96
4.3.2 Examples under Convolution & Parseval’s identity: . . . . . . . . . . . . . . . 97
4.4 Assignment IV[Fourier Transforms] . . . . . . . . . . . . . . . . . . . . . . . . . . . . 98

5 Z-Transforms(Z.T.) 99
5.1 Part-A . . . . . . . . . . . . . . . . . . . . . . . . . . . . . . . . . . . . . . . . . . . . 99
5.2 Part-B . . . . . . . . . . . . . . . . . . . . . . . . . . . . . . . . . . . . . . . . . . . . 103
5.3 Inverse Z-transform . . . . . . . . . . . . . . . . . . . . . . . . . . . . . . . . . . . . . 104
5.3.1 Evaluation of inverse Z-transform using the method of Convolution . . . . . . 105
5.3.2 Evaluation of inverse Z-transform using partial fractions method . . . . . . . . 109
5.3.3 Evaluation of inverse Z-transform by Residue method . . . . . . . . . . . . . . 111
5.3.4 Solution of difference equations using Z−transform . . . . . . . . . . . . . . . 116
5.4 Assignment V[Z-Transform and difference equations] . . . . . . . . . . . . . . . . . . 123
1 Partial Differential Equations(P.D.E.)

1.1 Part-A

1. Form the partial differential equation by respectively,


eliminating the arbitrary constants a and b
∂z
from z = (x + a)2 + (y + b)2 . 2(x − a) + 2z =0
∂x
Soln : Given z = (x + a)2 + (y + b)2 (1) ∂z
⇒ (x − a) + z =0
∂x
⇒ (x − a) = −zp
∂z p
p= = 2(x + a) ⇒ = (x + a) (2)
∂x 2 ⇒ (x − a)2 = (zp)2 (2)
∂z q ∂z
q= = 2(y + b) ⇒ = (y + b) (3) 2(y − b) + 2z =0
∂y 2 ∂y
∂z
⇒ (y − b) + z =0
Squaring (2) & (3) and substituting in ∂y

equation (1), we get ⇒ (y − b) = −zq


⇒ (y − b)2 = (zq)2 (3)
 p 2
= (x + a)2
2
 q 2 substituting (2) & (3) in equation (1), we
= (y + b)2
2 get
 p 2  q 2
∴ (1) ⇒ z = + (zp)2 + (zq)2 + z 2 = 1
2 2
p2 q 2 z 2 (p2 + q 2 + 1) = 1
= +
4 4
p + q2
2
3. Form the partial differential equation of all
=
4
the spheres whose centres are on the line
4z = p + q 2
2
x=y=z
Soln : The equation of all the spheres
2. Form the partial differential equation by having their center on the line x = y =
eliminating the arbitrary constants a and b z(=‘a’) and radius ‘b’ is given by
from (x − a)2 + (y − b)2 + z 2 = 1.
Soln : (x − a)2 + (y − a)2 + (z − a)2 = b2 (1)
2 2 2
Given (x − a) + (y − b) + z = 1 (1)
Differentiate (1) partially w.r.t. x and y Differentiate (1) partially w.r.t. x and y
2 Part-A : Unit I - PARTIAL DIFFERENTIAL EQUATIONS (P.D.E.)

respectively, respectively,
∂z
2(x − a) + 2z =0
∂x
∂z
⇒ (x − a) + z =0
∂z ∂x
2(x − a) + 2(z − a) =0
∂x ⇒ (x − a) = −zp
∂z
⇒ (x − a) + (z − a) =0 ⇒ (x − a)2 = (zp)2 (2)
∂x
⇒ (x − a) + (z − a)p = 0 ∂z
2(x − a) + 2z =0
∂y
⇒ a(1 + p) = (x + zp)
∂z
(x + zp) ⇒ (y − b) + z =0
∴a= (2) ∂y
(1 + p)
⇒ (y − b) = −zq
∂z
2(y − a) + 2(z − a) =0 ⇒ (y − b)2 = (zq)2 (3)
∂y
∂z
⇒ (y − a) + (z − a) =0 Substituting (2) & (3) in equation (1), we
∂y
⇒ (y − a) + (z − a)q = 0 get,

⇒ a(1 + q) = (y + zq) (zp)2 + (zq)2 + z 2 = 4


(y + zq) z 2 (p2 + q 2 + 1) = 4
∴a= (3)
(1 + q)
5. Form the partial differential equation by
eliminating the arbitrary constants ‘a’ and
‘b’ from 2z = (ax + y)2 + b.
Equating (2) & (3), we get Soln : Given 2z = (ax + y)2 + b (1)
Differentiate (1) partially w.r.t. x and y
respectively,

(x + zp) (y + zq) 2p = 2a(ax + y) + 0 (2)


a= =
(1 + p) (1 + q) 2q = 2(ax + y) (3)
⇒ x(1 + q) − y(1 + p) + z(p − q) = 0 (2) p
⇒ =a (4)
(3) q
From (3), q = ax + y
p
q = x+y [∵ from (4)]
q
⇒ px + qy = q 2
4. Form the partial differential equation by
eliminating the arbitrary constants a and b 6. Find the partial differential equations of all
from (x − a)2 + (y − b)2 = 4 − z 2 . planes cutting equal intercepts with x and
2 2 2
Soln : Given (x − a) + (y − b) + z = 4 y-axes. (N/D ’07)
(1) Soln : The equation of the plane making
Differentiate (1) partially w.r.t. x and y equal intercepts ′ a′ with x and y-axes is
MA8353 Transforms and Partial Differential Equations by K A Niranjan Kumar 3

given by 8. Form the partial differential equation by


x y z eliminating the arbitrary function from
+ + =1 (1) 
a a b z = f x2 + y 2 .

Soln : Given z = f x2 + y 2 (1)
Differentiate (1) partially w.r.t. x and y
Differentiate (1) partially w.r.t. x and y
respectively,
respectively,
∂z
1 1 p ∂z 
+ ∂x = 0 ⇒ =− (2) p= = 2xf ′ x2 + y 2
a b a b ∂x
∂z p
1 ∂y 1 q ⇒ = f′ (2)
+ =0⇒ =− (3) 2x
a b a b 
∂z
q= = 2yf ′ x2 + y 2
Equating (2) and (3) we get, p = q. ∂y
q
⇒ = f′ (3)
7. Form the partial differential equation by 2y
eliminating the arbitrary constants a and b Equating (2) and (3) ,we get,
from (x − a)2 + (y − b)2 = z 2 cot2 α. p q
= ⇒ py − qx = 0
2x 2y
Soln : Given (x − a)2 + (y − b)2 = z 2 cot2 α
9. Form the partial differential equation by
(1)
eliminating
 xy  the arbitrary function from
Differentiate (1) partially w.r.t. x and y
z=f .
respectively, z  xy 
Soln : Given z = f (1)
∂z z
2(x − a) = 2z cot2 α Differentiate (1) partially w.r.t. x and y
∂x
∂z respectively,
⇒ (x − a) = z cot2 α  
∂x  
∂z xy y xy ∂z
⇒ (x − a) = −zp cot2 α p= = f′ × − 2
∂x z z z ∂x
2  y xy 
⇒ (x − a)2 = zp cot2 α (2)
⇒ p = f′ × − 2p
∂z z z
2(y − b) = 2z cot2 α ′ p
∂y ⇒ f =  y xy  (2)
∂z − 2p
⇒ (y − b) = z cot2 α z z
∂y q
Similarly f ′ =  x xy  (3)
⇒ (y − b) = −zq cot2 α − 2q
2 z z
⇒ (y − b)2 = zq cot2 α (3) Equating (2) and (3) ,we get,
p q
 y xy  =  x xy 
substituting (2) & (3) in equation (1), we − 2p − 2q
z z z z
get,
2 2 z(px − qy) = 0
zp cot2 α + zq cot2 α = z 2 cot2 α
10. Form the partial differential equation by
Dividing by cot2 α on both sides, eliminating the
  x
 arbitrary function from
z 2 p2 cot2 α + q 2 cot2 α = z 2 φ z 2 − xy, = 0.
z  
p2 + q 2 = tan2 α x
Soln : Given φ z 2 − xy, =0 (1)
z
4 Part-A : Unit I - PARTIAL DIFFERENTIAL EQUATIONS (P.D.E.)

x Equation (1) reduces to a + b = ab


Let u = z 2 − xy, v =
z
ab − b = a
∂u ∂v

⇒ b(a − 1) = a
∂x ∂x =0
∂u ∂v a
⇒b=
∂y ∂y a−1

z − px The complete solution is given by
2zp − y a
z2 = 0
−xq z = ax + y + c.
2zq − x a−1
z2 √ √
2 2
 13. Find the complete integral of p+ q = 1.
px − q xy − 2z = xz √ √
Soln : Given p + q = 1 (1)
Let z = ax + by + c (2)
11. Form the partial differential equation by be a solution of the above equation.
eliminating the arbitrary function from Differentiate (2) partially w.r.t. x and y
2 2

z =f x +y + x + y. respectively,
2 2

Soln : Given z = f x + y + x + y (1) ∂z ∂z
=p=a ; =q=b
Differentiate (1) partially w.r.t. x and y ∂x ∂y
Equation (1) reduces to
respectively,
√ √ √ 2
a+ b=1⇒b= 1− a

∂z  The complete solution is given by


p= = 2xf ′ x2 + y 2 + 1 √ 2
∂x z = ax + 1 − a y + c
⇒ p = 2xf ′ + 1 (2)
 14. Find the complete solution of the partial
∂z
q= = 2yf ′ x2 + y 2 + 1 differential equation p2 + q 2 − 4pq = 0.
∂y
⇒ q = 2yf ′ + 1 (3) Soln : Given p2 + q 2 − 4pq = 0 (1)
Let z = ax + by + c (2)
(2) × y − (3) × x gives, be a solution of the above equation.
    Differentiate (2) partially w.r.t. x and y
py − qx = 2xyf ′ + y − 2xyf ′ + x
respectively,
=y−x ∂z ∂z
=p=a ; =q=b
py − qx = y − x ∂x ∂y
Equation (1) reduces to
12. Find the complete integral of p + q = pq. a2 + b2 − 4ab = 0 [Quadratic in a]
Soln : Given p + q = pq (1)
i.e., b2 − 4ab + a2 = 0[Quadratic in b]
Let z = ax + by + c (2) √
4a ± 16a − 4a2
be a solution of the above equation. ⇒b=
p 2
Differentiate (2) partially w.r.t. x and y = a ± 4a − a2
respectively,
The complete
 solution is given by
∂z ∂z p
=p=a ; =q=b z = ax + a ± 4a − a2 y + c
∂x ∂y
MA8353 Transforms and Partial Differential Equations by K A Niranjan Kumar 5

15. Write down the complete solution of Put m = 2


p
px + qy = 1 + p2 + q 2 + z.
p
Soln: Given px+qy = 1+p2 +q 2 +z (1) ∴ (1) ⇒ 23 − 2.22 − 4.2 + 8 = 0
Rearranging the equation
p m = 2 is a root of the auxiliary equation.
z = px + qy − 1 + p2 + q 2
Solve by synthetic division method
The above equation is a Clairaut’s equation.
2 1 -2 -4 8
Put p = a ; q=b
0 2 0 -8
The complete solution is given
p 1 0 -4 0
z = ax + by − 1 + a2 + b 2
∴ m2 − 4 = 0 ⇒ m = 2, −2 roots.
16. Write down the complete solution of The roots of the auxiliary equation are
z = px + qy + p2 − q 2 . (May/June 2006) 2, 2, −2.
2 2
Soln : Given z = px + qy + p − q The solution is given by
The above equation is a Clairaut’s equation. z = f (y+2x)+xg(y+2x)+x2 h(y−2x).
Put p = a ; q=b  ′

The complete solution is given 19. Solve D2 − 2DD′ + D 2 z = 0. (AU
z = ax + by + a2 − b2 . Apr/May 08)
 ′ ′
 Soln : The auxiliary equation is given by
3 2 3
17. Solve D − 3DD + 2D z = 0.
m2 − 2m + 1 = 0, ∴ m = 1, 1 roots.
Soln : The auxiliary equation is given by
The solution is given by
3
m − 3m + 2 = 0 (1) z = f (y + x) + xg(y + x).
Put m = 1
20. Find the particular Integral of

∴ (1) ⇒ 13 − 3.1 + 2 = 0 D3 + 2D2 D z = sin(x + 2y).

m = 1 is a root of the auxiliary equation. Soln : Particular Integral is given by


Solve by synthetic division method 1
P.I. = sin(x + 2y)
1 1 0 -3 2 D3 + 2D2 D′
0 1 1 -2 {Here sin(ax + by) ⇒ a = 1; b = 2}
1 1 -2 0 1
= sin(x + 2y)
−12 D + 2 (−12 D′ )
∴ m2 + m − 2 = 0 ⇒ m = 1, −2 roots.
1
The roots of the auxiliary equation are =− sin(x + 2y)
D + 2D′
 
1, 1, −2. Multiply Nr. & Dr.by D − 2D′
The solution is given by D − 2D′
=− ′ sin(x + 2y)
z = f (y + x) + xg(y + x) + h(y − 2x) D2 − 4D 2
cos(x + 2y) − 2 × 2 cos(x + 2y)
18. 
Solve =−
′ ′ ′
 −12 − 4 (−22 )
3 2 2 3
D − 2D D − 4DD + 8D z = 0. 3 cos(x + 2y) cos(x + 2y)
= =
Soln : The auxiliary equation is given by 15 5

m3 − 2m2 − 4m + 8 = 0 (1)
6 Part-B : Unit I - PARTIAL DIFFERENTIAL EQUATIONS (P.D.E.)

1.2 Part-B

1.2.1 Examples of (Number of arbitrary constants > Number of independent variables)

Example 1.1. Form the partial differential equation of all spheres whose radii are the
same.

Solution: The equation of all sphere with equal radius can be taken as

(x − a)2 + (y − b)2 + (z − c)2 = r2 (1)

where a, b, c are arbitrary constants and r is a given constant. Since the number of arbitrary constants
is more than the number of independent variables, we will get the p.d.e. of order greater than 1.
Differentiating (1) partially w.r.t ‘x’
∂z
2(x − a) + 2(z − c) =0 i.e., (x − a) + (z − c)p = 0 (2)
∂x
Differentiating (1) partially w.r.t ‘y’
∂z
2(y − b) + 2(z − c) =0 i.e., (y − b) + (z − c)q = 0 (3)
∂y
Differentiating (2) partially w.r.t ‘x’
∂ 2z ∂z ∂z
1 + (z − c) 2 + . =0 i.e., 1 + (z − c)r + p2 = 0 (4)
∂x ∂x ∂x
Differentiating (2) partially w.r.t ‘y’
∂ 2 z ∂z ∂z
1 + (z − c) 2 + . =0 i.e., 1 + (z − c)t + q2 = 0 (5)
∂y ∂y ∂y
−p2 − 1 −q 2 − 1
From (4) and (5), we get z − c = and z − c =
r t
−p2 − 1 −q 2 − 1
i.e., z − c = =
r t
2 2
t(p + 1) = r(q + 1)
Example 1.2. Obtain the partial differential equation by eliminating a, b, c from
x2 y2 z2
+ 2 + 2 =1
a2 b c
Solution: Since the number of arbitrary constants is more than the number of independent variables,
we will get the p.d.e of order greater than ’1’.
x2 y 2 z 2
Given 2 + 2 + 2 = 1 (1)
a b c
Differentiating (1) partially w.r.t ‘x’
2x 2z ∂z x z
+ 2 =0 i.e., + 2p = 0 (2)
a2 c ∂x a 2 c
Differentiating (1) partially w.r.t ‘y’
2y 2z ∂z y z
+ 2 =0 i.e., + 2q = 0 (3)
b2 c ∂y b 2 c
MA8353 Transforms and Partial Differential Equations by K A Niranjan Kumar 7

Differentiating (2)2 partially w.r.t.


 ‘x’
1 1 ∂ z ∂z ∂z 1 1 2 2

+ z 2+ . =0 i.e., + zr + p =0 (4)
a2 c 2 ∂x ∂x ∂x a2 c 2
Differentiating (3)2 partially w.r.t
 ‘y’
1 1 ∂ z ∂z ∂z 1 1 2 2

2
+ 2 z 2+ . =0 i.e., + zt + q =0 (5)
b c ∂y ∂y ∂y b2 c 2
Differentiating
 (2) partially w.r.t
 ‘y’
1 2
∂ z ∂z ∂z 1
0+ 2 z + =0 i.e., (zs + pq) = 0 (5)
c ∂y∂x ∂x ∂y c2
i.e., zs + pq = 0

Note : We may also get different partial differential equations. The answer is not unique.

1.2.2 Examples of (Number of arbitrary functions > 1 )

Example 1.3. Form PDE by eliminating arbitrary function f and g from


z = f (x + ay) + g(x − ay).

Solution: Given z = f (x + ay) + g(x − ay) (1)


Differentiating (1) partially w.r.t ‘x’
∂z
= f ′ (x + ay) + g ′ (x − ay) i.e., p = f ′ + g ′ (2)
∂x
Differentiating (1) partially w.r.t ‘y’
∂z
= af ′ (x + ay) − ag ′ (x − ay) i.e., q = af ′ − ag ′ (3)
∂y
Differentiating (2) partially w.r.t ‘x’
∂ 2z
= f ′′ + g ′′ i.e., r = f ′′ + g ′′ (4)
∂x2
Differentiating (3) partially w.r.t ‘y’
∂ 2z 
= a2 f ′′ + a2 g ′′ i.e., t = a2 f ′′ + g ′′ (5)
∂y 2
Using (4), we get t = a2 r.

Example 1.4. Eliminate the arbitrary function φ and ψ from z = xφ(y) + yψ(x) and
form the partial differential equation.

Solution: Given z = xφ(y) + yψ(x) (1)


Differentiating (1) partially w.r.t ‘x’

p = φ(y) + yψ ′ (x) (2)

Differentiating (1) partially w.r.t ‘y’

q = xφ′ (y) + ψ(x) (3)


8 Part-B : Unit I - PARTIAL DIFFERENTIAL EQUATIONS (P.D.E.)

Differentiating (2) partially w.r.t ‘y’

s = φ′ (y) + ψ ′ (x) (4)

Now, px + qy = xφ(y) + xyψ ′ (x) + xyφ′ (y) + yψ(x)


= xφ(y) + yψ(x) + xy(φ′ (y) + ψ ′ (x))
px + qy = z + xys [using (1) and (4)]
Example 1.5. Form  y  partial differential equation by eliminating arbitrary function f and
g from z = xf + yφ(x). [UQ]
x
y
Solution: Given z = xf + yφ(x) (1)
x
Differentiating (1) partially w.r.t ‘x’
∂z
 
y −y
 y
 
= xf ′ 2
+f + yφ′ (x)
∂x x x x
∂z −y ′ y
  y
= f +f + yφ′ (x)
∂x x x x
−y ′
i.e., p = f + f + yφ′ (2)
x
Differentiating (1) partially w.r.t ‘y’
∂z
 
y 1 ∂z y
 
= xf ′ . + φ(x) ⇒ = f′ + φ(x)
∂y x x ∂y x
i.e., q = f ′ + φ (3)

∂2z y
  
−y
 −y ′′
Now, = f ′′ . + φ′ (x) i.e., s = f + φ′ (4)
∂x∂y x x2 x2
∂ 2z
 
y 1 1 ′′
2
= f ′′ . i.e., t = f (5)
∂y x x x

∴ px + qy = −yf ′ + xf + xyφ′ + yf ′ + yφ [using (2) and (3)]


px + qy = xf + yφ + xyφ′

From (1), px + qy = z + xyφ′ (6)


−y yt
Using (5) in (4), s = 2 (xt) + φ′ ⇒ φ′ = s +
x x
Sub. the value of φ′ in (6), we get
 yt

px + qy = z + xy s +
x
2
px + qy = z + xys + y t
Example 1.6. Form a p.d.e by eliminating arbitrary function f and g from
z = f (x + y)g(x − y).

Solution: Given z = f (x + y)g(x − y) i.e., z = f g (1)


Differentiating (1) partially w.r.t ‘x’
MA8353 Transforms and Partial Differential Equations by K A Niranjan Kumar 9

∂z
= f g ′ + gf ′ i.e., p = f g ′ + gf ′ (2)
∂x
Differentiating (1) partially w.r.t ‘y’
∂z 
= f −g ′ + gf ′ i.e., q = gf ′ − f g ′ (3)
∂y
∂2z
2
= f g ′′ + g ′ f ′ + gf ′′ + f ′ g ′ i.e., r = f g ′′ + gf ′′ + 2f ′ g ′ (4)
∂x
∂ 2z
= −[f (−g ′′ ) + g ′ f ′ ] + gf ′′ + f ′ (−g ′ )
∂y 2
t = f g ′′ + gf ′′ − 2f ′ g ′ (5)

From (2) and (3), we get

p2 − q 2 = f 2 g ′2 + g 2 f ′2 + 2f gf ′ g ′ − (g 2 f ′2 + f 2 g ′2 − 2f gf ′ g ′ )
= 4f gf ′ g ′
p2 − q 2 = 4zf ′ g ′ [using(1)] (6)

From (4) and (5), we get

r − t = 4f ′ g ′ (7)

From (6) and (7), we get

p2 − q 2 = z(r − t)

1.2.3 Solution of standard types of first order partial differential equations

1.2.3.1 Type II: z = px + qy + f (p, q) (Clairaut’s Form)

Example 1.7. Find the complete and singular integrals of z = px + qy + pq.

Solution: Given z = px + qy + pq (1)


The given partial differential equation is in Clairaut’s form.
∴ The complete integral is z = ax + by + ab (2)
where a and b are arbitrary constants.
To find singular integral : Partially differentiating (2) w.r.t ‘a’ and ‘b’ then equating to zero,
we get

x+b=0
y+a=0

∴ a = −y and b = −x
10 Part-B : Unit I - PARTIAL DIFFERENTIAL EQUATIONS (P.D.E.)

Substituting the values of ‘a’ and ‘b’ in (2), we get

z = −yx − xy + (−y)(−x) = −yx − xy + yx


∴ z = −xy
which is the singular integral.
To find General Integral : Put b = β(a) in complete integral equation (2), we get

z = ax + β(a)y + aβ(a) (3)

Differentiate (3) w.r.t. ‘a’, we get


′ ′
z = x + β (a)y + aβ (a) + β(a) (4)

Eliminating ‘a’ from (3) and (4), we get general integral of the given p.d.e.

Example 1.8. Solve z = px + qy + p2 q 2 .

Solution: The given equation is in Clairaut’s form.


∴ The complete integral is z = ax + by + a2 b2 (1)
To find singular integral: Partially differentiating (1) w.r.t a and b then equating to zero, we get

x = −2ab2 (2)
y = −2a2 b (3)
xy = 4a3 b3 (4)
x
From (2), = −2ab
b
y
From (3), = −2ab
a

hx y
i
From (1), z = ab + + ab = ab(−2ab − 2ab + ab)
b a
2 2
z = −3a b (5)
Now, z 3 = −27a6 b6 i.e., z 3 = −27(a3 b3 )2
 xy 2 −27 2 2
3
Using (4), z = −27 ⇒ z3 = x y
4 16
which is the required singular integral.
The general integral is obtained as first example of this type.

Example 1.9. Solve z = px + qy + p2 + pq + q 2 .

Solution: The given equation is in Clairaut’s form.


∴ The complete integral is z = ax + by + a2 + ab + b2 (1)
MA8353 Transforms and Partial Differential Equations by K A Niranjan Kumar 11

To find singular integral: Partially differentiating (1) w.r.t a and b then equating to zero, we get

x + 2a + b = 0 (2)
y + a + 2b = 0 (3)

Solving (2) and (3), we get


1 1
a = (y − 2x), b = (x − 2y)
3 3
Substituting the values of a and b in (1), we get

3z = xy − x2 − y 2

which is the singular integral.


The general integral is obtained as first example of this type.
p
Example 1.10. Solve z = px + qy + 1 + p2 + q 2 .

Solution: The given equation is in Clairaut’s form.


p
∴ The complete integral is z = ax + by + 1 + a2 + b 2 (1)
To find singular integral: Partially differentiating (1) w.r.t a and b then equating to zero, we get
a −a
x+ √ =0⇒x= √ (2)
1 + a2 + b 2 1 + a2 + b 2
b −b
y+√ =0⇒y= √ (3)
1 + a2 + b 2 1 + a2 + b 2
From (2) and (3), we get
a2 + b 2
x2 + y 2 =
1 + a2 + b 2
a2 + b 2
1 − (x2 + y 2 ) = 1 −
1 + a2 + b 2
1
1 − x2 − y 2 =
1 + a2 + b 2
p 1
1 + a2 + b 2 = p (4)
1 − x2 − y 2

From (2) and (3), we get

−x
a= p (5)
1 − x2 − y 2
and
−y
b= p (6)
1 − x2 − y 2

Substituting (4), (5) and (6) in (1), we get


12 Part-B : Unit I - PARTIAL DIFFERENTIAL EQUATIONS (P.D.E.)

−x2 y2 1
z=p −p +p
1 − x2 − y 2 1 − x2 − y 2 1 − x2 − y 2
p
z= 1 − x2 − y 2
z 2 = 1 − x2 − y 2
∴ x2 + y 2 + z 2 = 1

which is the singular integral.


The general integral is obtained as first example of this type.

1.2.4 Examples of Lagrange’s Linear Equations by Method of multipliers

Example 1.11. Solve (y − z)p + (z − x)q = x − y.

Solution: Given (y − z)p + (z − x)q = x − y


dx dy dz
The subsidiary equations are = = (1)
y−z z−x x−y
dx + dy + dz dx + dy + dz
Each fraction of (1) = =
y−z+z−x+x−y 0
⇒ dx + dy + dz = 0
Integrating, x + y + z = a
Using x, y, z as multipliers,
xdx + ydy + zdz xdx + ydy + zdz
each fraction of (1) = =
x (y−z)+y (z −x)+z (x−y) 0
⇒ xdx + ydy + zdz = 0
x2 y 2 z 2
Integrating, + + =c
2 2 2
i.e., x2 + y 2 + z 2 = b

∴ The general solution is f x + y + z, x2 + y 2 + z 2 = 0.
∂z ∂z
Example 1.12. Solve (mz − ny) + (nx − lz) = ly − mx. [UQ]
∂x ∂y
Solution: Given (mz − ny) p + (nx − lz) q = ly − mx
dx dy dz
The subsidiary equations are = = (1)
mz − ny nx − lz ly − mx
Using x, y, z as multipliers, each fraction of (1)=

xdx + ydy + zdz xdx + ydy + zdz


=
x (mz − ny) + y (nx − lz) + z (ly − mx) 0
⇒ xdx + ydy + zdz = 0

x2 y 2 z 2
Integrating, we get + + = c1
2 2 2
i.e., x2 + y 2 + z 2 = a
Using l, m, n as multipliers, each fraction of (1) =
MA8353 Transforms and Partial Differential Equations by K A Niranjan Kumar 13

ldx + mdy + ndz ldx + mdy + ndz


=
l (mz − ny) + m (nx − lz) + n (ly − mx) 0
ldx + mdy + ndz = 0

Integrating, lx + my + nz = b
∴ The general solution is φ(x2 + y 2 + z 2 , lx + my + nz) = 0.
2 2
 2 2
 
Example 1.13. Solve x y − z p+y z −x q = z x − y2 .
2
[UQ]
  
Solution: Given x y 2 − z 2 p + y z 2 − x2 q = z x2 − y 2
The subsidiary equations are
dx dy dz
= =
x (y 2 2
−z ) 2 2
y (z − x ) z (x − y 2 )
2

Using x, y, z as multipliers, each fraction of (1)=


xdx + ydy + zdz xdx + ydy + zdz
=
x2 (y 2 − z 2 ) + y 2 (z 22 2 2 2
− x ) + z (x − y ) 0
⇒ xdx + ydy + zdz = 0

x2 y 2 z 2
Integrating, we get + + = c1
2 2 2
i.e., x2 + y 2 + z 2 = a
1 1 1
Using , , as multipliers, each fraction of (1) =
x y z
l 1 1 l 1
x dx + y dy + z dz x dx + y dy + z1 dz
=
(y 2 − z 2 ) + (z 2 − x2 ) + (x2− y2) 0
l 1 1
dx + dy + dz = 0
x y z

Integrating, log x + log y + log z = log b


i.e., xyz = b
∴ The general solution is φ(x2 + y 2 + z 2 , xyz) = 0.

Example 1.14. Solve (y + z)p + (x + z)q = x + y. [UQ]

Solution: Given (y + z)p + (x + z)q = x + y


dx dy dz
The subsidiary equations are = = (1)
y+z x+z x+y
dx + dy + dz dx − dy dy − dz
Each fraction of (1) = = =
2 (x + y + z) − (x − y) − (y − z)
d(x − y) d(y − z)
=
(x − y) (y − z)
Integrating, we get log(x − y) = log(y − z) + log a
x−y
i.e., =a
y−z
14 Part-B : Unit I - PARTIAL DIFFERENTIAL EQUATIONS (P.D.E.)

dx + dy + dz dx − dy
Taking = , we have
2 (x + y + z) − (x − y)
1 d(x + y + z) d(x − y)
=−
2 (x + y + z) (x − y)
1
Integrating, we get log(x + y + z) = − log(x − y) + log b
p 2
(x + y + z)(x − y) = b
 
x−y p
The general solution is φ , (x + y + z)(x − y) = 0.
y−z
Example 1.15. Solve (x2 − yz)p + (y 2 − zx)q = z 2 − xy. [UQ]

Solution: The subsidiary equations are


dx dy dz
2
= 2
= (1)
(x − yz) (y − zx) z2 − xy
Each fraction of (1) =
dx + dy + dz dx + dy + dz
= 2
(x2 2 2
− yz) + (y − zx) + (z − xy) x + y + z 2 − xy − yz − zx
2

Using x, y, z as multipliers, each fraction of (1) =


xdx + ydy + zdz dx + dy + dz
3 3 3
= 2 2 2
x +y +z − 3xyz x +y +z − xy − yz − zx
xdx + ydy + zdz dx + dy + dz
2 2 2
= 2 2 2
(x + y + z) (x + y + z − xy − yz − zx) x +y +z − xy − yz − zx
xdx + ydy + zdz dx + dy + dz
=
x+y+z 1
xdx + ydy + zdz = (x + y + z)d(x + y + z)

x2 y 2 z 2 (x + y + z)2
Integrating, + + = +c
2 2 2 2
xy + yz + zx = a
dx − dy dy − dz
Each fraction of (1) = 2 2
= 2
(x − yz) − (y − zx) (y − zx) − (z 2 − xy)
dx − dy dy − dz
i.e., 2 2
= 2
(x − y ) + z (x − y) (y − z 2 ) + x (y − z)
d (x − y) d (y − z)
=
(x − y) (x + y + z) (y − z) (x + y + z)
d (x − y) d (y − z)
=
(x − y) (y − z)
Integrating, log(x − y) = log(y − z) + log b
x−y
=b
y−z  
x−y
∴ The general solution is φ xy + yz + zx, = 0.
y−z
Example 1.16. Solve y 2 p − xyq = x(z − 2y). [UQ]
MA8353 Transforms and Partial Differential Equations by K A Niranjan Kumar 15

Solution: The subsidiary equations are

p − xyq = x(z − 2y) (1)


dx dy
Taking 2
= , we have
y −xy
dx dy
=
y −x
xdx = −ydy
x2 y2
Integrating, =− +c
2 2
2 2
i.e., x + y = a
dy dz
Taking = we have
−xy x (z − 2y)
dy dz
= ⇒(z − 2y)dy = −ydz
−y (z − 2y)
i.e., ydz + zdy − 2ydy = 0 ⇒d(yz) − 2ydy =0
Integrating, yz − y 2 = b

∴ The general solution is φ x2 + y 2 , yz − y 2 = 0.
Example 1.17. Solve (x2 − y 2 − z 2 )p + 2xyq − 2xz = 0. [UQ]
dx dy dz
Solution: The subsidiary equations are = = (1)
x2 2
−y −z 2 2xy 2xz
dy dz
Taking = , we have
2xy 2xz
dy dz
=
y z
log y = log z + log a
y
i.e., = a
z
Using x, y, z as multipliers
Each fraction of (1) =
xdx + ydy + zdz xdx + ydy + zdz
= 3
x (x2 2 2 2
− y − z ) + 2xy + 2xz 2 x + xy 2 + xz 2
xdx + ydy + zdz
=
x (x2 + y 2 + z 2 )
dy xdx + ydy + zdz dy 2 (xdx + ydy + zdz)
Taking = 2 2 2
⇒ =
2xy x (x + y + z ) y (x2 + y 2 + z 2 )

dy d x2 + y 2 + z 2
= 2 2 2
⇒ log y = log(x2 + y 2 + z 2 ) + log b
y (x + y + z )
y
i.e., =b
x2 + y2 + z2  
y y
∴ The general solution is φ , 2 = 0.
z x + y2 + z2
16 Part-B : Unit I - PARTIAL DIFFERENTIAL EQUATIONS (P.D.E.)

Example 1.18. Solve (x − 2z)p + (2z − y)q = y − x. [UQ]


dx dy dz
Solution: The subsidiary equations are = = (1)
x − 2z 2z − y y−x
dx + dy + dz dx + dy + dz
Each fraction of (1) = =
x − 2z + 2z − y + y − x 0
i.e., dx + dy + dz = 0
i.e., x + y + z = a
ydx + xdy ydx + xdy
Each fraction of (1) = =
xy − 2yz + 2xz − xy 2z(x − y)
ydx + xdy dz
Taking = , we have
2z (x − y) y−x

ydx + xdy = −2zdz


i.e., d(xy) = −2zdz

Integrating, xy = −z 2 + b
xy + z 2 = b

The general solution is φ x + y + z, xy + z 2 = 0.

Example 1.19. Solve (z 2 − 2yz − y 2 )p + (xy + zx)q = xy − zx. [UQ]

Solution: The subsidiary equations are


dx dy dz
= = (1)
z2 − 2yz − y 2 xy + zx xy − zx
Using x, y, z as multipliers each fraction of
xdx + ydy + zdz xdx + ydy + zdz
(1) = =
x (z 2 − 2yz − y 2 ) + y (xy + zx) + z (xy − zx) 0
we have

xdx + ydy + zdz = 0


x2 y 2 z 2
Integrating, we get + + = c1
2 2 2
x2 + y 2 + z 2 = a
dy dz
Taking = , we have
xy + zx xy − zx

dy dz
=
y+z y−z
i.e., (y − z)dy = (y + z)dz
ydy − zdy − ydz − zdz = 0
ydy − (ydz + zdy) − zdz = 0
ydy − d(yz) − zdz = 0
MA8353 Transforms and Partial Differential Equations by K A Niranjan Kumar 17

y2 z2
Integrating, − yz − = c2
2 2
i.e., y 2 − z 2 − 2yz = b

The general solution is φ x2 + y 2 + z 2 , y 2 − z 2 − 2yz = 0.

Example 1.20. Solve pz − qz = z 2 + (x + y)2 .


dx dy dz
Solution: The subsidiary equations are = = 2 (1)
z −z z + (x + y)2
dx dy
Taking = , we have dx = −dy
z −z
Integrating, x + y = a
dx dz
Taking = 2 , we have
z z + (x + y)2
dx dz
= 2 (∵ x + y = a)
z z + a2
zdz
i.e., dx = 2
z + a2
1
Integrating, x = log(z 2 + a2 ) + c
2
2x = log(z 2 + a2 ) + 2c
 
i.e., 2x − log z 2 + (x + y)2 = b
  
The general solution is φ x + y, 2x − log z 2 + (x + y)2 =0

1.2.5 Examples of Homogeneous Linear P.D.E.



Example 1.21. Solve the equation D 2 + 3DD ′ + 2D ′2 z = ex cosh y

Solution: The auxillary equation is

m2 + 3m + 2 = 0 ⇒ (m2 − 1)(m2 + 1) = 0
i.e., m = −1, −2
∴ C.F. is f1 (y − x) + f2 (y − 2x)
 y −y

1 x 1 x e +e
P.I. = e cosh y = e
D2 + 3DD′ + 2D′2 D2 + 3DD′ + 2D′2 2
1
h 1 1
i
= 2 ′ ′2
ex+y + 2 ex−y
2 D + 3DD + 2D D + 3DD′ + 2D′2
h
1 1 x+y 1 x−y
i
= e + e
2 6 0   
1 e x+y 1 1 ex+y 1 x−y
x−y
P.I. = +x e = +x e
2 6 2D + 3D′ 2 6 −1
 
1 ex+y
∴ P.I. = − xex−y
2 6
∴ The complete solution z = C.F. + P.I.
18 Part-B : Unit I - PARTIAL DIFFERENTIAL EQUATIONS (P.D.E.)

∂ 2z ∂ 2z
Example 1.22. Solve + = cos 2x cos 3y
∂x2 ∂y 2

Solution: Given D2 + D′2 z = cos 2x cos 3y
The auxillary equation is

m2 + 1 = 0 ⇒ m2 = −1
m = i, −i

∴ C.F. is f1 (y + ix) + f2 (y − ix)


1 1 1
P.I. = cos 2x cos 3y = 2 (cos(2x + 3y) + cos(2x − 3y))
D2
+D ′2 D +D 2 ′2
1
 1 1

= cos(2x + 3y) + cos(2x − 3y)
2 D2 + D′2 D2 + D′2
1
 1 1

= cos(2x + 3y) + cos(2x − 3y)
2 −4 − 9 −4 − 9
1 1
  4x   6y 
= − (cos(2x + 3y) + cos(2x − 3y)) = − 2 cos + cos
26 26 2 2
1
= − (cos 2xcos3y)
13
∴ The complete solution z = C.F. + P.I.


Example 1.23. Solve D 2 + 3DD ′ + 2D ′2 z = x2 y 2

Solution: The auxillary equation is

m2 + 3m + 2 = 0 ⇒ (m + 1)(m + 2) = 0
i.e., m = −1, −2

∴ C.F. = f1 (y − x) + f2 (y − 2x)
1 1
P.I. = x2 y 2 = h i x2 y 2
D2 ′
+ 3DD + 2D ′2
D2 1 + 3D
′ 2D′2
D + D2
  −1
1 3D′ 2D′2
= 2 1+ + x2 y 2
D D D2
"    2 #
1 3D′ 2D′2 3D′ 2D′2
= 2 1− + + + − . . . x2 y 2
D D D2 D D2
   
1 3D′ 2D′2 9D′2 2 2 1 3D′ 7D′2 2 2
= 1 − − + x y = 1 − + x y
D2 D D2 D2 D2 D D2
1
h 3 7
i 1 2 2 1 1
2 2 2 2 2
= x y − (2x y) + (2x ) = (x y ) − (6x y) + (14x2 )
D2 D D2 D2 D3 D4
1 4 2 1 5 7 6 1 x
h 1
i
= x y − x y+ x = [2 cos(x + y) + 3 cos(x + y)] − sin(x − y)
12 10 180 2 5  6 
1
h 1
i 1 m xm+n
P.I. = x cos(x + y) − sin(x − y) ∵ nx =
2 6 D (m + 1)(m + 1) . . . (m + n)
MA8353 Transforms and Partial Differential Equations by K A Niranjan Kumar 19

∴ The complete solution z = C.F. + P.I.


Example 1.24. Solve D 2 + 6DD ′ + 5D ′2 z = xy 4

Solution: The auxillary equation is

m2 + 6m + 5 = 0 ⇒ (m + 1)(m + 5) = 0
i.e., m = −1, −5

∴ C.F. = f1 (y − x) + f2 (y − 5x)

1 1
P.I. = xy 4 = h i xy 4
D2 ′
+ 6DD + 5D ′2 D2
5D′2 5D 6 D
′2 + 5 D′ +1
  −1
1 6D D2
= 1+ + xy 4
5D′2 5D′ 5D′2
"    2 #
1 6D D2 6D D2
= 1− + + + − . . . xy 4
5D′2 5D′ 5D′2 5D′ 5D′2
   
1 6D 4 1 4 6y 4
= 1 − xy = xy −
5D′2 5D′ 5D′2 5D′
1 4 6 4 xy 6 y7
= (xy ) − (y ) = −
5D′2 25D′3 150 875

∴ The complete solution z = C.F. + P.I.


Example 1.25. Solve D 2 − 4DD ′ + 4D ′2 z = ex−2y cos(2x − y)

Solution: The auxillary equation is

m2 − 4m + 4 = 0
i.e., m = 2, 2
20 Part-B : Unit I - PARTIAL DIFFERENTIAL EQUATIONS (P.D.E.)

∴ C.F. = f1 (y + 2x) + xf2 (y + 2x)


1
P.I. = ex−2y cos(2x − y)
D2− 4DD′ + 4D′2
1
= ex−2y cos(2x − y)
(D + 1) − 4(D + 1)(D′ − 2) + 4(D′ − 2)2
2

1
= ex−2y 2 cos(2x − y)
D + 10D − 4DD − 20D′ + 4D′2 + 25

1
= ex−2y cos(2x − y)
−4 + 10D − 8 − 20D′ − 4 + 25
1 ′
x−2y ((10D − 20D ) − 9) cos(2x − y)
= ex−2y cos(2x − y) = e
10D − 20D′ + 9 (10D − 20D′ )2 − 81
(10D − 20D′ − 9) cos(2x − y)
= ex−2y
100D2 − 400DD′ + 400D′2 − 81
(10D − 20D′ − 9)
= ex−2y cos(2x − y)
−400 − 800 − 400 − 81
ex−2y
=− (−20 sin(2x − y) − 20 sin(2x − y) − 9 cos(2x − y))
1681
1 x−2y
P.I. = e [40 sin(2x − y) + 9 cos(2x − y)]
1681
∴ The complete solution z = C.F. + P.I.

Example 1.26. Solve D 2 − DD ′ − 2D ′2 z = (y − 1)ex

Solution: The auxillary equation is

m2 − m − 2 = 0 ⇒ (m − 2)(m + 1) = 0
i.e., m = 2, −1
∴ C.F. = f1 (y + 2x) + f2 (y − x)
1 1
P.I. = (y − 1)ex = ex (y − 1)
D2 ′
− DD − 2D ′2 (D + 1) − (D + 1)D′ − 2D′2
2

1
= ex 2 (y − 1)
D + 2D + 1 − DD′ − D′ − 2D′2
 −1
= ex 1 + D2 + 2D − DD′ − D′ − 2D′2 (y − 1)
x
  
=e 1 − D2 + 2D − DD′ − D′ − 2D ′2
+ . . . (y − 1)
 
= ex 1 + DD′ + D′ (y − 1) = ex [y − 1 + D(1) + 1] = ex y
∴ The complete solution z = C.F. + P.I.
i.e., z = f1 (y + 2x) + f2 (y − x) + yex .

Example 1.27. Solve D 2 + DD ′ − 6D ′2 z = y cos x

Solution: The auxillary equation is

m2 + m − 6 = 0 ⇒ (m + 3)(m − 2) = 0
i.e., m = 2, −3
MA8353 Transforms and Partial Differential Equations by K A Niranjan Kumar 21

∴ C.F. = f1 (y + 2x) + f2 (y − 3x)


1 1
P.I. = y cos x = y cos x
D2 ′
+ DD − 6D ′2 (D − 2D )(D + 3D′ )

Z
1
= (c + 3x) cos xdx where y = c + 3x
D − 2D′
1
= [(c + 3x) sin x + 3 cos x]
D − 2D′
1
= [(y − 3x + 3x) sin x + 3 cos x] ∵ c = y − 3x
D − 2D′
1
= ′
(y sin x + 3 cos x)
ZD − 2D
= [(c1 − 2x) sin x + 3 cos x]dx where y = c1 − 2x

= [(c1 − 2x)(− cos x) − (−2)(− sin x)] + 3 sin x


= −(c1 − 2x) cos x − 2 sin x + 3 sin x = −y cos x + sin x
P.I. = sin x − y cos x

∴ The complete solution z = C.F. + P.I.

1.3 Assignment I[Partial Differential Equations]

1. (i) Form a partial differential equation by eliminating a and b from the expression
(x − a)2 + (y − b)2 + z 2 = c2 .
(ii) Find the partial differential equation of all planes which are at a constant distance a1 from
the origin.

2. (i) Form the partial differential equation by eliminating arbitrary function from
z = xf (2x + y) + g(2x + y).
(ii) Form the PDE by eliminating the arbitrary function φ from
2 2 2

φ x + y + z , ax + by + cz = 0.

3. Solve the following:


(i) p(1 + q)z = qz
(ii) x2 p + y 2 q = 0 (iii) xp + yq = 0

4. Solve the following:


(i) z = px + qy + log pq (ii) z = px + qy + pq
p
(iii) z = px + qy + 1 + p2 + q 2 (iv) z = px + qy + p2 q 2

5. Solve the following:


(i) p(1 + q) = qz (ii) x4 p2 − yzq = z 2
22 Assignment : Unit I - PARTIAL DIFFERENTIAL EQUATIONS (P.D.E.)


(iii) x4 p2 + y 2 qz = 2z 2 (iv) z 2 p2 x2 + q 2 = 1

6. Solve the following:



(i) p2 + q 2 = x2 + y 2 (ii) z 2 p2 + q 2 = x2 + y 2

(iii) 4z 2 q 2 = y + 2zp − x (iv) p2 + q 2 = z 2 x2 + y 2

7. Solve the following:


 
(i) x2 (y − z)p = y 2 (z − x)q = z 2 (x − y) (ii) x2 − yz p + y 2 − zx q = z 2 − xy

(iii) (y − xz)p + (yz − x)q = (x + y)(x − y) (iv) x2 − y 2 − z 2 p + 2xyq = 2zx
  
(v) x z 2 −y 2 p+y x2 −z 2 q = z y 2 − x2 (vi) (3z −4y)p+(4x−2z)q = (2y−3x)
  
(vii) x y 2 + z p + y x2 + z q = z x2 − y 2

8. Solve the following:



(i) D3 + D2 D′ − DD′2 − D′3 z = e2x+y + cos(x + y)
∂3z ∂ 3z
(ii) − 2 = ex+2y + 4 sin (x + y)
∂x3 ∂x2 ∂y 
(iii) D3 − 7DD′2 − 6D′3 z = cos (x + 2y) + 4

(iv) D2 + 2DD′ + D′2 z = sinh(x + y) + ex+2y

(v) D2 + 2DD′ + D′2 z = x2 y + ex−y

(vi) D2 − DD′ − 2D′2 z = 2x + 3y + e3x+4y

(vii) D2 − D′2 z = ex−y sin(x + 2y)
2 ′ ′2

(viii) D − 5DD + 6D z = y sin x
(ix) r + s − 6t = y cos x

9. Solve the following:



(i) D2 − D′2 − 3D + 3D′ z = xy + 7

(ii) D2 + 2DD′ − 2D − 2D′2 z = sin(x + 2y)

(iii) 2D2 − DD′ − D′2 + 6D + 3D′ z = xey

(iv) D2 + 2DD′ + D′2 − 2D − 2D z = e3x+y + 4 ′

(v) D2 + D′2 + 2DD′ + 2D + 2D′ + 1 z = e2x+y
2 Fourier Series(F.S.)

2.1 Part-A

1. Find the constant term in the Fourier series discontinuity in the extremum.
corresponding to f (x) = cos2 x expressed in
the interval (−π, π).
Soln : f (−2) + f (2)
f (x = 2) =
2 
1 + cos 2x
Givenf (x) = cos2 x = x2 + x x=−2 + x2 + x x=2
2 =
2
f (−x) = cos (−x) = cos2 x = f (x)
2
(4 − 2) + (4 + 2)
= =4
2
Therefore the function is even.
Fourier series is

a0 X
f (x) = + an cos nx
2
n=1 3. If the Fourier series of f (x) = x2 in −π <
Here the constant term is a0 x < π is equal to
X∞
Z π2 (−1)n cos nx
1
π f (x) = +4 , prove that
i.e., a0 = cos2 x dx 3 n2
n=1
π ∞
Z−π
π
X 1 π2
2 = .
= cos2 xdx n2 6
π n=1
Z0 π Soln : Given f (x) = x2 , f (x) =
2 1 + cos 2x ∞
= dx π2 X (−1)n cos nx
π 0 2 +4 , (1)
h i 3 n2
1 sin 2x x=π n=1
= x+ Put x = π (a point of discontinuity in the
π 2 x=0
1
h sin 2π
 i
= π+ − (0 + 0) extremum)
π 2
= 1.

f (x = −π) + f (x = π)
2. If f (x) = x2 + x is expanded as a Fourier f (x = π) = = π2
2
series in the interval (−2, 2) to which value π2 + π2
=
this series converges at x = 2? 2
Soln : Here given x = 2 is a point of = π2
24 Part-A : Unit II - FOURIER SERIES (F.S.)

X (−1)n cos nπ ∞
π2
2
5. State Dirichlet’s conditionin Fourier
∴ (1) ⇒ π = +4 1
3 n2 series and examine whether can be
n=1 1−x
π2

X (−1) (−1)n
n expanded in a Fourier series in any interval
2
⇒π − =4 including the point x = 1.
3 n2
n=1
X∞ Soln : The expansion of a function f (x) in
2π 2 (−1)2n
⇒ =4 a trigonometry is possible if it satisfies the
3 n2
n=1

following condition in any interval.
π2 X 1
⇒ =
6 n2
n=1 a) f (x) is well defined and single valued
Hence Proved. function.

4. If f (x) = |cos x| is expanded in a Fourier


b) f (x) has finite number of points of
series in the interval of −π < x < π, find
continuity.
a0 .
Soln : Given f (x) = |cos x|
c) f (x) has only finite number of maxima
f (−x) = |− cos x|
and minima.
= |cos x| = f (x)
Therefore the function s even.
f (x) is not well defined
 1 at  x = 1,
⇒ bn = 0.
since f (x = 1) = lim = ∞.
∴ Fourier series in −π < x < π is x→1 1 − x

a0 X So f (x) can not be expanded in a Fourier
f (x) = + an cos nx
2 series in any interval including the point
n=1
To find a0 , x = 1.


1
a0 = f (x) dx
π
−π 6. Find the constant term in the Fourier series
Zπ corresponding to f (x) = 2x − x2 expressed
2
= |cos x| dx
π in the interval (−2, 2).
0
(Z
π/2
2
= cos xdx+
π 0 Soln : Given f (x) = 2x − x2
Z π 
(− cos x) dx f (−x) = 2 (−x) − (−x)2
π/2
n o = −2x − x2
2 π/2 (
= [sin x]0 + [− sin x]ππ/2
π f (x)
2 6=
= [(1 − 0) + (−0 + 1)] −f (x)
π
4
=
π
∴ The function is neither even nor odd.
MA8353 Transforms and Partial Differential Equations by K A Niranjan Kumar 25

To find constant a0 , To find the constant term a0 ,


Z Z π
1
π 1
a0 = f (x) dx a0 = f (x) dx
π π
Z−π Z−π
π
1
π  2
= 2x − x2 dx = f (x) dx
π π 0
−π 2 π Z π
2 √
1 2x x3 = 1 − cos xdx
= − π 0
π 2 3 −π Z r
h π  
1  1 3 i =
2
2 sin2
x
dx
= π2 − π2 − π + π3 π 0 2
π 3 Z π
3 2 √ x

=− = 2 sin dx
3 π 0 2
 √ " x
 #π
2 2 − cos 2
7. The function f (x) = {−π, −π < x < = 1
π 2 0
0&x, 0 < x < π is expanded as a Fourier √ h i
4 2 π
series of period 2π. What is the sum of the = − cos + cos 0
π √π 2
series at x = 0 and x = . 4 2
2 = [0 + 1]
Soln : ( Given √π
−π, −π < x < 0 4 2
f (x) = =
x, 0<x<π π
Here x = 0 is a point of discontinuity in
9. (
If the Fourier series for the function f (x) =
middle.
0, 0 < x < π
Sum of the series at (x = 0) is is
sin x, π < x < 2π
h
−1 2 cos 2x cos 4x
i sin x
f (0−) + f (0+) f (x) = + + + ... + .
f (x = 0) = π π 1.3 3.5 2
2 1 1 π−2
(−π) + 0 −π Deduce that − + ... =
= = . 1.3 3.5( 4
2 2 0, 0 < x < π
Soln : Given f (x) =
Here x = π/2 is a point of continuity in sin x, π < x < 2π
1 1 π−2
second interval. Let − + ... = (1)
1.3 3.5 4
Sum of the series at (x = π/2) is π
Put x = is a point of continuity,
h i π 2
f (x = π/2) = x = ∴ f x = π2 = 0.
π 2
2
−1 2
h
1 1
i 1
∴ (1) ⇒ 0 = + − + − ... +
8. Obtain the constant term in the π
1 1 2
h 1π 1.3 3.5
1
i 2
Fourier series corresponding to f (x) = − + = − + − ...
p 2 π π 1.3 3.5
(1 − cos x) in the integral of (−π, π). −π + 2
h
2 1 1
i
p =− − + ...
Soln : Gn. f (x) = (1 − cos x)
p h π − 2π i
2 π
h π 1.3 3.5
1 1
i
f (−x) = [1 − cos (−x)] = − + ...
√ h2π
π−2
2
i h 1.3 3.5
1 1
i
= 1 − cos x = f (x)
= − + ...
Therefore the function is even. 4 1.3 3.5
26 Part-A : Unit II - FOURIER SERIES (F.S.)

10. To what value , the half range sine series The half range Fourier sine series in
corresponding to f (x) = x2 expressed in the (0, ℓ = 1) is
interval (0, 2) converges at x = 2. ∞
X  nπx 
Soln : Given f (x) = x2 . f (x) = bn sin

n=1
Here x = 2 is a point of discontinuity at
extreme. where Z
2
ℓ  nπx 
f (x = −2) + f (x = 2) bn = f (x) sin dx
f (x = 2) = ℓ 0 ℓ
  2   here ℓ = 1
x2 + x2 Z
x=−2 x=2 1
=
2 ∴ bn = 2 ex sin (nπx) dx
(−2)2 + (2)2 0
= ex
2 =2
4+4 8 12 + (nπ)2
= = x=1
2 2
=4 [sin (nπx) − nπ cos (nπx)]
x=0

e1
11. If the Fourier series corresponding to =2 (sin nπ−nπ cos nπ)
12 +(nπ)2
f (x ) = x in the interval (0, 2π) is 
∞ 1
a0 X − (−nπ)
+ (an cos nx + bn sin nx) without 12 +(nπ)2
2 
n=1 e1  n

finding the values of a0 , an , bn find the value =2 −nπ (−1)
∞ 12 + (nπ)2
a20 X 2  
of + an + b2n . 1
2 − [−nπ]
n=1
Soln : By Parseval’s theorem 12 + (nπ)2
 
nπ  n 
Z ∞ =2 −e (−1) + 1
a2 X 2
2π 
1 2 12 + (nπ)2
[f (x)] dx = 0 + an + b2n
π 0 2
n=1

X Z 2π Fourier series is
a20  1 2
+ a2n + b2n = [f (x)] dx
2 π 0
n=1
Z ∞ 
X 
1
2π 2nπ  
= [x]2 dx f (x) = 2
−e (−1)n + 1 sin (nπx)
π 0 n=1
12 + (nπ)
 2π
1 x3
= 13. The cosine series for f (x) = x sin x f or 0 <
π 3 0
1 3 x < π is given as the
= 8π X∞
3π 1 (−1)n
8π 2 x sin x = 1 − cos x − 2 cos nx
= 2 n2 − 1
3 h 1n=2 1 i π
and deduce that 1 + 2 − + ... = .
1.3 3.5 2
12. Find the half range sine series for f (x) = ex Soln : Given
X∞
in 0 < x < 1. 1 (−1)n
x sin x = 1 − cos x − 2 cos nx
Soln : Given f (x) = ex . 2 n2 −1
n=2
MA8353 Transforms and Partial Differential Equations by K A Niranjan Kumar 27

π
Put x = (point of continuity)
2
X (−1)n ∞
π π 1 π π 16. State whether true or false: Fourier series of
sin = 1− cos −2 cos n
2 2 2 2 n2 −1 2 period 2 for x sin x in (−1, 1) contains only
n=2
X (−1) ∞ n
π π sine terms. Justify your answer.
=1−2 2
cos n
2 n −1 2 Soln : False, the Fourier series doesn’t
n=2
X∞ contain sine terms. Since,
π (−1)n π
=1−2 cos n
2 (n + 1) (n − 1) 2 f (−x) = (−x) sin (−x) = (−x) (− sin x) =
 n=2
π 1 −1 x sin x = f (x)
= 1−2 (−1)+ (0)
2 (3) (1) (4) (2)
 Therefore the function is even.
1
+ (1)+... ∴ Fourier series is
(5) (3) a0 X
∞  nπx 
π
h 1 1
i f (x) = + an cos .
∴ = 1+2 − +... 2 ℓ
n=0
2 1.3 3.5 So, it contains only cosine terms in the
Hence deduced.
series.
14. If f (x) = sin2 x, −π < x < π, then find
17. The Fourier series for f (x) = x2 in −π <
b21 + b22 + b23 + · · ·.
Soln : Given f (x) = sin2 x, −π < x < π x < π will contain only cosine terms. State
whether true or false. Justify your answer.
f (−x) = sin2 (−x) = (− sin x)2
Soln : True , the Fourier series contain only
= sin2 x = f (x)
cosine terms.
∴ The function is even.
Since, f (−x) = (−x)2 = x2 = f (x)
This implies bn = 0.

X ∞
X Therefore the function is even.

Therefore, b2n = (0)2 = 0 a0 X
n=1 n=1
∴ Fourier series is f (x) = + an cos nx
2
This implies b21 + b22 + b23 + · · · = 0 n=0
So, it contains only cosine terms in the
15. If f (x) = x − x2 in (−1, 1), find the RMS series.
value of f (x).
18. Find the Half range Fourier sine series of
Soln : Given f (x) = x − x2 .
(i) f (x) = x in (0, 2)
The RMS value in an integral (−1, 1). (ii) f (x) = 1, 0 < x < π. (Apr ’04)
Soln :
v v
u Z1 u Z1
u1 u1
u u
2
y=t [f (x)] dx = t [x − x2 ]2 dx
2 2 (i) Given f (x) = x in (0, 2).
−1 −1
v
u Z1 s  1
Half range Fourier sine series in (0, ℓ = 2) is
u1 1 x3 x5 x4
u
=t [x +x −2x ]dx =
2 4 3 + −2 ∞
2 2 3 5 4 −1 X nπx
f (x) = bn sin
−1


s    
1 1 1 1 −1 −1 1 n=1
= + − − + −
2 3 5 2 3 5 2 ∞
s   s
X nπx
= bn sin
 r
1 2 2 1 1 8
=
2 3
+
5
=
3
+
5
=
15 2
n=1
28 Part-A : Unit II - FOURIER SERIES (F.S.)

where where

Zℓ
2 nπx
Zℓ bn = f (x) sin dx
2 nπx ℓ ℓ
bn = f (x) sin dx 0
ℓ ℓ
0 Zπ
2
Z2 Z2 = 1 sin nxdx
2 nπx nπx π
= f (x) sin dx = x sin dx 0
2 2 2

0  nπx 
0
2
− cos = sin nxdx
= (x) 2 π

0
2
!)2 h
2 − cos nx π
i
− sin nπx =
−(1) 2 π n 0

nπ 2 −2  
 2
 0
 = (−1)n − [1]
n nπ
(−1) (
= −2 nπ + 0 − [0 + 0] 0, n is even
2 = 4
n , n is odd
(−1) nπ
= −4

Half range Fourier sine series is,

X
∴ Half range Fourier sine series is nπx
f (x) = bn sin

n=1


4 X 1
X nπx = sin nx
f (x) = bn sin π n
n=odd
ℓ ∞
n=1
4 X 1
∞ = sin nx
4 X (−1)n nπx π n
=− sin n=1,3,5,···
π n 2
n=1

19. Find the constant term in the Fourier cosine


π − x2
series corresponding to f (x) =
(ii) Given f (x) = 1 in (0, π). 2
expressed in the interval (0, 3).
Half range Fourier sine series in (0, ℓ = π) is
π − x2
Soln : Given f (x) = .
2
Fourier cosine series in (0, ℓ = 3) is

X nπx
f (x) = bn sin ∞
n=1
ℓ a0 X nπx
f (x) = + an cos
X∞ 2 ℓ
nπx n=0
= bn sin
π
n=1
X∞ ∞
a0 X nπx
= bn sin nx = + an cos
2 3
n=1 n=0
MA8353 Transforms and Partial Differential Equations by K A Niranjan Kumar 29

Zℓ 23. Define complex form of Fourier series.


2
where, a0 = f (x)dx Soln : Let f (x) be a function defined in

0 interval (0, 2π) and satisfies the Dirichlet’s
Z3 conditions, then we have a Fourier series,
2 (π − x2 )
= dx ∞
3 2 a0 X
0 f (x) = + (an cos nx + bn sin nx)
 3 2
1 x3 n=1
= πx − eix + e−ix eix − e−ix
3 3 0 If cos nx = & sin nx = ,
1 2 2i
= [(3π − 9) − (0 − 0)] then
3
The complex form of Fourier series is,
=π−3

X
20. State Parseval’s theorem on Fourier f (x) = Cn einx
n=−∞
coefficient.
Soln : If the Fourier series corresponding
Z2π
1
where Cn = f (x) e−inx dx
to f (x) converges uniformly to f (x) in an 2π
0
integral (−π, π) then,
Zπ ∞ 24. What is Harmonic analysis?
1 2 a20 X 2 
[f (x)] dx = + an + b2n Soln : The process of finding Fourier series
π 2
−π n=1 for a function f (x) given by numerical values
21. State Parseval’s identity for Half range is known as “Harmonic Analysis” Consider
Fourier cosine series (HRFCS) and Half the Fourier series in an interval of f (x) as,
range Fourier sine series (HRFSS) a0
f (x) = +a1 cos x+a2 cos 2x+· · ·+
Soln : Parseval’s identity for Half range 2
Fourier cosine series, since bn = 0, b1 sin x+b2 sin 2x+· · ·
Zπ ∞
2 a2 X 2 
2 where,
[f (x)] dx = 0 + an
π 2
0 n=1 a0 = 2 [Mean value of f (x)]
2 X
h i
For Half range Fourier sine series, a0 =
= f (x)
0, an = 0 n
Zπ ∞ an = 2 [Mean value of f (x) cos nx]
2 X  h i
[f (x)] dx = 2
b2n 2 X
π = f (x) cos nx
0 n=1 n
bn = 2 [Mean value of f (x) sin nx]
22. Define root mean square value of f (x). h i
2 X
Soln : The root mean value of the function = f (x) sin nx
n
y = f (x) over the interval (a, b) and it is ∞
π2 X (−1)n
defined as, 2
25. If x = +4 cos nx , show that
v 3 n2
u n=1
u Zb
u 1 π4 1 1 1
y=t [f (x)]2 dx = 4 + 4 + 4 + · · · in −π < x < π .
(b − a) 90 1 2 3
a Soln : Given
30 Part-A : Unit II - FOURIER SERIES (F.S.)


π2 X (−1)n
2
Soln : (i) Given y = x2 (−π, π)
f (x) = x = +4 cos nx
3 n2 The RMS value in an interval (−π, π) is
n=1
n v
a0 π2 4 (−1) 2π 2 u
Here = ⇒ a0 = , an = u Zb
2 3 3 n2 u 1
By Parseval’s identity, y=t [f (x)]2 dx
(b − a)
a
Zπ v v
1 a20

X  u Zπ u Zπ
[f (x)]2 dx = + a2n u u
π 2 u1 2 2 u1
n=1
=t [x ] dx = t x4 dx
−π 2π 2π
 
2 2
−π −π
Zπ 2π ∞ 
X n 2 v
1  2 2 3 4 (−1) u Zπ s  
x dx = + u π
π 2 n2 u2 4
1 x5
n=1 =t x dx =
−π
  2π π 5 0
0
Zπ ∞  2n 
4

2 9 X 16 (−1) r r
x4 dx = + 1 π5
π 2 n4 = [(π 5 ) − (05 )] =
0 n=1 5π 5π
  r
2 x5 π 4π 4 X 1 ∞   π4 π2
= + 16 = =√
π 5 0
18 n4 5 5
n=1
 ∞ 
X  (ii) Given y = x + x2 (−π, π) The RMS
2 2π 4 1
π5 = + 16 value in an interval (−π, π) is
5π 9 n4
n=1 v
 X∞   u
2 4 2π 4 1 u Zb
π = + 16 u 1
5 9 n4 y=t [f (x)]2 dx
n=1 (b − a)
∞ 
X  a
2 4 2π 4 1 v
π − = 16 u Zπ
5 9 n4 u
n=1 u1 2
X
∞  =t [x + x2 ] dx
1 4 π4 1 2π
π − =8 −π
5 9 n4 v
n=1 u Zπ
1  ∞ 
X  u
1 1 u1
π4 − =8 =t (x2 + x4 + 2x3 )dx
5 9 n4 2π
n=1 −π
9 − 5 X∞ 
1
 v
π4 =8 u Zπ Zπ
45 n4
u
u2 1
n=1 =t (x2 + x4 )dx + (2x3 )dx
4 X∞   2π 2π
4 1 0 −π
π =8 s  s 
45 n4 π 
n=1 1 x3 x5 1 π3 π5
π4 X

1
 =
π 3
+
5
+0=
π 3
+
5
= 0
90 n4 r r
n=1 5π 3 + 3π 5 5π 2 + 3π 4
π4 1 1 1 = =
= 4 + 4 + 4 + ··· 15π 15
90 1 2 3
27. What is the equivalent Fourier constant to
2
26. Find the R.M.S. value of y = x in (−π, π) the expression 2π mean value of f (x) =
. And x + x2 in (−π, π). cos nx in (c, c + 2π).
MA8353 Transforms and Partial Differential Equations by K A Niranjan Kumar 31

Soln : If f (x) is a periodic function with a To find Fourier constant a0 :


period 2π and if f (x) can be represented in
a trigonometry series, Z
c+2π Z
c+2π
1 1
∞ a0 = f (x)dx = cos nxdx
a0 X π π
f (x) = + (an cos nx + bn sin nx) c c
2
n=1  
1 sin nx c+2π
where, =
π n c
Z
c+2π
1
1 = (sin n(c + 2π) − sin nc)
a0 = f (x)dx nπ
π 1
c = [sin nc cos 2nπ + cos nc sin 2nπ − sin nc]
Z
c+2π nπ
1 1
an = f (x) cos nxdx = [sin nc(1) − sin nc] = 0
π nπ
c
Z
c+2π
1
bn = f (x) sin nxdx
π
c

2.2 Part-B

2.2.1 Examples under (0, 2π)

Example
n 2.1. Find the Fourier
o series of f (x) = x2 in (0, 2π) and with period 2π.
8 2 4 −4π
a0 = π , an = 2 , b n =
3 n n

Solution : Given f (x) = x2 defined in the interval (0, 2π).


∴ The Fourier series of f (x) is given by

a0 X
f (x) = + (an cos nx + bn sin nx) (1)
2
n=1
Z2π
1
where a0 = f (x) dx
π
0
Z2π
1
an = f (x) cos nxdx
π
0
Z2π
1
bn = f (x) sin nxdx
π
0
32 Unit II - FOURIER SERIES (F.S.)

Z2π    
1 1 x3 1 8π 3 8π 2
Now, a0 = x2 dx = = −0 =
π π 3 π 3 3
0

Z2π
1
an = x2 cos nxdx
π
0
1
h  sin nx   − cos nx   − sin nx i2π
2
= x − 2x +2
π n n2 n3 0
h
1 2 sin nx cos nx
i
sin nx 2π
= x + 2x 2 − 2 3
π n n n
1
h 4π
 i 0
= 0 + 2 − 0 − (0 + 0 − 0) [∵ cos 2nπ = 1]
π n
4
∴ an = 2
n

Z2π
1
bn = x2 sin nxdx
π
0
1
h  − cos nx   − sin nx   cos nx i2π
= x2 − 2x +2
π n n2 n3 0
1
h cos nx sin nx
i
cos nx 2π
= −x2 + 2x 2 + 2 3
π  n n
 n 0
1 −4π 2 2
 2

= +0+ 3 − 0+0+ 3
π n n n
 
1 −4π 2
=
π n
−4π
∴ bn =
n

Sub. the value of a0 , an , bn in (1)

  ∞ 
X 
1 8π 2 4 4π
f (x) = + cos nx − sin nx
2 3 n2 n
n=1

4π 2
∞ 
X cos nx π sin nx 
= +4 −
3 n2 n
n=1

(
x, (0, π)
Example 2.2. Find the Fourier series of periodicity 2π for f (x) =
2π − x, (π, 2π)
(
x, (0, π)
Solution: Given f (x) =
2π − x, (π, 2π)
Since the function f (x) is defined in the interval (0, 2π).
MA8353 Transforms and Partial Differential Equations by K A Niranjan Kumar 33

∴ The Fourier series of f (x) is given by


a0 X
f (x) = + (an cos nx + bn sin nx) (1)
2
n=1
Z2π Z2π Z2π
1 1 1
where a0 = f (x) dx,an = f (x) cos nxdx, bn = f (x) sin nxdx
π π π
0 0 0

 
Zπ Z2π
1
Now, a0 = xdx + (2π − x) dx
π
0 π
"   2π #
1 x 2 π x2
= + 2πx −
π 2 0
2 π
    
1 π2 2 2
 π2
2
= −0 + 4π − 2π − 2π −
π 2 2
   
1 π2 3π 2 1 π 2 + 4π 2 − 3π 2
= + 2π 2 − =
π 2 2 π 2
 
1 2π 2
=
π 2
a0 = π

 
Zπ Z2π
1
an = x cos nxdx + (2π − x) cos nxdx
π
π
h0    − cos nx iπ h  sin nx   − cos nx i2π 
1 sin nx
= x −1· + (2π − x) − (−1)
π n n2 0 n n2 π
h i h i 
1 x sin nx cos nx π sin nx cos nx 2π
= + + (2π − x) −
π n n2 0 n n2 π
 n      
1 (−1) 1 1 (−1)n
= 0+ − 0+ 2 + 0− − 0−
π n2 n n2 n
 
1 (−1)n 1 1 (−1)n
= − − +
π n2 n2 n2 n2
 
1 2 (−1)n 2
= 2
− 2
π n n
2  
an = 2 (−1)n − 1
n π
34 Unit II - FOURIER SERIES (F.S.)

 
Zπ Z2π
1
bn = x sin nxdx + (2π − x) sin nxdx
π
π
h0    − sin nx iπ h  − cos nx   − sin nx i2π 
1 − cos nx
= x −1 + (2π − x) − (−1)
π n n2 0 n n2 π
h i h i2π 
1 −x cos nx sin nx π cos nx sin nx
= + + − (2π − x) −
π n n2 0 n n2 π
 n     n 
1 −π (−1) −π (−1)
= +0 − (0 + 0) + (0 − 0) − −0
π n n
 
1 −π (−1)n π (−1)n
= +
π n n
bn = 0

Sub. the value of a0 , an and bn in (1)

π X 2
∞   
n
f (x) = + (−1) − 1 cos nx + 0
2 n2 π
n=1
X∞  
π 2 (−1)n − 1
f (x) = + cos nx
2 π n2
n=1

Example 2.3. Find the Fourier series for f (x) = x sin x in (0, 2π) and deduce
1 1 1 π−2
− + − ··· = .
(1)(3) (3)(4) (5)(7) n 4 o
−2 −1 π
a0 = −2, an = (n 6= 1) , a1 = , bn = 0 (n 6= 1) , b1 = π&x =
1−n2 2 2
Solution : Given f (x) = x sin x in (0, 2π).
∴ The Fourier series of f (x) is given by

a0 X
f (x) = + (an cos nx + bn sin nx) (1)
2
n=1
Z2π Z2π Z2π
1 1 1
where a0 = f (x) dx, an = f (x) cos nxdx, bn = f (x) sin nxdx
π π π
0 0 0

Z2π
1
Now, a0 = x sin xdx
π
0
1
= [x (− cos x) − 1 (− sin x)]2π
0
π
1
= [−x cos x + sin x]2π
0
π
1
= [(−2π + 0) − (0 + 0)]
π
a0 = −2
MA8353 Transforms and Partial Differential Equations by K A Niranjan Kumar 35

Z2π
1
an = x sin x cos nxdx
π
0
Z2π
1
= x cos nx sin xdx
π
0
Z2π
1 1
= x (sin (n + 1) x − sin (n − 1) x) dx
π 2
0
Z2π
1
= x (sin (n + 1) x − sin (n − 1) x) dx

0
    2π
1 − cos (n + 1) x cos (n − 1) x − sin (n + 1) x sin (n − 1) x
= x + −1 +
2π n+1 n−1 (n + 1)2 (n − 1)2 0
   2π
1 cos (n − 1) x cos (n + 1) x sin (n + 1) x sin (n − 1) x
= x − + −
2π n−1 n+1 (n + 1)2 (n − 1)2 0
1
hh  1 1
 i i
= 2π − + 0 − 0 − [0 + 0 − 0]
2π n−1 n+1
1 1 n + 1 − (n − 1)
= − =
n−1 n+1 n2 − 1
2
an = 2 , n 6= 1
n −1

Z2π
1 1
R2π
a1 = x sin x cos xdx ( ∵ an = π x sin x cos nxdx)
π 0
0
Z2π
1
= x2 sin x cos xdx

0
Z2π
1
= x sin 2xdx

0
1
h  − cos 2x   − sin 2x i2π
= x −1·
2π 2 4 0
h
1 −x cos 2x sin 2x
i 2π
1
h −2π
 i
= + = + 0 − (0 + 0)
2π 2 4 0 2π 2
−1
a1 =
2
36 Unit II - FOURIER SERIES (F.S.)

Z2π
1
bn = x sin x sin nxdx
π
0
Z2π
1
= x sin nx sin xdx
π
0
Z2π
1 1
= x (cos (n − 1) x − cos (n + 1) x)dx
π 2
0
Z2π
1
= x (cos (n − 1) x − cos (n + 1) x)dx
π
0
    2π
1 sin (n − 1) x sin (n + 1) x − cos (n − 1) x cos (n + 1) x
= x − −1 +
2π n−1 n+1 (n − 1)2 (n + 1)2 0
    2π
1 sin (n + 1) x sin (n − 1) x cos (n − 1) x cos (n + 1) x
= x − + 2

2π n+1 n−1 (n − 1) (n + 1)2 0
   
1 1 1 1 1
= 0+ − − 0+ −
2π (n − 1)2 (n + 1)2 (n − 1)2 (n + 1)2
bn = 0, n 6= 1

Z2π Z2π
1 1
b1 = x sin x sin xdx = x sin2 xdx
π π
0 0
Z2π
1 1
= x (1 − cos 2x) dx
π 2
0
Z2π
1
= (x − x cos 2x) dx

0
 h    i2π
1 x2 sin 2x − cos 2x
= − x −1
2π 2 2 4 0
 2π
1 x2 x sin 2x cos 2x
= − −
2π 2 2 4 0
   
1 1 4π 2 1
= −0− − 0−0−
2π 2 4 4
1 
= 2π 2

b1 = π

From (1),
MA8353 Transforms and Partial Differential Equations by K A Niranjan Kumar 37


a0 X
f (x) = + (an cos nx + bn sin nx)
2
n=1
X∞ ∞
X
a0
= + an cos nx + bn sin nx
2
n=1 n=1

X ∞
X
a0
= + a1 cos x + an cos nx + b1 sin x + bn sin nx
2
n=2 n=2
∞ 
X 
1 1 2
= (−2) − cos x + cos nx + π sin x + 0
2 2 n2 −1
n=2

P
∞ 
i.e., f (x) = −1 − 12 cos x + 2 cos nx
n2 −1
+ π sin x
n=2

Example 2.4. Obtain Fourier series for f (x) = eax in (0, 2π).

Solution : Given f (x) = eax in (0, 2π).


The Fourier series of f (x) is given by

a0 X
f (x) = + (an cos nx + bn sin nx) (1)
2
n=1
where

Z2π Z2π Z2π


1 1 1
a0 = f (x) dx, an = f (x) cos nxdx, bn = f (x) sin nxdx
π π π
0 0 0

Z2π  2π
1 ax 1 eax 1 ax 2π 1  a2π 
Now, a0 = e dx = = [e ]0 = e − e0
π π a 0
aπ aπ
0
1 
∴ a0 = e2aπ − 1

Z2π  2π
1 1 eax
an = eax cos nxdx = (a cos nx + n sin nx)
π π a2 + n2 0
0 
1 e2aπ 1
= 2 2
(a + 0) − 2 (a + 0)
π a +n a + n2
 
1 ae2aπ a a  2aπ 
= − = e − 1
π a2 + n 2 a2 + n2 π (a2 + n2 )

a e2aπ − 1
∴ an =
π (a2 + n2 )
38 Unit II - FOURIER SERIES (F.S.)

Z2π  2π
1 1 eax
bn = eax sin nxdx = (a sin nx − cos nx)
π π a2 + n2 0
0 
1 e2aπ 1
= 2 2
(0 − n) − 2 (0 − n)
π a +n a + n2
 
1 −ne2aπ n −n  2aπ 
= + = e − 1
π a2 + n 2 a2 + n2 π (a2 + n2 )

−n e2aπ − 1
∴ bn =
π (a2 + n2 )
Sub. the value of a0 , an , bn in (1)
  ∞
"   #
1 e2aπ−1 X a e2aπ − 1 n e2aπ − 1
f (x) = + cos nx − sin nx
2 aπ π (a2 + n2 ) π (a2 + n2 )
n=1

X  
e2aπ − 1 e2aπ −1
= + [a cos nx − n sin nx]
2aπ π (a + n2 )
2
n=1
∞ 
" #
e2aπ −1 e2aπ−1 X 1

= + (a cos nx − n sin nx)
2aπ π a2 + n2
n=1
" ∞
#
e2aπ −1 1 X 1
= + (a cos nx − n sin nx)
π 2a a2 + n2
n=1

2.2.2 Examples under (−π, π)

Example 2.5. Find the Fourier series for the function f (x) = x2 in [−π, π]with period 2π and
deduce
1 1 1 π2 1 1 1 π2
(i) 2 + 2 + 2 + ... = −(ii) + − ... =
1 2 3 6 12 22 32 12
1 1 1 π2 1 1 1 π4
(iii) 2 + 2 + 2 + ... = (iv) 4 + 4 + 4 + ... =
1 3 5 8  1 2 3 90

2 2 4 (−1)n
a0 = π , an = , bn = 0(∵ even)
3 n2
( )
(i)x = π(or − π) (ii)x = 0 (iii) add i and ii
(iv) U se P arseval′ s identity

Solution: Given f (x) = x2 in [−π, π].


We know that, the Fourier series of f (x) is given by (Refer above example),

X (−1)n ∞
π2
f (x) = +4 cos nx (1)
3 n2
n=1

Deduction:
(i) Put x = π. [Here x = π is a point of discontinuity which is one end of the given interval (−π, π)]
f (−π) + f (π)
∴ Sum of the Fourier series of f (x) is
2
MA8353 Transforms and Partial Differential Equations by K A Niranjan Kumar 39

X (−1)n ∞
f (−π) + f (π) π2
i.e., = +4 2
(−1)n
2 3 n
n=1
X∞
π2 + π2 π2 (−1)2n
= +4
2 3 n2
n=1

X
2π 2 π2 1
= +4
2 3 n2
n=1
X 1 ∞
2 π2
π − =4
3 n2
n=1

X
2π 2 1
=4
3 n2
n=1

X
π2 1
=
6 n2
n=1
1 1 1 π2
2
+ 2 + 2 + ··· = (2)
1 2 3 6
(ii) Put x = 0
X (−1)n ∞
π2
0= +4
3 n2
n=1
−π 2
h −1
1 1
i
= 4 2 + 2 − 2 + ···
3 1 2 3
−π 2
h1 1 1
i
= −4 2 − 2 + 2 − · · ·
3 1 2 3
π2 1 1 1
= 2 − 2 + 2 − ···
12 1 2 3
1 1 1 π 2
2
− 2 + 2 − ··· = (3)
1 2 3 12
(iii) (1) + (2) ⇒
2 2 π2 π2
+ 0 + + 0 + ··· = +
12 32 6 12
1 1 1
 3π 2
2 2 + 2 + 2 + ··· =
1 3 5 12
1 1 1 π2
2
+ 2 + 2 + ··· = ·
1 3 5 8
Example 2.6. Expand f (x) = | cos x| in a Fourier series for in the interval (−π, π).

Solution : Given f (x) =|cos x|


f (−x) = |cos(−x)| = |cos x|
f (−x) =f (x)
40 Unit II - FOURIER SERIES (F.S.)

∴ f (x) is an even function in −π < x < π.


∴ The Fourier series for the even function f (x) is given by


a0 X
f (x) = + an cos nx (1)
2
n=1

Z π Z π
2 2
where a0 = f (x)dx, an = f (x) cos nxdx
π 0 π 0

Now,

Z π
2
a0 = |cos x|dx
π 0
 
Z π Z π
2 2
=  |cos x| dx + π |cos x| dx
π 0
 2 
Z π Z π
2 2
=  cos xdx + π (− cos x)dx
π 0
 2 
Z π Z π
2 2
=  cos xdx − π cos xdx
π 0
 2 
π
2
= (sin x)02 − (sin x)ππ 
π
2
2 2
= [(1 − 0) − (0 − 1)] = [1 + 1]
π π
4
∴ a0 =
π
MA8353 Transforms and Partial Differential Equations by K A Niranjan Kumar 41

Z π
2
an = |cos x| cos nxdx
π 0
 
Z π Z π
2 2
=  |cos x| cos nxdx + π |cos x| cos nxdx
π 0
 2 
Z π Z π
2 2
=  cos x cos nxdx + π (− cos x) cos nxdx
π 0
 2 
Z π Z π
2 2
=  cos nx cos xdx − π cos nx cos xdx
π 0
 π 2 
Z Z π 
2 2 1 1
= [cos (n + 1) x + cos (n − 1) x] dx − π [cos (n + 1) x + cos (n − 1) x] dx
π 0 2 2 
h 2 i
1
∵ cos A cos B = cos (A + B) + cos (A − B)
 π 2 
 Z Z π 
1 2
= [cos (n + 1) x + cos (n − 1) x] dx − π [cos (n + 1) x + cos (n − 1) x] dx
π  0 
2
 
 π  π 
1 sin (n + 1) x sin (n − 1) x 2 sin (n + 1) x sin (n − 1) x
= + − + π
π  n+1 n−1 0
n+1 n−1
 2
π π    π π  
 sin (n + 1) sin (n − 1) sin (n + 1) sin (n − 1) 
1  2 + 2  − 0 − 0 −  2 + 2 
=
π  n+1 n−1 n+1 n−1 
 π π
1 2 sin (n + 1) 2 sin (n − 1)
=  2 + 2
π n+1 n−1
  nπ π
  nπ 
π 
2 sin + sin −
=  2 2 + 2 2 
π n+1 n−1
 nπ π π π nπ π π π
2 sin cos + cos n sin sin cos − cos n sin
=  2 2 2 2 + 2 2 2 2
π n+1 n−1
nπ  nπ 
2 coscos
=  2 − 2 
π n+1 n−1
nπ nπ  
2 cos h 1 1
i 2 cos −1 − (+1)
= 2 − = 2
π n+1 n−1 π n2 − 1

−4 cos
∴ an = 2 ,n 6= 1
π (n2 − 1)
42 Unit II - FOURIER SERIES (F.S.)

 Z π
Z 2
2
π  ∵ an =
π Z0
|cos x| cos nxdx
Now, a1 = |cos x| cos xdx 
 π
π 0
2
∴ a1 = |cos x| cos xdx
π 0
 
Z π Z π
2 2
=  cos x cos xdx + π − cos x cos xdx
π 0
 π 2 
Z Z π
2 2
=  cos2 xdx − π cos2 xdx
π 0
 π 2 
Z Z π
2 2 1 1
=  (1 + cos 2x) dx − π (1 + cos 2x) dx
π 0 2 2
 π 2 
Z Z π
1 2 1
=  (1 + cos 2x) dx − π (1 + cos 2x) dx
π 0 2
 2 
π
1
 
sin 2x 2
 sin 2x
 π
=  x+ − x+ π
π 2 0 2
2
1
hh π  i h π ii
= − 0 − 0 − (π + 0) − +0
π 2 2
h
1 π π
i
= −
π 2 2
∴ a1 =0

From (1), the Fourier series of f (x) is

X ∞
a0
f (x) = + a1 cos x + an cos nx
2
n=2

 
1 4

X −4 cos
= +0+ 2 cos nx
2 π 2
π (n − 1)
n=2


2 4 X cos 2
= − cos nx.
π π n2 − 1
n=1

2.2.3 Half range Fourier series in (0, π)

Example 2.7. Expand x(π − x) in half range sine series in the interval (0, π).

Solution: Given f (x) = x(π − x) in (0, π)


= πx − x2
MA8353 Transforms and Partial Differential Equations by K A Niranjan Kumar 43

The sine series of f (x) is given by



X
f (x) = bn sin nx (1)
n=1

Zπ Zπ
2 2 
where bn = f (x) sin nxdx = πx − x2 sin nxdx
π π
0 0
2
h   − cos nx   − sin nx   cos nx iπ
2
= πx − x − (π − 2x) + (−2)
π n n2 n3 0
2
h  cos nx sin nx
i
cos nx π
= − πx − x2 + (π − 2x) −2
π  n  n2  n3 0
2 2 (−1)n
 2

= 0+0− − 0 + 0 −
π n3 n3
 
2 2 2 (−1)n
= −
π n3 n3
4  
bn = 3
1 − (−1)n
πn
Sub. the value of bn in (1)

X 4 
f (x) = 1 − (−1)n sin nx
πn3
n=1
X∞  
4 1 − (−1)n
= sin nx
π n3
n=1
4 2
h 2
i
= 3
sin x + 0 + 3 sin 3x + 0 + · · ·
π 1 3
h
8 sin x sin 3x sin 5x
i
= + + + · · ·
π 13 33 53

2.2.4 Examples under (0, 2ℓ)

2.2.5 Examples under (−ℓ, ℓ)

2.2.6 Examples under (0, ℓ)

2.2.7 Complex form of Fourier Series

Example 2.8. Derive complex form for f (x) = eax in (0, 2π). ( )
(a + in) e2aπ − 1
cn =
2π (a2 + n2 )

Example
 2.9. Find the complex form of the series for the function f (x) = x in (−ℓ, ℓ).
n
−ℓ (−1)
cn =
inπ
44 Unit II - FOURIER SERIES (F.S.)

2.2.8 Harmonic Analysis

2.2.8.1 Examples under π form(Radian form)

Example 2.10. The table of values of the function y = f (x) is given below:
x 0 π/3 2π/3 π 4π/3 5π/3 2π
. Find the Fourier series upto 2nd harmonic to
f (x) 1.0 1.4 1.9 1.7 1.5 1.2 1.0
represent y = f (x) in terms of x in (0, 2π).

Solution : Let
a0
f (x) = + a1 cos x + a2 cos 2x + b1 sin x + b2 sin 2x (1)
2
be the F.S. upto second harmonic.
Since the first and last values of y are same in the given table, leave the first column (or) last
column of the table. Hence only the first six column values will be used.

x y cos x cos 2x sin x sin 2x y cos x y cos 2x y sin x y sin 2x


0 1.0 1 1 0 0 1 1 0 0
π
1.4 0.5 -0.5 0.866 0.866 0.7 -0.7 1.2124 1.2124
3

1.9 -0.5 -0.5 0.866 -0.866 -0.95 -0.95 1.6454 -1.6454
3
π 1.7 -1 1 0 0 -1.7 1.7 0 0

1.5 -0.5 -0.5 -0.866 0.866 -0.75 -0.75 -1.299 1.299
3

1.2 0.5 -0.5 -0.866 -0.866 0.6 -0.6 -1.0392 -1.0392
3
Sum 8.7 - - - - -1.1 -0.3 0.5196 -0.1732

h
1 X
i 1 a0 2.9
a0 =2 × y = 2 × [8.7] = 2.9 ⇒ = = 1.45
6 6 2 2
h
1 X
i 1
a1 =2 × y cos x = 2 × [−1.1] = −0.37
6 6
h
1 X
i 1
a2 =2 × y cos 2x = 2 × [−0.3] = −0.1
6 6
h
1 X
i 1
b1 =2 × y sin x = 2 × [0.5196] = 0.17
6 6
h
1 X
i 1
b2 =2 × y sin 2x = 2 × [−0.1732] = −0.06
6 6

Hence the required Fourier Series upto second harmonic for the data is

(1) ⇒ y = f (x) = 1.45 − 0.37 cos x − 0.1 cos 2x + 0.17 sin x − 0.06 sin 2x

2.2.8.2 Examples under θ◦ form(Degree form)

Example 2.11. Find an emprical form of the function


f (x ) = a0 + a1 cos x + b1 sin x with period 2π.
MA8353 Transforms and Partial Differential Equations by K A Niranjan Kumar 45

xo : 0 60 120 180 240 300 360


y = f (x) 40 31 –13 20 3.7 –21 40

Solution: Since the last value of y is a repetition of the first, only the first six values will be used.

xo : y cos x sin x y cos x y sin x


0 40 1 0 40 0
60 31 0.5 0.866 15.5 26.846
120 -13.7 -0.5 0.866 6.85 -11864
180 20 -1 0 -20 0
240 3.7 -0.5 -0.866 -1.85 -3.204
300 -21 0.5 0.5 -10.5 18.186
P P P
y = 60 y cos x = 30 y sin x = 329.964

1 X
h 1
i
a0 =2 × y = 2 × 60 = 20
6
1 X
h i 6 1
a1 =2 × y cos x = 2 × [30] = 10
6 6
1 X
h i 1
b1 =2 × y sin x = 2 × [29.964] = 9.988
6 6
∴ f (x) = 20 + 10 cos x + 9.988 sin x

2.2.8.3 Examples under T form: (θ = 2πx/T )

Example 2.12. The following table gives the vibration of periodic current over a period. Find
Fourier series upto 1st harmonic.

T (sec) 0 T /6 T /3 T /2 2T /3 5T /6 T
I (Amp) 1.98 1.3 1.05 1.3 –0.88 –0.25 1.98

Solution : Here first and last values are same. Hence omit the last value.

When x varies from 0 to T


θ varies from 0 to 2π

a0
Let f (x) = F (θ) = + a1 cos θ + b1 sin θ.
2
46 Unit II - FOURIER SERIES (F.S.)

2πX
X: θ= y cos θ sin θ y cos θ y sin θ
T
0 0 1.98 1.0 0 1.98 0
T π
1.3 0.5 0.866 0.65 1.1258
6 3
T 2π
1.05 -0.5 0.866 -0.525 0.9093
3 3
T
π 1.3 -1 0 -1.3 0
2
2T 4π
-0.88 -0.5 -0.866 0.44 0.762
3 3
5T 5π
-0.25 0.5 -0.866 -0.125 0.2165
6 P 3 P P
y = 4.5 y cos θ = 1.12 y sin θ = 3.013

1 X
h i 1 a0
a0 =2 × y = 2 × 4.5 = 1.5 ⇒ = 0.75
6 6 2
h
1 X
i 1
a1 =2 × y cos x = 2 × [1.12] = 0.373
6 6
h
1 X
i 1
b1 =2 × y sin x = 2 × [3.013] = 1.004
6 6
2πx 2πx
∴ f (x) = 0.75 + 0.373 cos + 1.004 sin
T T

2.2.8.4 Problems under ℓ form: (2ℓ = Number of data)

Example 2.13. Find the first harmonic of the Fourier series for f (x) for the data.
x 0 1 2 3 4 5
f(x) 9 18 24 28 26 20
Solution : Here the length of the interval is 2ℓ = 6, i.e., ℓ = 3.
∴ THe Fourier series upto second harmonic can be represented by
a
 πx πx
  2πx 2πx

0
y= + a1 cos + b1 sin + a2 cos + b2 sin
2 ℓ ℓ ℓ ℓ
a0
 πx πx
  2πx 2πx

y= + a1 cos + b1 sin + a2 cos + b2 sin (1)
2 3 3 3 3

πx πx πx πx 2πx
X: y cos sin y cos y sin y cos
3 3 3 3 3
0 9 1 0 9 0 9
1 18 0.5 0.866 9 15.588 -9
2 24 0.5 0.866 -12 20785 -12
3 28 -1 0 -28 0 28
4 26 -0.5 -0.866 -13 -22517 -13
5 20 0.5 -.866 10 -17.321 -10
P P πx P πx P 2πx P 2πx
y = 125 y cos = −25 y sin = −3.465 y cos = −7 y sin = 0.000
3 3 3 3
MA8353 Transforms and Partial Differential Equations by K A Niranjan Kumar 47

1 X
h i 1 a0
a0 =2 × y = 2 × 125 = 41.67 ⇒ = 20.84
6 6 2
1 X
h πx
i 1
a1 =2 × y cos = 2 × [−25] = −8.33
6 3 6
1 X
h 2πx
i 1
a2 =2 × y cos = 2 × [−7] = −2.333
6 3 6
1 X
h πx
i 1
b1 =2 × y sin = 2 × [−3.465] = −1.16
6 3 6
1 X
h 2πx
i 1
b2 =2 × y sin = 2 × [0.0004] = 0.00013
6 3 6
πx πx 2πx 2πx
∴ y = f (x) = 20.84 − 8.33 cos − 1.161.004 sin − 2.333 cos + 0.00013 sin
3 3 3 3

2.3 Assignment II[Fourier series]


1
1. Find the Fourier series of f (x) = (π − x) in the interval (0, 2π). Hence deduce that
2
1 1 1 π
1 − + − + ···∞ = .
3 5 7 4
2. Find the Fourier series expansion of f (x) = ex in the interval (0, 2ℓ).
(
1 − x, −π < x < 0
3. Obtain the Fourier series for the function f (x) = . Hence deduce that
1 + x, 0<x<π
1 1 1 π2
+ + + ...∞ = .
12 32 52 8

 1 + 2x , −ℓ ≤ x ≤ 0
4. Find the Fourier series for the function given by f (x) = ℓ . Hence
 1 − 2x , 0≤x≤ℓ


X 1 π2
deduce that = .
n=1
(2n − 1)2 8

 1 − x, 0 < x < 1
5. Express f (x) as a Fourier sine series where f (x) = 4 2 .
 x − 3, 1 < x < 1
4 2
6. Find the half range Fourier cosine series for the function f (x) = x (π − x) in 0 < x < π. Deduce
1 1 1 π4
that 4 + 4 + 4 + · · · ∞ = .
1 2 3 90
7. Find the complex form of the Fourier series of f (x) = eax , −π < x < π.

8. Obtain a Fourier series upto the second harmonics from the data
x: 0 π/3 2π/3 π 4π/3 5π/3 2π
f(x): 0.8 0.6 0.4 0.7 0.9 1.1 0.8

9. The following table gives the vibration of periodic current over a period. Find the Fourier upto
2nd harmonic.
48 Unit II - FOURIER SERIES (F.S.)

T(sec): 0 T /6 T /6 T /2 2T /3 5T /6 T
I(Amp): 1.98 1.3 1.05 1.3 -0.88 -0.25 1.98

10. Find the Fourier series as for as the second harmonic to represent the function given in the
x: 0 1 2 3 4 5
following table
f(x): 9 18 24 28 26 20
3 Applications of P.D.E.(A.P.D.E.)

3.1 Part-A

1. Classify the following partial differential Soln :


equations. (i) Here A = 1, B = 0, C = −1.

(i) y 2 Uxx −2xyUxy +x2 Uyy +2Ux − 3U = 0 ∴ B 2 − 4AC = −4(−1) = 4 > 0

(ii) y 2 Uxx + Uyy + Ux2 + Uy2 + 7 = 0 The given partial differential equation
is Hyperbolic.
Soln :
(ii) Here A = 0, B = 1, C = 0.
(i)Comparing the given equation with the
∴ B 2 − 4AC = 1 > 0
general second order linear PDE,
The given partial differential equation is
Hyperbolic.
AUxx +BUxy +CUyy +DUx +EUy +F u = 0.
Here A = y 2 , B = −2xy, C = x2 3. Classify the partial differential equation
∂2u ∂ 2u
  +x 2 =0
∴ B 2 − 4AC = (−2xy)2 − 4 x2 y2 ∂x2 ∂y
Soln : Here A = 1, B = 0, C = x.
= 4x2 y 2 − 4y 2 x2
∴ B 2 − 4AC = −4x
=0
The equation is Elliptic if x > 0,
The given partial differential equation
Hyperbolic if x < 0
is Parabolic.
(ii) Here A = y 2 , B = 0, C = 1 and Parabolic if x = 0.

∴ B 2 − 4AC = −4y 2 < 0 4. What is the constant a2 in the wave


∂ 2y ∂ 2y
The given partial differential equation equation 2 = a2 2 ?
∂t ∂x
is Elliptic. Soln : Here
T
2. Classify the following partial differential a2 =
M
equations. The tension in the string
=
Mass per unit length of the string
∂ 2u ∂ 2u
(i) = 5. Write the possible solutions of the one
∂x2 ∂y 2
   ∂u ∂ 2u
∂ 2u ∂u ∂u dimensional heat equation = a2 2 .
(ii) = + xy ∂t ∂x
∂x∂y ∂x ∂y Soln :
50 Part-A : Unit III - APPLICATIONS OF P.D.E.(A.P.D.E.)

2 2
u(x, t) = (A cos px + B sin px) e−α p t 9. Write the boundary conditions and initial
 2 2 conditions for solving the vibration of string
u(x, t) = Aepx + Be −px
eα p t

equation, if the string is subjected to initial


u(x, t) = (Ax + B)
displacement f (x) and initial velocity g(x).
6. Write the possible solutions of the one
Soln : The boundary and initial conditions
∂ 2y ∂ 2y
dimensional wave 2 = a2 2 . are
∂t ∂x
Soln : (i) y(x = 0, t) = 0

y(x, t) = (A cos px+B sin px) (ii) y(x = ℓ, t) = 0


∂y
(C cos pat+D sin pat) (iii) (x, t = 0) = g(x)
  ∂t
y(x, t) = Aepx + Be−px Cepat + De−pat (iv) y(x, t = 0) = f (x)
y(x, t) = (A3 x + B3 ) (C3 t + D3 )
10. In one dimensional heat equation ut =

7. Write any two solutions of the Laplace α2 uxx , what does α2 stands for?
∂ 2u ∂ 2u Soln :
equation + =0
∂x2 ∂y 2 k
Soln : α2 =
ρc
u(x, y) = (A cos px + B sin px) Thermal Conductivity
py −py
 =
Ce + De (Density Specific Heat)(Specific Heat)

u(x, y) = Aepx + Be−px = diffusivity of the material

(C cos py + D sin py)


11. State any two laws which are assumed to
u(x, y) = (Ax + B) (Cy + D) derive one dimensional heat equation.
8. A tightly stretched string of length 2ℓ is Soln :
fastened at both ends. The mid point of (1) Heat flows from higher temperature to
the string is displaced to a distance b and lower temperature.
released from rest in this position write the
(2) The amount of heat required to
initial conditions.
produce a given temperature change
Soln : The initial conditions are
  in a body is proportional to the mass
∂y
(i) = 0, 0 ≤ x ≤ ℓ of the body and to the temperature
∂t t=0
 change.
 bx
, 0≤x≤ℓ
(ii) y(x, 0) = ℓ (3) The rate at which heat flows through
 b(2ℓ − x) , ℓ ≤ x ≤ 2ℓ
ℓ an area is proportional to the area and
  to the temperature gradient normal to
y x bx
Since equation of OA is = ⇒ y =
b ℓ ℓ the area.

y−b ℓ−x
Since equation of AB is = 12. In steady state conditions derive the
b ℓ
 solution of one dimensional heat flow
b
⇒ y = (2ℓ − x)
ℓ equation.
MA8353 Transforms and Partial Differential Equations by K A Niranjan Kumar 51

Soln : In the steady state equation reduces to


The temperature function u will be a ∂2u
= 0.
∂x2
function of x alone in the steady state Integrating we get, u = ax + b (1)
condition. When x = 0, u = 20, ∴ b = 20
∂u
i.e., = 0 , under steady state condition. When x = 60, u = 80 ⇒ 80 = 60a + b
∂t
∂u ∂ 2u 80 = 60a + 20 ⇒ 60a = 60 ⇒ a = 1
∴ The heat flow equation = α2 2
∂t ∂x
∂ 2u ∴ (1) ⇒ u = x + 20
reduces to =0
∂x2
Integrating we get, u(x) = ax + b. 15. A rod 30cm long has its ends a and b kept
at 200c and 800c respectively until steady
13. State one dimensional heat equation with
state conditions prevail. Find the steady
the initial and boundary conditions.
state temperature in the rod.
Soln : The one dimensional heat flow
∂u 2 Soln :
2∂ u
equation is =α
∂t ∂x2 Method : 1
The boundary conditions are
u(x=30cm) − u(x=0cm)
(i) u(x = 0, t) = k1◦ C for all t > 0 u= x + u(x=0cm)
length of the rod
(ii) u(x = ℓ, t) = k2◦ C for all t > 0 800 − 200
= x + 20
30
(iii) u(x, t = 0) = f (x) in (0, ℓ) ∴ u = 20x + 200
−→ initial condition
(OR)
14. An insulated rod of length 60cm has its ends Method : 2
at A and B maintained at 20◦ C and 80◦ C The equation of heat flow in one dimension
respectively. Find the steady state solution ∂u ∂2u
is given by = α2 2
of the heat equation in one dimension along ∂t ∂x
In the steady state equation reduces to
the rod. ∂2u
= 0.
Soln : ∂x2
Integrating we get, u = ax + b
Method : 1
When x = 0, u = 200, ∴ b = 200
u(x=60cm) − u(x=0cm)
u= x + u(x=0cm) When x = 30, u = 800 ⇒ 800 = 30a + b
length of the rod
80 − 20
= x + 20 800 = 30a + 200
60
∴ u = x + 20 ⇒ 30a = 600
⇒ a = 20
(OR)
Method : 2 ∴ u = 20x + 200
The equation of heat flow in one dimension
16. What is the basic difference between the
is given by
2 solutions of one dimensional wave equation
∂u 2∂ u
=α and one dimensional heat equation.
∂t ∂x2
52 Part-A : Unit III - APPLICATIONS OF P.D.E.(A.P.D.E.)

Soln : tension t.
# one one
(iv) The effect of friction is negligible.
dimensional dimensional
wave equation heat equation 18. Define temperature gradient.
∂ 2y 2 ∂
y ∂u ∂ 2u
= a = a2 2 Soln :
∂t2 ∂x2 ∂t ∂x
1. It is a It is a parabolic The rate of change of temperature with
hyperbolic p.d.e. respect to distance is called temperature
∂u
p.d.e. gradient and is denoted as .
∂x
2. The suitable The suitable
19. Distinguish between steady and unsteady
solution of one solution of one
states in heat conduction problems.
dim. dim.
Soln :
wave eqn. is heat eqn. is
In unsteady state the temperature at any
periodic w.r.t. non-periodic
point of the body depends on the position
time ‘t’. w.r.t. time ‘t’.
of the point and also the time t.
17. State any two assumptions made in In steady state, the temperature at any
the derivation of one-dimensional wave point depends only on the position of the
equation. point and is independent of time ′ t′ .
Soln :
20. Write down the two dimensional heat
(i) The motion takes entirely in one plane
equation both in transient and steady states.
and in this plane each particle moves
Soln :  
in a direction perpendicular to the ∂u ∂ 2u ∂ 2u
Transient state : = α2 +
equilibrium position of the string. ∂t ∂x2 ∂y 2
(ii) The tension in the string is constant. ∂ 2u ∂ 2u
Steady state : + =0
 ∂x2 ∂y 2 
(iii) The gravitational force may be ∂u
∵ = 0 in Transient state
neglected in comparison with the ∂t

3.2 Part-B

3.2.1 One dimensional wave equation

3.2.1.1 Zero Velocity Problems

Example 3.1. A tightly stretched string of length ℓ is fastened at both ends. Motion is started by

displacing the string into the form y = k ℓx − x2 , from which it is released at time t = 0. Find
the displacement at any point on the string at a distance x from one end at time t. [UQ]

Solution : The displacement function y(x, t) of the string is the solution of wave equation
∂ 2y 2
2∂ y
=a . (I)
∂t2 ∂x2
MA8353 Transforms and Partial Differential Equations by K A Niranjan Kumar 53

T Tension
where a2 = =
M Mass
Solving the equation (I) by method of separation of variable, we get the following three possible
solutions.
 
(1)y(x, t) = A1 epx + A2 e−px A3 eapt + A4 e−apt
(2)y(x, t) = (A5 cos px + A6 sin px) (A7 cos apt + A8 sin apt)
(3)y(x, t) = (A9 x + A10 ) (A11 t + A12 )

out of these solutions, we have to select that solution which suits the physical nature of the
problem and the boundary conditions.
Boundary conditions
(i)y(x = 0, t) = 0, ∀t > 0
(ii)y(x = ℓ, t) = 0, ∀t > 0
Initial conditions
∂y
(iii) (x, t = 0) = 0, ∀x ∈ (0, ℓ)
∂t

(iv)y(x, t = 0) = f (x) = k ℓx − x2 , ∀x ∈ (0, ℓ)
The suitable solution which satisfies above boundary conditions is
y(x, t) = (c1 cos px + c2 sin px) (c3 cos apt + c4 sin apt) (1)
Applying condition (i) in equation (1), we get
We have (i) ⇒ y(x = 0, t) =0, ∀t > 0
(1) ⇒ (c1 + 0) (c3 cos pat + c4 sin pat) =0
Here (c3 cos pat + c4 sin pat) 6=0 (∵ it is defined ∀t > 0)

∴ c1 = 0

∴ Now (1) ⇒ y(x, t) = c2 sin px (c3 cos pat + c4 sin pat) (2)
Applying condition (ii) in equation (2), we get
We have (ii) ⇒ y(x = ℓ, t) =0, ∀t > 0
(2) ⇒ (c2 sin pℓ) (c3 cos pat + c4 sin pat) =0
Here (c3 cos pat + c4 sin pat) 6=0 (∵ it is defined ∀t > 0)
∴ c2 6=0 (∵ it gives trivial solution)
∴ sin pℓ =0
= sin nπ


∴p=

 nπx   nπ nπ

∴ Now (2) ⇒ y(x, t) = c2 sin c3 cosat + c4 sin at (3)
ℓ ℓ ℓ
Before applying condition (iii) in (3), differentiate (3) w.r.t. ‘t’
∂y nπx
h  nπ  nπ  nπ  nπ
i
(x, t) = c2 sin −c3 a sin at + c4 a cos at (4)
∂t ℓ ℓ ℓ ℓ ℓ
54 Part-A : Unit III - APPLICATIONS OF P.D.E.(A.P.D.E.)

Applying condition (iii) in equation (4), we get


∂y
We have (iii) ⇒ (x, t = 0) = 0, ∀x ∈ (0, ℓ)
h∂t  i
nπx nπ
(4) ⇒ c2 sin c4 a =0
ℓ ℓ

Here, c2 6= 0 (∵ it gives trivial solution)


nπx
sin 6= 0 (∵ it is defined ∀x ∈ (0, ℓ))


a 6= 0 (∵ all are constants)

∴ c4 = 0

 nπ   nπ 
∴ Now (3) ⇒ y(x, t) = c2 sin x c3 cos at
ℓ nπ  ℓ 
 nπ
= c2 c3 sin x cos at
 nπℓ   nπℓ 
= cn sin x cos at
ℓ ℓ
where cn = c2 c3
∴ By superposition principle( i.e., adding all such solutions ), the most general solution is


X  nπx   nπa 
y(x, t) = cn sin cos t (5)
ℓ ℓ
n=1

Applying condition (iv) in equation (5), we get

We have (iv) ⇒ y(x, t = 0) = f (x), ∀x ∈ (0, ℓ)



X  nπx  
(5) ⇒ cn sin = f (x) = k ℓx − x2 (6)

n=1

To find cn ,
Expand f (x) in a half range Fourier sine series in the interval (0, ℓ).


X nπx
f (x) = bn sin (7)

n=1

Zℓ
2 nπx
where bn = f (x) sin dx
ℓ ℓ
0
From (6) and (7), we have cn = bn
MA8353 Transforms and Partial Differential Equations by K A Niranjan Kumar 55

Zℓ
2 nπx
∴ cn = f (x) sin dx
ℓ ℓ
0
Zℓ
2 nπx
= k(ℓx − x2 ) sin dx
ℓ ℓ
0
   
2k 2ℓ3 2ℓ3
= 0 + 0 − 3 3 (−1)n − 0+0− 3 3
ℓ n π n π
(sin nπ = 0 , cos nπ = (−1)n )
4kℓ2
= 3π3
[1 − (−1)n ]
n

 0, if n is even
∴ cn = 8kℓ2
 , if n is odd
n3 π 3
∴ The most general solution (5) reduces to

X 8kℓ2
 nπ   nπa 
y(x, t) = sin x cos t
n3 π 3 ℓ ℓ
n=odd

This is the required displacement function y(x, t).

Example 3.2. A string is stretched and fastened to two points at a distance ‘ℓ’ apart. Motion is
πx
started by displacing the string in the form y = k sin from which it is released at time t = 0.

Show that the displacement of any point on the string at a distance x from one end at time t is given
πx πat
by k sin cos .
ℓ ℓ
Solution : The displacement function y(x, t) of the string is the solution of wave equation
∂ 2y 2
2∂ y
= a . (I)
∂t2 ∂x2
T Tension
where a2 = =
M Mass
Solving the equation (I) by method of separation of variable, we get the following three possible
solutions.
 
(1)y(x, t) = A1 epx + A2 e−px A3 eapt + A4 e−apt
(2)y(x, t) = (A5 cos px + A6 sin px) (A7 cos apt + A8 sin apt)
(3)y(x, t) = (A9 x + A10 ) (A11 t + A12 )
out of these solutions, we have to select that solution which suits the physical nature of the
problem and the boundary conditions.
Boundary conditions
(i)y(x = 0, t) = 0, ∀t > 0
(ii)y(x = ℓ, t) = 0, ∀t > 0
Initial conditions
56 Part-A : Unit III - APPLICATIONS OF P.D.E.(A.P.D.E.)

∂y
(iii) (x, t = 0) = 0, ∀x ∈ (0, ℓ)
∂t
πx
(iv)y(x, t = 0) = f (x) = k sin , ∀x ∈ (0, ℓ)

The suitable solution which satisfies above boundary conditions is
y(x, t) = (c1 cos px + c2 sin px) (c3 cos apt + c4 sin apt) (1)
Applying condition (i) in equation (1), we get
We have (i) ⇒ y(x = 0, t) =0, ∀t > 0
(1) ⇒ (c1 + 0) (c3 cos pat + c4 sin pat) =0
Here (c3 cos pat + c4 sin pat) 6=0 (∵ it is defined ∀t > 0)
∴ c1 =0
∴ Now (1) ⇒ y(x, t) = c2 sin px (c3 cos pat + c4 sin pat) (2)
Applying condition (ii) in equation (2), we get
We have (ii) ⇒ y(x = ℓ, t) =0, ∀t > 0
(2) ⇒ (c2 sin pℓ) (c3 cos pat + c4 sin pat) =0
Here (c3 cos pat + c4 sin pat) 6=0 (∵ it is defined ∀t > 0)
∴ c2 6=0 (∵ it gives trivial solution)
∴ sin pℓ =0
= sin nπ

∴p=
ℓ  
nπx nπ nπ
∴ Now (2) ⇒ y(x, t) = c2 sin c3 cos at + c4 sin at (3)
ℓ ℓ ℓ
Before applying condition (iii) in (3), differentiate (3) w.r.t. ‘t’
∂y nπx
h  nπ  nπ  nπ  nπ
i
(x, t) = c2 sin −c3 a sin at + c4 a cos at (4)
∂t ℓ ℓ ℓ ℓ ℓ
Applying condition (iii) in equation (4), we get
∂y
We have (iii) ⇒ (x, t = 0) = 0, ∀x ∈ (0, ℓ)
∂t
h  nπ i
nπx
(4) ⇒ c2 sin c4 a =0
ℓ ℓ

Here, c2 6= 0 (∵ it gives trivial solution)


nπx
sin 6= 0 (∵ it is defined ∀x ∈ (0, ℓ))


a 6= 0 (∵ all are constants)

∴ c4 = 0
 nπ   nπ 
∴ Now (3) ⇒ y(x, t) = c2 sin x c3 cos at
ℓ nπ  ℓ 
 nπ
= c2 c3 sin x cos at
 nπℓ   nπℓ 
= cn sin x cos at
ℓ ℓ
where cn = c2 c3
MA8353 Transforms and Partial Differential Equations by K A Niranjan Kumar 57

∴ By superposition principle( i.e., adding all such solutions ), the most general solution is
X∞  nπx   nπa 
y(x, t) = cn sin cos t (5)
ℓ ℓ
n=1

Applying condition (iv) in equation (5), we get

We have (iv) ⇒ y(x, t = 0) = f (x), ∀x ∈ (0, ℓ)



X  nπx  πx
(5) ⇒ cn sin = f (x) = k sin (6)
ℓ ℓ
n=1

To find cn ,
Expand f (x) in a half range Fourier sine series in the interval (0, ℓ).

X nπx
f (x) = bn sin (7)

n=1

Zℓ
2 nπx
where bn = f (x) sin dx
ℓ ℓ
0
From (6) and (7), we have cn = bn

πx X

πx
 
k sin = cn sin
ℓ ℓ
n=1
nπx πx
 
2πx 3πx
   
k sin = c1 sin + c2 sin + c3 sin + ...
ℓ ℓ ℓ ℓ
Comparing the like coefficients, we get

∴ c1 = k, c2 = c3 = c4 = · · · = 0

∴ The most general solution (5) reduces to


πx πat
∴ y(x, t) = k sin cos
ℓ ℓ
This is the required displacement function y(x, t).

Example 3.3. [Section problem]


A uniform string is fixed at the ends x = 0 and x = ℓ are fixed. One end is taken at the origin and
at a distance ‘b’ from this end the string is displaced a distance ‘h’ transversely and is released from
rest in that position. Find the displacement of any of the string at any subsequent time.

Solution : This is of Zero Velocity example format as previous.


∂ 2y ∂ 2y
The displacement function y(x, t) of the string is the solution of wave equation 2 = a2 2 . (I)
∂t ∂x
2 T Tension
where a = =
M Mass
Solving the equation (I) by method of separation of variable, we get the following three possible
solutions.
58 Part-A : Unit III - APPLICATIONS OF P.D.E.(A.P.D.E.)

 
(1)y(x, t) = A1 epx + A2 e−px A3 eapt + A4 e−apt
(2)y(x, t) = (A5 cos px + A6 sin px) (A7 cos apt + A8 sin apt)
(3)y(x, t) = (A9 x + A10 ) (A11 t + A12 )
out of these solutions, we have to select that solution which suits the physical nature of the
problem and the boundary conditions.
Boundary conditions
(i)y(x = 0, t) = 0, ∀t > 0
(ii)y(x = ℓ, t) = 0, ∀t > 0
Initial conditions
∂y
(iii) (x, t = 0) = 0, ∀x ∈ (0, ℓ)
∂t
(iv)y(x, t = 0) = f (x), ∀x ∈ (0, ℓ), where f (x) is to be find later.
The suitable solution which satisfies above boundary conditions is
y(x, t) = (c1 cos px + c2 sin px) (c3 cos apt + c4 sin apt) (1)
As in previous example, the solution of (I) satisfying the boundary conditions (i),(ii)
and (iii) is
nπx nπat
y(x, t) = cn sin cos (where cn = c2 c3 )
ℓ ℓ
∴ By superposition principle( i.e., adding all such solutions ), the most general solution is
X∞  nπx   nπa 
y(x, t) = cn sin cos t (5)
ℓ ℓ
n=1

Applying condition (iv) in equation (5), we get

We have (iv) ⇒ y(x, t = 0) = f (x), ∀x ∈ (0, ℓ)



X  nπx 
(5) ⇒ cn sin = f (x) (6)

n=1

To find cn ,
Expand f (x) in a half range Fourier sine series in the interval (0, ℓ).

X nπx
f (x) = bn sin (7)

n=1

Zℓ
2 nπx
where bn = f (x) sin dx
ℓ ℓ
0
From (6) and (7), we have cn = bn
Zℓ
2 nπx
cn = f (x) sin dx (8)
ℓ ℓ
0
MA8353 Transforms and Partial Differential Equations by K A Niranjan Kumar 59

where f (x) is not yet find, we will find f (x) as follows


Taking OA = ℓ, length of the string with B as its points of bisection.

C (b, h)

A (ℓ, 0)
X
O B (b, 0)
(0, 0)

The initial position(


of the string is given by the equations to the lines OB and BA.
Equation of OC, 0 < x < b
To find y = f ( x )=
Equation of CA, b < x < ℓ
Equation of the line OC joining (0, 0) & (b, h) is given by
y − y1 x − x1
=
y2 − y1 x2 − x1
y−0 x−0
=
h−0 b−0
h
∴ y = x, 0 ≤ x ≤ b
b
h
∴ y = x, 0 ≤ x ≤ b
b
Equation of the line CA joining (b, h) & (ℓ, 0) is given by
y − y1 x − x1
=
y2 − y1 x2 − x1
y−h x−b y−h x−b
= ⇒ =
0−h ℓ−b −h ℓ−b
(y − h)(ℓ − b) = h(x − b) ⇒ yℓ − yb − hℓ + hb = −hx + hb
y(ℓ − b) = h(ℓ − x) ⇒ y(b − ℓ) = h(x − ℓ)
h
∴y= [x − ℓ], b ≤ x ≤ ℓ
b−ℓ
∴ The required equation of the string is

 h
x ,0 ≤ x ≤ b
y = f (x) = b
 h
[x − ℓ] , b ≤ x ≤ ℓ
b−ℓ
∴ (8) ⇒
60 Part-A : Unit III - APPLICATIONS OF P.D.E.(A.P.D.E.)

 
Zb Zℓ
2 hx nπx h (x − ℓ) nπx 
cn =  sin dx + sin dx
ℓ b ℓ b−ℓ ℓ
0 b
( "  !# b
nπx 
2h 1 − cos ℓ − sin nπx

= x nπ − (1) 
ℓ b nπ 2
ℓ ℓ 0
"  !#ℓ 
nπx  
1 − cos ℓ − sin nπx

+ (x − ℓ) nπ − (1) 2
b−ℓ ℓ
nπ 
ℓ b
( "  2  #b
2h 1 ℓ

nπx
 ℓ nπx

= −x cos + sin
ℓ b nπ ℓ nπ ℓ
0
"  2  #ℓ 
1 ℓ nπx
  ℓ nπx
 
+ − (x − ℓ) cos + sin
b−ℓ nπ ℓ nπ ℓ 
b
( "    2  #
2h 1 bℓ nπb ℓ nπb
= − cos + sin − [0 + 0]
ℓ b nπ ℓ nπ ℓ
" "    2  ##)
1 ℓ nπb ℓ nπb
+ [0 + 0]− −(b−ℓ) cos + sin
b−ℓ nπ ℓ nπ ℓ
(    2    
2h −ℓ nπb ℓ 1 nπb ℓ nπb
= cos + sin + cos
ℓ nπ ℓ nπ b ℓ nπ ℓ
 2  )
1 ℓ nπb
− sin
b − ℓ nπ ℓ
" 2    2  #
2h ℓ 1 nπb 1 ℓ nπb
= sin − sin
ℓ nπ b ℓ b−ℓ nπ ℓ
" 2   #
2h ℓ nπb 1 1

= sin −
ℓ nπ ℓ b b−ℓ
 
−2hℓ2 nπb
= 2 2
sin
b(b − ℓ)n π ℓ
∴ The most general solution (5) reduces to
∞      nπa 
−2hℓ2 X 1 nπb nπx
i.e., (4) ⇒ y(x, t) = sin sin cos t
b(b − ℓ)π 2 n2 ℓ ℓ ℓ
n=1
This is the required displacement solution y(x, t).

3.2.1.2 [Non - Zero Velocity Problem]

Example 3.4. A tightly stretched string with fixed end points x = 0 and x = ℓ is initially at rest
in its equilibrium position. If it is set vibrating by giving to each of its points a velocity λx(ℓ − x),
find y(x, t).
MA8353 Transforms and Partial Differential Equations by K A Niranjan Kumar 61

Solution : The displacement function y(x, t) of the string is the solution of wave equation
∂ 2y 2
2∂ y
= a . (I)
∂t2 ∂x2
T Tension
where a2 = =
M Mass
Solving the equation (I) by method of separation of variable, we get the following three possible
solutions.

 
(1)y(x, t) = A1 epx + A2 e−px A3 eapt + A4 e−apt
(2)y(x, t) = (A5 cos px + A6 sin px) (A7 cos apt + A8 sin apt)
(3)y(x, t) = (A9 x + A10 ) (A11 t + A12 )

out of these solutions, we have to select that solution which suits the physical nature of the
problem and the boundary conditions.
Boundary conditions
(i)y(x = 0, t) = 0, ∀t > 0
(ii)y(x = ℓ, t) = 0, ∀t > 0
Initial conditions
(iii)y(x, t = 0) = 0, ∀x ∈ (0, ℓ)
∂y 
(iv) (x, t = 0) = f (x) = λ ℓx − x2 , ∀x ∈ (0, ℓ)
∂t
The suitable solution which satisfies above boundary conditions is
y(x, t) = (c1 cos px + c2 sin px) (c3 cos apt + c4 sin apt) (1)
Applying condition (i) in equation (1), we get
We have (i) ⇒ y(x = 0, t) =0, ∀t > 0
(1) ⇒ (c1 + 0) (c3 cos pat + c4 sin pat) =0
Here (c3 cos pat + c4 sin pat) 6=0 (∵ it is defined ∀t > 0)
∴ c1 =0
∴ Now (1) ⇒ y(x, t) = c2 sin px (c3 cos pat + c4 sin pat) (2)
Applying condition (ii) in equation (2), we get
We have (ii) ⇒ y(x = ℓ, t) =0, ∀t > 0
(2) ⇒ (c2 sin pℓ) (c3 cos pat + c4 sin pat) =0
Here (c3 cos pat + c4 sin pat) 6=0 (∵ it is defined ∀t > 0)
∴ c2 6=0 (∵ it gives trivial solution)
∴ sin pℓ =0
= sin nπ

∴p=
ℓ  
nπx nπ nπ
∴ Now (2) ⇒ y(x, t) = c2 sin c3 cos at + c4 sin at (3)
ℓ ℓ ℓ
Applying condition (iii) in (3), we get
62 Part-A : Unit III - APPLICATIONS OF P.D.E.(A.P.D.E.)

We have (iii) ⇒ y(x, t = 0) = 0


h nπx
i
∴ (3) ⇒ c2 sin (c3 ) = 0

c2 =6 0 (∵ it gives trivial solution)


nπx
sin 6= 0 (∵ it is defined for all x)

∴ c3 = 0
 nπ   nπ 
∴ Now (3) ⇒ y(x, t) = c2 sin x c4 sin at
ℓ nπ  ℓ 
 nπ
= c2 c4 sin x sin at
 nπℓ   nπℓ 
= cn sin x sin at
ℓ ℓ
where cn = c2 c4
∴ By superposition principle(i.e., adding all such above solutions ), the most general solution is

X  nπx   nπa 
y(x, t) = cn sin sin t (4)
ℓ ℓ
n=1

Before applying condition (iv), differentiate (4) w.r.t. ‘t’

∂y X ∞
nπx
  nπa  nπa 
(x, t) = cn sin cos t (5)
∂t ℓ ℓ ℓ
n=1

Applying condition (iv) in equation (5), we get

∂y
We have (iv) ⇒ (x, t = 0) = f (x), ∀x ∈ (0, ℓ)
∂t

X nπa
 nπx  
(5) ⇒ cn sin = f (x) = λ ℓx − x2 (6)
ℓ ℓ
n=1

To find cn ,
Expand f (x) in a half range Fourier sine series in the interval (0, ℓ)

X nπx
f (x) = bn sin (7)

n=1
Zℓ
2 nπx
where bn = f (x) sin dx
ℓ ℓ
0
nπa
From (6) and (7), we have bn = cn

Zℓ
nπa 2 nπx
cn = f (x) sin dx
ℓ ℓ ℓ
0
MA8353 Transforms and Partial Differential Equations by K A Niranjan Kumar 63

Zℓ
2 nπx
= λ(lx − x2 ) sin dx
ℓ ℓ
0
   
2λ 2ℓ3 2ℓ3
= 0 + 0 − 3 3 (−1)n − 0+0− 3 3
ℓ n π n π
(∵ sin nπ = 0 , cos nπ = (−1)n )
4λℓ2
= 3π3
[1 − (−1)n ]
n

 0 , if n is even
∴ cn = 8λℓ3
 , if n is odd
n4 π 4
∴ The most general solution is

8λℓ3 X 1
 nπx   nπa 
i.e., (4) ⇒ y(x, t) = sin sin t
π4 n4 ℓ ℓ
n=odd
This is the required displacement y(x, t).

Example 3.5. A tightly stretched string with  fixed


 end points x = 0 and x = ℓ is initially in a
3 πx
position with velocity ν given by ν = y0 sin . Find the displacement y(x, t) in terms of Fourier

coefficients of f (x).

Solution : The displacement function y(x, t) of the string is the solution of wave equation
∂ 2y 2
2∂ y
= a . (I)
∂t2 ∂x2
T Tension
where a2 = =
M Mass
Solving the equation (I) by method of separation of variable, we get the following three possible
solutions.
 
(1)y(x, t) = A1 epx + A2 e−px A3 eapt + A4 e−apt
(2)y(x, t) = (A5 cos px + A6 sin px) (A7 cos apt + A8 sin apt)
(3)y(x, t) = (A9 x + A10 ) (A11 t + A12 )

out of these solutions, we have to select that solution which suits the physical nature of the
problem and the boundary conditions.
Boundary conditions
(i)y(x = 0, t) = 0, ∀t > 0
(ii)y(x = ℓ, t) = 0, ∀t > 0
Initial conditions
(iii)y(x, t = 0) = 0, ∀x ∈ (0, ℓ)
∂y πx
 
(iv) (x, t = 0) = f (x) = ν = y0 sin3
∂t ℓ
The suitable solution which satisfies above boundary conditions is
y(x, t) = (c1 cos px + c2 sin px) (c3 cos apt + c4 sin apt) (1)
64 Part-A : Unit III - APPLICATIONS OF P.D.E.(A.P.D.E.)

Applying condition (i),(ii) and (iii) as in previous problem, we get


∴ By superposition principle(i.e., adding all such above solutions ), the most general solution is

X  nπx   nπa 
y(x, t) = cn sin sin t (4)
ℓ ℓ
n=1

Before applying condition (iv), differentiate (4) w.r.t. ‘t’

∂y X ∞
nπx
  nπa  nπa 
(x, t) = cn sin cos t (5)
∂t ℓ ℓ ℓ
n=1

Applying condition (iv) in equation (5), we get


∂y
We have (iv) ⇒ (x, t = 0) = f (x)
∂t

X nπa
 nπx   πx 
(5) ⇒ cn sin = f (x) = ν = y0 sin3 (6)
ℓ ℓ ℓ
n=1

To find cn ,
Expand f (x) in a half range Fourier sine series in the interval (0, ℓ)
X∞
nπx
f (x) = bn sin (7)

n=1
Zℓ
2 nπx
where bn = f (x) sin dx
ℓ ℓ
0
nπa
From (6) and (7), we have bn = cn

To find cn ,
 πx  X
∞  nπx 
3
y0 sin = bn sin
ℓ ℓ
n=1
"  #
3 sin πx
−sin 3 πx hπ i h 2π i h 3π i
ℓ ℓ
y0 =b1 sin x +b2 sin x +b3 sin x +. . .
4 ℓ ℓ ℓ
 
3 sin A − sin 3A 3
∵ sin A =
4
3y0
h πx
i y0
h 3πx
i h π
i h 2π
i h 3π i
sin − sin =b1 sin x +b2 sin x +b3 sin x +. . .
4 ℓ 4 ℓ ℓ ℓ ℓ
Comparing the like coefficients, we get
3y0 −y0
∴ b1 = and b3 = , bn =0 for n 6= 2, 4, 5, 6
4 4
ℓ 3y ℓ ℓ −y0 ℓ
∴ c1 =b1 = 0 and c3 =b3 = , cn =0 for n 6= 2, 4, 5, 6
πa 4 πa 3πa 4 3πa
 
nπa ℓ
∵ bn = c n ⇒ c n = bn
ℓ nπa
∴ The most general solution is
MA8353 Transforms and Partial Differential Equations by K A Niranjan Kumar 65

hπ i h πa i h 3π i h 3πa i
i.e.,(4) ⇒ y(x, t) = c1 sin x sin t + c3 sin
x sin t
ℓ ℓ ℓ ℓ
3y0 ℓ
h π i h πa i −y ℓ h 3π i h 3πa i
0
= sin x sin t + sin x sin t
4 πa ℓ ℓ 4 3πa ℓ ℓ
3y0 ℓ
h π i h πa i y0 ℓ
h 3π i h 3πa i
= sin x sin t − sin x sin t
4πa ℓ ℓ 12πa ℓ ℓ
This is the required displacement y(x, t).

3.2.2 One Dimensional heat flow equation

3.2.2.1 Steady state conditions and zero boundary conditions

Example 3.6. A rod of length ℓ has its ends A and B kept at 0◦ C and 120◦ C respectively until
steady state conditions prevail. If the temperature at B is reduced to 0◦ C and kept so while that of
A is maintained, find the resulting temperature distribution u(x, t) taking origin at A.

Solution : The temperature function u(x, t) satisfies the one dimensional heat equation is
∂u ∂ 2u
= a2 2 . (I)
∂t ∂x
k Thermal conductivity
where a2 = =
ρc (Density)(Specific heat)
Solving the equation (I) by method of separation of variable, we get the following three possible
solutions.
 2 2
(1)u(x, t) = A1 epx + A2 e−px A3 ea p t

2 2
(2)u(x, t) = (A4 cos px + A5 sin px) A6 e−a p t

(3)u(x, t) = (A7 + A8 x) A9

out of these solutions, we have to select that solution which suits the physical nature of the
problem and the boundary conditions.
First let us find the temperature distribution at any distance x, before the end A and B are
reduced to zero. Prior to the temperature change at the ends A and B, when t = 0, the heat flow
was independent of time (steady state conditions). When the temperature u depends only on x and
not on t,
When steady state conditions prevail
(I) reduces to
∂u ∂ 2u
=0⇒ =0 (II)
∂t ∂x2
The general solution of (II) is u = ax + b (III)
where a and b are arbitrary constants.
Given that u = 0 when x = 0
∴ From (III), we get 0 = a(0) + b ⇒ b = 0
Also given that u = 120 when x = ℓ
66 Part-A : Unit III - APPLICATIONS OF P.D.E.(A.P.D.E.)

From (III),aℓ + b = 120


aℓ + 0 = 120
120
a= x

u = 0◦ C u = 120◦ C
A B
x=0 x=ℓ

120
∴ (III) becomes u = x, which is initial temprature distribution.

After steady state conditions
When the temperature at A and B are reduced to 0◦ C, the state is not more steady state. For
this transient state, the boundary conditions are

(i)u(x = 0, t) = 0, ∀t > 0
(ii)u(x = ℓ, t) = 0, ∀t > 0
120
(iii)u(x, t = 0) = f (x) = x, 0 < x < ℓ

u = 0◦ C u = 0◦ C
A B
x=0 x=ℓ

The suitable solution which satisfies above boundary conditions is


2 2
u(x, t) = (A cos px + B sin px) e−a p t
(1)
Applying condition (i) in equation (1) we get
We have

(i) ⇒ u(x = 0, t) = 0
2 2
(1) ⇒ Ae−a p t
=0
2 2
Here e−a p t
6= 0 (∵ it is defined for all t)
∴A=0
2 2
∴ Now (1) ⇒ u(x, t) = B sin pxe−a p t
(2)
Applying condition (ii) in equation (2) we get
We have

(ii) ⇒ u(x = ℓ, t) = 0
2 2
(2) ⇒ B sin pxe−a p t
=0
MA8353 Transforms and Partial Differential Equations by K A Niranjan Kumar 67

2 2
Here e−a p t
6= 0 (∵ it is defined for all t)
B 6= 0 (∵ it gives trivial solution )
∴ sin pℓ = 0
= sin nπ

∴p=

nπx 2 n2 π 2
−a t
∴ Now (2) ⇒ u(x, t) = B sin e ℓ2

∴ By superposition principle(i.e., adding all such above solutions ), the most general solution is
X∞
nπx −a2 n2 π2 t
u(x, t) = Bn sin e ℓ2 (3)

n=1
Applying condition (iii) in equation (3), we get
120
We have (iii) ⇒ u(x, t = 0) = f (x) = x, 0 < x < ℓ

X∞
nπx 120
∴ (3) ⇒ Bn sin = f (x) = x (4)
ℓ ℓ
n=1
To find Bn ,
Expand f (x) in a half range Fourier sine series in the interval (0, ℓ)
X∞
nπx
f (x) = bn sin (5)

n=1
Zℓ
2 nπx
where bn = f (x) sin dx
ℓ ℓ
0
From (4) and (5), we have Bn = bn

Zℓ
2 nπx
∴ Bn = f (x) sin dx
ℓ ℓ
0
Zℓ
2 120 nπx
= x sin dx
ℓ ℓ ℓ
0
"   !#ℓ
240 − cos nπx
ℓ − sin nπx

= 2 x nπ − (1) 
ℓ nπ 2

  ℓ  0
240 ℓ
= 2 −ℓ (−1)n + 0 − (0 + 0)
ℓ nπ
240
∴ Bn = (−1)n+1

∴ The required most general solution (3) is

X 240 nπx −a2 n22 π2 t
u(x, t) = (−1)n+1 sin e ℓ
nπ ℓ
n=1
68 Part-A : Unit III - APPLICATIONS OF P.D.E.(A.P.D.E.)

3.2.2.2 Steady state conditions and non-zero boundary conditions

Example 3.7. Two ends A and B of rod of length 20 cm have the temperatures at 30◦ C and 80◦ C
respectively until steady state conditions prevail. Then the temperature at the ends A and B changed
to 40◦ C and 60◦ C respectively. Find the resulting temperature distribution u(x, t) taking origin at
A.

Solution : The temperature function u(x, t) satisfies the one dimensional heat equation is
∂u ∂ 2u
= a2 2 . (I)
∂t ∂x
k Thermal conductivity
where a2 = =
ρc (Density)(Specific heat)
Solving the equation (I) by method of separation of variable, we get the following three possible
solutions.
 2 2
(1)u(x, t) = A1 epx + A2 e−px A3 ea p t

2 2
(2)u(x, t) = (A4 cos px + A5 sin px) A6 e−a p t

(3)u(x, t) = (A7 + A8 x) A9

out of these solutions, we have to select that solution which suits the physical nature of the
problem and the boundary conditions.
First let us find the temperature distribution at any distance x, before the end A and B are
reduced to zero. Prior to the temperature change at the ends A and B, when t = 0, the heat flow
was independent of time (steady state conditions). When the temperature u depends only on x and
not on t,
When steady state conditions prevail
(I) reduces to
∂u ∂ 2u
=0⇒ =0 (II)
∂t ∂x2
The general solution of (II) is u = ax + b (III)
Where a and b are arbitrary constants. Given that u = 30 when x = 0
∴ From (III), we get 30 = a(0) + b ⇒ b = 30
Also given that u = 80 when x = 20
From (III),

20a + b = 80
20a + 30 = 80
5
a=
2

u = 30◦ C u = 80◦ C
A B
x=0 x = 20
MA8353 Transforms and Partial Differential Equations by K A Niranjan Kumar 69

5
∴ (III) becomes u = x + 30, which is initial temperature distribution. (IV)
2
After steady state conditions with non-zero boundary conditions
When the temperature at A and B are reduced to 40◦ C and 60◦ C, the state is not more steady
state. For this transient state, the boundary conditions are

(i)u(x = 0, t) = 40, ∀t > 0


(ii)u(x = ℓ, t) = 60, ∀t > 0 where ℓ = 20
5
(iii)u(x, t = 0) = f (x) = x + 30, 0 < x < ℓ
2

u = 40◦ C u = 60◦ C
A B
x=0 x = 20

Here we have non zero boundary values. In such case, the temperature function u(x, t) is given
by
u(x, t) = us (x) + ut (x, t) (V)
Where us (x) is a solution of (I) involving x only and satisfying the boundary conditions (i) and
(ii). ut (x, t) is a function defined by (V) satisfying (I).
i.e., us (x) is a steady state solution of (I) and ut (x, t) may therefore regarded as transient solution
which decreases with increase of time t.
us (x) satisfies (I).
To find us (x)
The general solution of (I) is us (x) = a1 x + b1 (VI)
Where a1 and b1 are arbitrary constants.
By the condition (i), we have u = 40 when x = 0
From (VI), we get us (0) = b1 = 40
By the condition (ii), we have u = 60 when x = ℓ
From (VI), us (ℓ) = a1 ℓ + b1 = 60
a1 ℓ + 40 = 60
20
a1 =

20
∴ (VI) becomes us (x) = x + 40 (VII)

To find ut (x, t) [zero boundary conditions]
Hence the boundary conditions for the transient solution ut (x, t) by using (V) are
70 Part-A : Unit III - APPLICATIONS OF P.D.E.(A.P.D.E.)

(iv)ut (x = 0, t) = u(0, t) − us (0) = 40 − 40 = 0, ∀t > 0


(v)ut (x = ℓ, t) = u(ℓ, t) − us (ℓ) = 60 − 60 = 0, ∀t > 0
(vi)ut (x, t = 0) = f (x) = u(x, 0) − us (x)
5   20 
= x + 30 − x + 40
 25  ℓ
= x + 30 − (x + 40) (∵ ℓ = 20)
2
3
= x − 10, 0 < x < ℓ = 20
2
The suitable solution which satisfies above boundary conditions is
2 2
ut (x, t) = (A cos px + B sin px) e−a p t
(1)
Applying condition (iv) in equation (1) we get
We have

(iv) ⇒ ut (x = 0, t) = 0
2 2
(1) ⇒ Ae−a p t
=0
2 2
Here e−a p t
6= 0 (∵ it is defined for all t)
∴A=0
2 2
∴ Now (1) ⇒ ut (x, t) = B sin pxe−a p t
(2)
Applying condition (v) in equation (2) we get
We have

(v) ⇒ ut (x = ℓ, t) = 0
2 2
(2) ⇒ B sin pℓe−a p t
=0
2 2
Here e−a p t
6= 0 (∵ it is defined for all t)
B 6= 0 (∵ it gives trivial solution )
∴ sin pℓ = 0
= sin nπ

∴p=

nπx 2 n2 π 2
−a t
∴ Now (2) ⇒ ut (x, t) = B sin e ℓ2

∴ By superposition principle(i.e., adding all such above solutions ), the most general solution is
X∞
nπx −a2 n2 π2 t
ut (x, t) = Bn sin e ℓ2 (3)

n=1
Applying condition (vi) in equation (3), we get
3
We have (vi) ⇒ ut (x, t = 0) = f (x) = x − 10, 0 < x < ℓ = 20
2
X∞
nπx 3
Bn sin = f (x) = x − 10 (4)
ℓ 2
n=1
MA8353 Transforms and Partial Differential Equations by K A Niranjan Kumar 71

To find Bn ,
Expand f (x) in a half range Fourier sine series in the interval (0, ℓ = 20)
X∞
nπx
f (x) = bn sin (5)

n=1
Zℓ
2 nπx
where bn = f (x) sin dx
ℓ ℓ
0
From (4) and (5), we have Bn = bn

Zℓ Zℓ  
2 nπx 2 3 nπx
∴ Bn = f (x) sin dx = x − 10 sin dx
ℓ ℓ ℓ 2 ℓ
0 0
" !#ℓ
2
3  − cos nπx  3 − sin nπx
ℓ ℓ
= x − 10 nπ − 
ℓ 2 2 nπ 2
ℓ ℓ 0
      
2 3ℓ ℓ 10ℓ
= − 10 (− cos nπ) + 0 − +0
ℓ 2 nπ nπ
1
h 400 200
i
= − (−1)n − (∵ ℓ = 20)
10 nπ nπ
20
∴ Bn = − [1 + 2(−1)n ]

∴ The required most general solution (3) is


X∞ n o nπx
20 −a2 n2 π 2 t
ut (x, t) = − [1 + 2(−1)n ] sin e ℓ2
nπ ℓ
n=1
∞ 
X 
20 [1 + 2(−1)n ] nπx −a2 n22 π2 t
Hence u(x, t) = x + 40 − sin e ℓ
π n ℓ
n=1

3.2.3 Two dimensional heat flow equations

3.2.3.1 Examples of two dimensional finite plates

Examples of Type I : f (x) 6= 0 parallel to X axis

Example 3.8. Solve for the steady state temperature at any point of a rectangular plate of sides a
and b insulated on the lateral surface and satisfy u(0, y) = 0, u(a, y) = 0, u(x, b) = 0 and u(x, 0) =
x(a − x).

Solution :
72 Part-A : Unit III - APPLICATIONS OF P.D.E.(A.P.D.E.)

u(x, b) = 0
y=b

u(a, y) = 0
u(0, y) = 0

x=a
x=0
y=0
X
O u(x, 0) = f (x)
(0, 0)

Let u(x, y) be the temperature distribution satisfying the two dimensional heat flow equation in
steady state conditions is
∂ 2u ∂ 2u
+ =0 (I)
∂x2 ∂y 2
Solving the equation (I) by method of separation of variable, we get the following three possible
solutions.

(1)u(x, t) = C1 epx + C2 e−px (C3 cos py + C4 sin py)

(2)u(x, t) = (C5 cos px + C6 sin px) C7 epy + C8 e−py
(3)u(x, t) = (C9 + C10 x) (C11 + C12 y)
out of these solutions, we have to select that solution which suits the physical nature of the
problem and the boundary conditions.
For this problem, the boundary and initial conditions are

(i)u(x = 0, y) = 0 for 0 < y < b


(ii)u(x = a, y) = 0 for 0 < y < b
(iii)u(x, y = b) = 0 for 0 < x < a
(iv)u(x, y = 0) = f (x) = x(a − x)

= ax − x2 for 0 < x < a
The suitable solution which satisfies above boundary conditions is

u(x, y) = (A cos px + B sin px) Cepy + De−py (1)
Applying condition (i) in equation (1) we get

We have (i) ⇒ u(x = 0, y) = 0



(1) ⇒ A Cepy + De−py = 0
Here Cepy + De−py 6= 0 (∵ it is defined ∀y)
∴A=0
MA8353 Transforms and Partial Differential Equations by K A Niranjan Kumar 73


∴ Now (1) ⇒ u(x, y) = B sin px Cepy + De−py (2)
Applying condition (ii) in equation (2) we get

We have (ii) ⇒ u(x = a, y) = 0



(2) ⇒ B sin pa Cepy + De−py = 0
Here Cepy + De−py 6= 0 (∵ it is defined ∀y)
∴ B 6= 0 (∵ it gives trivial solution )
sin pa = 0
= sin nπ
∴ pa = nπ

i.e., p =
a
 nπx   nπy

− nπ
a y
∴ Now (2) ⇒ u(x, y) = B sin Ce + De a (3)
a
Applying condition (iii) in equation (3), we get

We have (iii) ⇒ u(x, y = b) = 0


 nπx  nπ nπ 
(3) ⇒ B sin Ce a b + De− a b = 0
a  nπx 
Here sin 6= 0 (∵ it is defined ∀x)
a
B 6= 0 (∵ it gives trivial solution )

a b − nπ
a b
∴ Ce + De =0
nπ nπ
De− a b = −Ce a b


eab
D = −C − nπ b
e a
 nπx   nπ eab

nπy

∴ (3) ⇒ u(x, y) = B sin Ce a y
− C − nπ b e− a
a e a
 nπx   e nπ nπ
a (y−b) − e a (b−y)

= BC sin nπ
a e− a b

 nπx  nπ nπ 
b
= BCe a sin e− a (b−y) − e a (b−y)

anπx  nπ nπ 
b
= −BCe a sin e a (b−y) − e− a (b−y)
a 
 nπx

b nπ
= −2BCe a sin sinh (b − y)
a a
∴ By superposition principle(i.e., adding all such above solutions ), the most general solution is

X  nπx  nπ
u(x, y) = Bn sin sinh (b − y) (4)
a a
n=1
74 Part-A : Unit III - APPLICATIONS OF P.D.E.(A.P.D.E.)

Applying condition (iv) in equation (4), we get



We have (iv) ⇒ u(x, y = 0) = f (x) = ax − x2

X  nπx  nπ
(4) ⇒ Bn sin sinh b = f (x)
a a
n=1

X  nπx 
∴ An sin = f (x) (5)
a
n=1


An = Bn sinh b
a
To find An ,
Expand f (x) in a half range Fourier sine series in the interval (0, a)
X∞
nπx
f (x) = bn sin (6)
a
n=1
Za
2 nπx
where bn = f (x) sin dx
a a
0
From (5) and (6), we have An = bn
Za
2 nπx
An = f (x) sin dx
a a
0
Za
2  nπx
= ax − x2 sin dx
a a
"0   ! !#a
2 − cos nπx − sin nπx cos nπx
= (ax−x2 ) nπ
a
−(a−2x) a
 +(−2) a

a nπ 2 nπ 3
a a a
"  3 #0 a
2 a nπx a
 2 nπx ℓ nπx
= (ax−x2 ) sin +(a − 2x) cos +2 sin
a nπ a nπ ℓ nπ ℓ
0
  a 3    a 3 
2
= 0+0−2 (−1)n − 0 + 0 − 2
a nπ nπ
  
2 a 3
= 2 [1 − (−1)n ]
a nπ
4a2
= 3π3
[1 − (−1)n ]
n

 8a2
if n is odd
∴ An = n3 π 3
 0 if n is even

 8a2
nπb
An 3 3
sinh if n is odd
∴ Bn = nπb
= n π a
sinh a  0 if n is even
∴ The required most general solution (4) is
MA8353 Transforms and Partial Differential Equations by K A Niranjan Kumar 75


X 8a2 nπb nπx nπ
u(x, y) = sinh sin sinh (b − y)
n3 π 3 a a a
n=odd

8a2 X 1 nπb nπx nπ
= 3 3
sinh sin sinh (b − y)
π n a a a
n=odd

8a2 X
(2n−1)πx
sin
a (2n − 1)π
= 3
sinh (b − y)
π
n=1
(2n − 1) sinh nπb
3
a
a

Examples of Type II(f (y) 6= 0 parallel to Y axis) and III(f (x) 6= 0 & f (y) 6= 0)

Example 3.9. Solve Find the steady state temperature distribution at a point of a rectangular plate,
if, two adjacent edges are kept at 0◦ C and other at 100◦ C.

Solution : Let X & Y axes with 0◦ C and X = a & Y = b with 100◦ C. Then
Y

u(x, b) = 100◦
y=b

u(a, y) = 100◦
u(0, y) = 0◦

x=a
x=0

y=0
X
O u(x, 0) = 0◦
(0, 0)

Let u(x, y) be the temperature distribution satisfying the two dimensional heat flow equation in
steady state conditions is
∂ 2u ∂ 2u
+ =0 (I)
∂x2 ∂y 2
Solving the equation (I) by method of separation of variable, we get the following three possible
solutions.

(1)u(x, t) = C1 epx + C2 e−px (C3 cos py + C4 sin py)

(2)u(x, t) = (C5 cos px + C6 sin px) C7 epy + C8 e−py
(3)u(x, t) = (C9 + C10 x) (C11 + C12 y)
out of these solutions, we have to select that solution which suits the physical nature of the
problem and the boundary conditions.
Let the temperature along the adjacent edges AB and BC be 100◦ C and the temperature along
the other edges OC and OA be 0◦ C.
The Boundary conditions are
76 Part-A : Unit III - APPLICATIONS OF P.D.E.(A.P.D.E.)

(i)u(x = 0, y) = 0 for 0 < y < b


(ii)u(x, y = 0) = 0 for 0 < x < a
(iii)u(x = a, y) = 100 for 0 < y < b
(iv)u(x, y = b) = 100 for 0 < x < a

Y Y

u(x, b) = 100◦ u(x, b) = 0◦


y=b y=b

u(a, y) = 100◦
u(a, y) = 0◦
u(0, y) = 0◦

u(0, y) = 0◦
x=a

x=a
x=0

x=0
+
y=0 y=0
X X
O u(x, 0) = 0◦ O u(x, 0) = 0◦
(0, 0) (0, 0)

∴ u(x, y) = u1 (x, y) + u2 (x, y).


Where u1 (x, y) and u2 (x, y) are solutions of (I) and further u1 (x, y) is the temperature at the
edge BC kept at 100◦ C and the other three sides at 0◦ C while u2 (x, y) is the temperature at the
edge AB maintained at 100◦ C and the other three edges at 0◦ C.
b and B
Boundary conditions for the function u1 (x, y) and u2 (x, y) as A b are
b
A b
B
(i) u1 (x = 0, y) = 0 in 0 < y < b (v) u2 (x, y = 0) = 0 in 0 < x < a
(ii) u1 (x = a, y) = 0 in 0 < y < b (vi) u2 (x, y = b) = 0 in 0 < x <a
(iii) u1 (x, y = 0) = 0 in 0 < x < a (vii) u2 (x = 0, y) = 0 in 0 < y < b
(iv) u1 (x, y = b) = 100 in 0 < x < a (viii) u2 (x = a, y) = 100 in 0 < y < b
 
b is
The suitable solution which satisfies above boundary conditions A
py −py

u1 (x, y) = (A cos px + B sin px) Ce + De (1)
Applying condition (i),(ii),(iii) as in previous problem, we get
∴ By superposition principle(i.e., adding all such above solutions ), the most general solution is
X∞  nπx  nπy
u(x, y) = Cn sin sinh (4)
a a
n=1
Applying condition (iv) in equation (4), we get

We have (iv) ⇒ u(x, y = b) = f (x) = 100



X  nπx  nπb
(4) ⇒ Cn sin sinh = f (x) = 100
a a
n=1

X  nπx 
∴ Bn sin = f (x) = 100 (5)
a
n=1
MA8353 Transforms and Partial Differential Equations by K A Niranjan Kumar 77

nπb
where Bn = Cn sinh
a
To find Bn ,
Expand f (x) in a half range Fourier sine series in (0, a), we get
X∞
nπx
f (x) = bn sin (6)
a
n=1
Za
2 nπx
where bn = f (x) sin dx
a a
0
From (5) and (6), we have Bn = bn
Za Za
2 nπx 200 nπx
Bn = 100 sin dx = sin dx
a a a a
0 0
 nπx a
200 − cos a
= nπ
a
 200  a 0
200
=− [cos nπ − 1] = − [(−1)n − 1]
nπ nπ
( 400
if n is odd
∴ Bn = nπ
0 if n is even

 400
1
if n is odd
Bn nπ sinh nπb
∴ Cn = = a
sinh nπb
a

0 if n is even
∴ The required solution (4) is

X 400 1 nπx nπy
u1 (x, y) = sin sinh
nπ sinh nπb
a
a a
n=odd
X∞ (2n−1)πx
400 sin a (2n − 1)π
= (2n−1)πb
sinh y
π a
n=1 (2n − 1) sinh a

This is the required solution of u1 (x, y).  


b is
Similarly, the suitable solution which satisfies above boundary conditions B
px −px

u2 (x, y) = Ae + Be (C cos py + D sin py) (7)
Similarly by applying condition (v),(vi),(vii) and (viii), we get the required solution of u2 (x, y) as


X 400 1 nπy nπx
u2 (x, y) = nπa nπa sin sinh
nπ sinh b sinh b b b
n=odd
X∞ (2n−1)πy
400 sin b (2n − 1)π
= (2n−1)πa
sinh x
π b
n=1 (2n − 1) sinh b

This is the required solution of u2 (x, y).


∴ Finally, the required most general solution = u(x, y) = u1 (x, y) + u2 (x, y).
78 Part-A : Unit III - APPLICATIONS OF P.D.E.(A.P.D.E.)

3.2.3.2 Examples of two dimensional infinite plates

Examples of Type (IV) : f (x) 6= 0 parallel to X axis

Example 3.10. A rectangular plate with insulated surfaces is 10 cm wide and so long compared to
its width that it may be considered infinite in length without introducing an appreciable error. If
the temperature along the short edge y = 0 is given by
(
20x, 0 < x < 5
u(x, 0) =
20(10 − x), 5 < x < 10

while the two long edges x = 0 and x = 10 as well as the other short edge are kept at 0◦ C, find the
temperature function u(x, y) in steady state.

Solution :
Y

u(x, ∞) = 0◦
y→∞

u(10, y) = 0◦
u(0, y) = 0◦

x = ℓ = 10
x=0

y=0
X
O u(x, 0) = f (x)
(0, 0)

Let u(x, y) be the temperature at any point P (x, y) in the steady state. Then u(x, y) satisfying the
two dimensional heat flow equation in steady state conditions is
∂ 2u ∂ 2u
+ =0 (I)
∂x2 ∂y 2
Solving the equation (I) by method of separation of variable, we get the following three possible
solutions.

(1)u(x, t) = C1 epx + C2 e−px (C3 cos py + C4 sin py)

(2)u(x, t) = (C5 cos px + C6 sin px) C7 epy + C8 e−py
(3)u(x, t) = (C9 + C10 x) (C11 + C12 y)

out of these solutions, we have to select that solution which suits the physical nature of the
problem and the boundary conditions.
For this problem, the boundary and initial conditions are
MA8353 Transforms and Partial Differential Equations by K A Niranjan Kumar 79

(i)u(x = 0, y) = 0 for all y


(ii)u(x = 10, y) = 0 for all y
(iii)u(x, y → ∞) = 0 for 0 < x < 10
(
20x, 0 < x < 5
(iv)u(x, y = 0) = f (x) =
20(10 − x), 5 < x < 10

The suitable solution which satisfies above boundary conditions is



u(x, y) = (A cos px + B sin px) Cepy + De−py (1)
Applying condition (i),(ii),(iii) as in previous problem, we get
∴ By superposition principle(i.e., adding all such above solutions ), the most general solution is

X  nπ  nπy
u(x, y) = En sin x e− 10 (4)
10
n=1
Applying condition (iv) in equation (4), we get
We have (iv) ⇒ u(x, y = 0) = f (x)
(

X  nπx  20x, 0 < x < 5
(4) ⇒ En sin = f (x) = (5)
ℓ 20(10 − x), 5 < x < 10
n=1
To find En ,
Expand f (x) in a half range series in (0, 10), we have
X∞
nπx
f (x) = bn sin (6)
10
n=1
Z10
2 nπx
where bn = f (x) sin dx
10 10
0
From (5) and (6), we have En = bn

Z10
2 nπx
En = f (x) sin dx
10 10
0 5 
Z Z10
1 nπx nπx 
=  20x sin dx + 20(10 − x) sin dx
5 10 10
 05 5

Z Z10
nπx nπx 
= 4 x sin dx + (10 − x) sin dx
10 10
0 5
80 Part-A : Unit III - APPLICATIONS OF P.D.E.(A.P.D.E.)

(
10
nπx 10
  2  nπx
 5
=4 x − cos − (1) − sin
nπ 10 nπ 10 0
 )
10 nπx
 10
  2  nπx
10
+ (10 − x) − cos − (−1) − sin
nπ 10 nπ 10 5
nh 50  nπ
 100
 nπ
i
=4 − cos +2π2
sin − [0 + 0]
nπ 2
h −50  n nπ  2
100
 nπ
io
+ [0 + 0] − cos − 2 2 sin
2 nπ n π 2
800 nπ
∴ En = 2 2 sin
n π 2
∴ The required most general solution (4) is

X 800 nπ nπx − nπy
u(x, y) = sin sin e 10
n2 π 2 2 10
n=1

800 X 1 nπ nπx − nπy
= 2 2
sin sin e 10
π n 2 10
n=1

Examples of Type V : f (y) 6= 0 parallel to Y axis

Example 3.11. An infinitely long rectangular plate with insulated surface is 10 cm wide. The two
long edges and one short
( edge are kept at zero temperature, while the other short edge x = 0 is kept
20y, 0 < y < 5
at temperature u = . Find the temperature function u(x, y) in steady
20(10 − y), 5 < y < 10
state.

Solution :
Y

u(x, 10) = 0◦
y = 10◦
u(0, y) = f (y)

u(∞, y) = 0◦
x=0

x→∞

y=0
X
O u(x, 0) = 0
(0, 0)

Let u(x, y) be the temperature at any point P (x, y) in the steady state. Then u(x, y) satisfying the
two dimensional heat flow equation in steady state conditions is
∂2u ∂2u
+ =0 (I)
∂x2 ∂y 2
MA8353 Transforms and Partial Differential Equations by K A Niranjan Kumar 81

Solving the equation (I) by method of separation of variable, we get the following three possible
solutions.

(1)u(x, t) = C1 epx + C2 e−px (C3 cos py + C4 sin py)

(2)u(x, t) = (C5 cos px + C6 sin px) C7 epy + C8 e−py
(3)u(x, t) = (C9 + C10 x) (C11 + C12 y)

out of these solutions, we have to select that solution which suits the physical nature of the
problem and the boundary conditions.
For this problem, the boundary and initial conditions are

(i)u(x, y = 0) = 0 for all x


(ii)u(x, y = 10) = 0 for all x
(iii)u(x → ∞, y) = 0 for 0 < y < 10
(
20y, 0 < y ≤ 5
(iv)u(x = 0, y) = f (y) =
20(10 − y), 5 < y ≤ 10

The suitable solution which satisfies above boundary conditions is



u(x, y) = Aepx + Be−px (C cos py + D sin py) (1)
Applying condition (i) in equation (1) we get

We have (i) ⇒ u(x, y = 0) = 0



(1) ⇒ Aepx + Be−px C = 0
Here Aepx + Be−px 6= 0 (∵ it is defined ∀x)
∴C=0

∴ Now (1) ⇒ u(x, y) = Aepx + Be −px
D sin py (2)
Applying condition (ii) in equation (2) we get

We have (ii) ⇒ u(x, y = 10) = 0



(2) ⇒ Aepx + Be−px D sin 10p = 0
Here Aepx + Be−px 6= 0 (∵ it is defined ∀x)
D 6= 0 (∵ it gives trivial solution )
∴ sin 10p = 0
= sin nπ

∴p=
10
nπx nπx  nπy
∴ Now (2) ⇒ u(x, y) = Ae 10 + Be− 10 D sin (3)
10
Applying condition (iii) in equation (3), we get
82 Part-A : Unit III - APPLICATIONS OF P.D.E.(A.P.D.E.)

We have (iii) ⇒ u(x → ∞, y) = 0


 nπy
(3) ⇒ Ae∞ + Be−∞ D sin =0
10
nπy
(Ae∞ ) D sin =0
10
nπy
Here sin 6= 0 (∵ it is defined ∀y)
10
B 6= 0 (∵ it gives trivial solution )
∴A=0 (∵ e∞ = ∞)

nπx   sin nπy 


∴ Now (3) ⇒ u(x, y) = Be 10 D
10
nπy − nπx 
= BD sin e 10
10
nπy − nπx
= E sin e 10
10
where E = BD
∴ By superposition principle(i.e., adding all such above solutions ), the most general solution is

X nπy nπx
u(x, y) = En sin e− 10 (4)
10
n=1
Applying condition (iv) in equation (4), we get

We have (iv) ⇒ u(x = 0, y) = f (y)



X nπy
(4) ⇒ En sin = f (y) (5)
10
n=1

To find En ,
Expand f (y) in a half range Fourier sine series in the interval (0, 10)
X∞
nπy
f (y) = bn sin (6)
10
n=1
Z10
2 nπy
where bn = f (y) sin dy
10 10
0
From (5) and (6), we have En = bn
MA8353 Transforms and Partial Differential Equations by K A Niranjan Kumar 83

 
Z5 Z10
1 nπy nπy 
En = 20y sin dy + 20(10 − y) sin dy
5 10 10
 05 5

Z Z10
nπy nπy 
= 4  y sin dy + (10 − y) sin dy
10 10
0 5
(
10
 nπy
  10 2  nπy
 5
=4 y − cos − (1) − sin
nπ 10 nπ 10 0
 )
10 nπy
   10 2  nπy
10
+ (10 − y) − cos − (−1) − sin
nπ 10 nπ 10 5
nh 50  nπ 100
 nπ
 i
=4 − cos + 2 2 sin − [0 + 0]
nπ 2
h −50  nnππ 1002  nπ io
= + [0 + 0] − cos − 2 2 sin
nπ 2 n π 2
800 nπ
∴ En = 2 2 sin
n π 2
∴ The required most general solution (4) is

X 800 nπ nπy − nπx
u(x, y) = sin sin e 10
n2 π 2 2 10
n=1

800 X 1 nπ nπy − nπx
= 2 2
sin sin e 10
π n 2 10
n=1

3.3 Assignment III[Applications of Partial Differential Equations]

1. A string is stretched and fastened to two points ℓ apart. Motion is started by displacing the
string into the form y = k(ℓx−x2 ) from which it is released at time t = 0. Find the displacement
of any point of the string at a distance x from one end at any time t.

2. A tightly stretched string of length ‘2ℓ’ has its ends fastened at x = 0, x = 2ℓ. The mid point
of the string is then taken to height ‘b’ and then released from rest in that position. Find the
lateral displacement of a point of the string at time ‘t’ from the instant of release.

3. A tightly stretched string of length ‘ℓ’is initially


 at rest in its equilibrium position and each of
πx
its points is given the velocity V0 sin3 Find the displacement of y(x, t).

4. A string is stretched between two fixed points at a distance 2ℓ apart and the points of the
string (
are given initial velocities ν where
cx
in 0 ≤ x ≤ ℓ
ν = c ℓ x being the distance from one end point. Find the
(2ℓ − x) in ℓ ≤ x ≤ 2ℓ

displacement of the string at any subsequent time.
84 Part-A : Unit III - APPLICATIONS OF P.D.E.(A.P.D.E.)

5. The ends of A and B of a rod ℓ c.m. long have their temperature kept at 30◦ C and 80◦ C, until
steady state conditions prevail. The temperature of the end B is suddenly reduced to 60◦ C and
that of A is increased to 40◦ C. Find the temperature distribution in the rod after time t.

6. Find the solution of the one dimensional diffusion equation satisfying theboundary
 conditions:
∂u
(i) u is bounded as t → ∞ (ii) = 0 for all t
∂x x=0
 
∂u
(iii) = 0 for all t (iv) u(x, 0) = x(a − x), 0 < x < a
∂x x=a
7. Find the steady state temperature distribution in a rectangular plate of sides a and b insulated
at the lateral surface and satisfying the boundary conditions u(0, y) = u(a, y) = 0 for 0 ≤ y ≤ b;
u(x, b) = 0 and u(x, 0) = x(a − x) for 0 ≤ x ≤ a.

8. A square plate is bounded by the lines x = 0, y = 0, x = 20 and y = 20. Its faces are insulated.
The temperature along the upper horizontal edge is given by while the other two edges are kept
at 0c ircC. Find the steady state temperature distribution in the plate.

9. A rectangular plate with insulated surface is 10 cm wide and so long compared to its width that
it may be considered infinite in length
( without introducing appreciable error. The temperature
20x, 0 ≤ x ≤ 5
at short edge y = 0 is given by and all the other three edges are
20(10 − x), 5 ≤ x ≤ 10
kept at 0◦ C. Find the steady-state temperature at any point of the plate.

10. An infinitely long rectangular plate with insulated surface is 10 cm wide. The two long edges
and one short edge
( are kept at zero temperature, while the other short edge x = 0 is kept at
20y, 0 < y ≤ 5
temperature u = . Find the temperature function u(x, y) in steady
20(10 − y), 5 < y ≤ 10
state.

11. A rectangular plate with insulated surface is 20 cm wide and so long compared to its width
rectangular plate with insulated surface is 20 cm wide and so long compared to its width
that it may be considered infinite in length without introducing an appreciable error. If the
temperature of the short edge x = 0 is given by and the two long edges as well as the other
short edge are kept at 0◦ C. Find the steady state temperature distribution in the plate.
4 Fourier Transforms(F.T.)

4.1 Part-A

1. If F (s) is the Fourier transform of f (x),


write the formula for the Fourier transform
Z∞
of f (x) cos ax. 1
F [f (x − a)] = √ eis(a+t) f (t)dt
Soln : 2π
−∞
F [f (x) cos ax] Z∞
1
Z∞ = eias √ eist f (t)dt
1 2π
=√ eisx f (x) cos ax dx −∞
2π ias
−∞ =e F (s)
Z∞  
1 isx eiax + e −iax
=√ e f (x) dx
2π 2 (
−∞ 1, |x| < a
Z∞ 3. If the Fourier transform of f (x) =
1   0, |x| > a
= √ ei(s+a)x + ei(s−a)x f (x) dx r Z∞
2 2π 2 sin as
 sin t 2
−∞ is , evaluate dt.
 π s t
Z∞
1 1 0 r
= √ ei(s+a)x f (x) dx
2 2π 2 sin as
−∞
Soln : Given F [f (x)] = F (s) = .
 π s
Z∞ By Parseval’s identity,
1
+√ ei(s−a)x f (x) dx Z∞ Z∞

−∞ |f (x)|2 dx = |F (s)|2 ds
1
= [F (s + a) + F (s − a)] −∞ −∞
2 Za Z∞ r !2
2 sin as
2. If the Fourier transform of f (x) is F (s). (1)2 dx = ds
π s
What is the Fourier transform of f (x − a). −a −∞
Soln : Z∞  
Z∞ 2 sin2 as
1 2a = ds
F [f (x − a)] = √ eisx f (x − a) dx, π s2
2π −∞
−∞
Set x − a = t. Then dx = dt. Set as = t. Then ds = dt/a.
When x = −∞, t = −∞ and when When s = −∞, t = −∞ and when s =
x = ∞, t = ∞. ∞, t = ∞.
86 Part-A : Unit IV - FOURIER TRANSFORMS (F.T.)

Z∞   5. If F (s) is the Fourier transform of f (x),


1 sin2 t
a= dt/a prove that the Fourier transform of f (ax)
π (t/a)2 1
−∞
is F (s/a) , a 6= 0.
Z∞   |a|
a sin2 t WKT
a= dt Z∞
π t2 1
−∞ Soln : F [f (x)] = √ f (x) eisx dx
Z∞   2π
1 sin2 t −∞
1= dt Z∞
π t2 1
−∞ F [f (ax)] = √ f (ax) eisx dx
Z∞   2π
2 sin2 t −∞
⇒1= dt Set t = ax. Then dt = adx.
π t2
0 When x = −∞, t = −∞ and when x =
Z∞   ∞, t = ∞.
sin2 t π
dt =
t2 2 Z∞
1 dt
0 F [f (ax)] = √ f (t) eist/a
4. Find the(Fourier transform of 2π a
−∞
x, |x| < a Z∞
f (x) = . 1
0, |x| > a = √ f (t) eist/a dt
Soln : a 2π
−∞
Z∞ Z∞
1 1
F [f (x)] = √ eisx f (x) dx = √
s
f (t) ei( a )t dt
2π a 2π
−∞ −∞
Za Za Z∞
1 1 1
=√ eisx (0) dx + √ eisx xdx = √
s
f (x) ei( a )x dx
2π 2π a 2π
−∞ −a −∞
Z∞
1 [∵ x is a dummy variable]
+√ eisx (0) dx
2π 1
a = F (s/a) , a 6= 0
a
Za
1 Similarly if a < 0,
=√ eisx xdx
2π Z∞
−a −1
Za F [f (ax)] = √ f (ax) eisx dx
1 2π
=√ x(cos sx + i sin sx)dx −∞
2π Z∞
−a −1 s dt
Za Za =√ f (t) ei( a )t
1 i 2π a
=√ x cos sxdx + √ x sin sxdx −∞
2π 2π −1
−a −a = F (s/a)
Za a
2i 1
=0+ √ x sin sxdx ∴ In general, F [f (ax)] = F (s/a)
2π |a|
r h 0 i x2 s2
2 sin as − as cos as 6. If the Fourier transform of e− 2 is e− 2 ,
=i x2
π s2 what is the Fourier transform of xe− 2 ?
MA8353 Transforms and Partial Differential Equations by K A Niranjan Kumar 87

Soln : If F (s) = F [f (x)], then Fourier Cosine transform pair:

d Fourier Cosine transform of f (x) is


F [xf (x)] = −i F (s) r Z∞
  ds   2
− x2
2
d − x2
2 FC [f (x)] = f (x) cos sx dx = FC (s)
F xe = −i F e π
ds 0
 
d s 2 Inverse Fourier Cosine transform of
= −i e− 2
ds FC [f (x)] = FC [s] is
2
− s2 r Z∞
= −ise 2
f (x) = FC [f (x)] cos sx ds
π
0
7. Write the Fourier transform pair.
Soln : Fourier transform: 9. If FS (s) is the Fourier sine transform of
The Fourier transform of f (x) is given f (x), prove that the Fourier sine transform
1
by of f (ax) is FS (s/a).
a
Soln : WKT
Z∞ r Z∞
1 isx 2
F [f (x)] = √ f (x) e dx (1) FS [f (ax)] = f (ax) sin sxdx
2π π
−∞ 0
Set ax = t. Then dx = dt/a.
Inverse Fourier transform:
When x = 0, t = 0 and when x → ∞, t →
The Inverse Fourier transform of F (s)
∞.
is given by
r Z∞  st  dt
Z∞ 2
1 FS [f (ax)] = f (t) sin
f (x) = √ F [f (x)] e−isx ds (2) π a a
2π 0
−∞ r Z∞ s
1 2
The above equations (1) and (2) are jointly = f (t) sin t dt
a π a
0
called as Fourier Transform pair. hsi
1
= FS
a a
8. Write the Fourier sine and cosine transform
pair. 10. State Fourier integral theorem.
Soln : Soln : If f (x) is piecewise continuously
Fourier Sine transform pair: differentiable and absolutely integrable in
Fourier Sine transform of f (x) is (−∞, ∞), then
r Z∞
2 Z∞ Z∞
FS [f (x)] = f (x) sin sx dx = FS (s). 1
π f (x) = f (t) eis(x−t) dt ds

0 −∞ −∞
Inverse Fourier Sine transform of
(or)
FS [f (x)] = FS [s] is
r Z∞ Z∞ Z∞
1
2 f (x) = f (t) cos λ (x − t) dt dλ
f (x) = FS [f (x)] sin sx ds π
π 0 −∞
0
88 Part-A : Unit IV - FOURIER TRANSFORMS (F.T.)

11. Obtain the Fourier cosine transform of Soln : We have to prove


−ax
 
e , a > 0.. F f ′ (x) = −isF (s), if f (x) → 0 as x →
Soln :  ±∞
FC e−ax W.K.T.
r Z∞
2
= e−ax cos sxdx Z∞
π 1
r 0 F [f (x)] = √ f (x) eisx dx
∞ 2π
2 e−ax −∞
= (−a cos sx + s sin sx)
π a2 + s 2 0
Z∞
r  1
2 a
 =√ f’ (x) eisx dx
= 2π
π a + s2
2 −∞
Z∞
1
12. Obtain the Fourier sine transform of =√ eisx d [f (x)]

e−ax , a > 0. −∞
1
n ∞
Soln :  =√ eisx f (x) −∞
FS e−ax 2π 
r Z∞ Z∞ 
2 − f (x) eisx (is) dx
= e−ax sin sxdx 
π −∞
r 0 ∞ 1 
2 e−ax =√ [0 − 0]∞
−∞
= (−a sin sx − s cos sx) 2π 
π a2 + s 2 Z∞
r  0 
2 s
 −is f (x) eisx dx
= 
π a2 + s 2 −∞
 ∞ 
13. Obtain the Fourier sine transform of 1/x. Z 
r Z∞ −is
2 1 =√ f (x) eisx dx
Soln : FS (1/x) = sin sxdx 2π  
π x −∞
0
 
Set sx = t. Then dx = dt/s. F f ′ (x) = −isF (s)

When x = −∞, t = −∞ and when x =


∞, t = ∞. In general,
r Z∞
2 1 dt
FS (1/x) = t sin t F [f n (x)] = (−is)n F (s)
π s
s
0
r Z∞  
Z∞
2 1 sin t π if f, f ′ , f ′′ , ...f n−1 → 0 as x → ±∞
= sin tdt ∵ dt = 
π t t 2
0 0
r r
2π π 15. Prove that F (eiax f (x)) = F (s + a), where
= =
π2 2 F [f (x)] = F (s).
Z∞
14. Obtain the Fourier transform of the 1
Soln : WKT F [f (x)] = √ f (x) eisx dx

derivative of a function. −∞
MA8353 Transforms and Partial Differential Equations by K A Niranjan Kumar 89

Z∞ r Z1 Z2
  1 2
F eiax f (x) = √ eiax f (x) eisx dx = x cos sxdx + (2 − x) cos sxdx
2π π
−∞ 0 1
Z∞ 
Z∞
1
=√ f (x) ei(s+a)x dx + (0) cos sxdx

−∞
r 2n  sin sx   − cos sx o1
= F (s + a) 2
= (x) − (1)
π s s2 0
r n  sin sx   − cos sx o2
2
−a|x| + (2 − x) − (−1)
16. Find the Fourier transform of e , a > 0. π s s2 1
r  
Soln :  2 2 cos s − 1 − cos 2s
F e−a|x| =
π s2
Z∞
1 18. If F [f (x)] = F (s) then find the value of
=√ e−a|x| eisx dx
2π FS [f (x) cos ax].
−∞
Z∞ Soln :
1 −a|x| r Z∞
=√ e (cos sx + i sin sx)dx
2π 2
−∞ Fs [f (x) cos ax] = f (x) cos ax cos sxdx
π
Z∞ 0
1 −a|x| r Z∞
=√ e cos sxdx
2π 2
−∞
= f (x) cos sx cos axdx
π
Z∞ 0
i −a|x| r Z∞
+√ e sin sxdx 1 2
2π = f (x) [cos (s − a) x
−∞ 2 π
Z∞ 0

2 −a|x|
=√ e cos sxdx
2π + cos (s + a) x] dx
0 r ∞ 
r   Z
2 a  
= 1 2
π a2 + s2 = f (x) [cos (s − a) x] dx
2 π 
0
r ∞ 
 Z 
1 2
17. Obtain the Fourier cosine transform of + f (x) [cos (s + a) x] dx
 x, 0 < x < 1 2 π 
 0
f (x) = 2 − x, 1<x<2 . 1

 = [Fc (s + a) + Fc (s − a)]
0, x>2 2
Soln : 19. Find the Fourier sine transform of e−x .
Soln : Same as Problem 12th with a = 1.
F [f (x)] 20. Find the Fourier cosine transform of f (x),
(
r Z∞ cos x, 0 < x < a
2 if f (x) =
= f (x) cos sxdx 0, x > a
π
0 Soln :
90 Part-A : Unit IV - FOURIER TRANSFORMS (F.T.)

FC [f (x)] sin x
22. The area covered by the curve between
r Z∞ x
π
2 x = 0 and x = ∞ with x− axis is . Find
= f (x) cos sxdx 1 2
π
0 FS .
x
r Za Z∞
2 sin x π
= cos x cos sxdx Soln : Given dx =
π x 2
0 0
r Za   1 r Z∞
2 cos (s + 1) x + cos (s − 1) x 2 1
= dx ∴ FS = sin sxdx
π 2 x π x
0 0
 a Set sx = t. Then dx = dt/s.
1 sin (s + 1) x sin (s − 1) x
=√ + When x = −∞, t = −∞ and when
2π (s + 1) (s − 1)
 0
1 sin (s + 1) a sin (s − 1) a x = ∞, t = ∞.
=√ +
2π (s + 1) (s − 1)
r Z∞
21. If the Fourier cosine transform of e −x
is 2 1
r  Z∞ FS (1/x) = sin t dt/s
2 1
 dx π π t/s
2
, prove that 2
= 0
π s +1 (x2 + 1) 4 r Z∞
0 2 1
using Parseval’s identity. = sin t dt
π t
Soln : Given  0

 Z∞
sin t π
FC [f (x)] = FC e−x ∵ dt = 
r  t 2
2 1
 0
= FC (s) = r r
π s2 +1 2π π
= =
By Parseval’s identity, π2 2

Z∞ Z∞
|f (x)|2 dx = |FC (s)|2 ds 23. Find the
( Fourier sine transform of
0 0
sin x, 0 < x < a
f (x) = .
Z∞ Z∞ r  
2
0, a > 0
−x 2
e dx = 2 1
ds Soln :
2
π s +1
0 0
Z∞ Z∞ FS [f (x)]
2 1
e−2x dx = 2
ds r Z∞
π (s2 + 1)
0 0 2
= f (x) sin sxdx
 
−2x ∞
Z∞ π
e 2 1 0
= 2
dx r Za
−2 π (x2 + 1)
0
0 2
= sin x sin sxdx
π
(s is a dummy variable) 0
Z∞ r Za  
1 π 2 cos (s − 1) x − cos (s + 1) x
2
dx = = dx
(x2 + 1) 4 π 2
0 0
MA8353 Transforms and Partial Differential Equations by K A Niranjan Kumar 91

r  a
2 sin (s − 1) x sin (s + 1) x By Parseval’s identity,
= −
π (s − 1) (s + 1) 0 Z∞ Z∞
r   2
2 sin (s − 1) a sin (s + 1) a |f (x)| dx = |FS (s)|2 ds
= −
π (s − 1) (s + 1) 0 0
Z∞ Z∞ r 
 2
−x 2 2 s
e dx = ds
2
π s +1
0 0
Z∞ Z∞
24. Find the Fourier sine transform of −2x 2 s2
( e dx = 2
ds
x, 0 ≤ x < π π (s2 + 1)
f (x) = . 0 0
0, x ≥ π   Z∞
−2x ∞
e 2 x2
Soln : = dx
−2 π 2
FS [f (x)] 0 (x2 + 1)
0
r Z∞
2 (∵ s is a dummy variable)
= f (x) sin sxdx
π Z∞
0 x2 π
r Zπ 2
dx =
(x2 + 1) 4
2 0
= x sin sxdx
π
r h0  − cos sx   − sin sx iπ
26. State the convolution theorem and
2 Parseval’s Identity for Fourier transforms.
= (x) −(1)
π s s2 0
r h  − cos sπ   sin sπ i Soln : Convolution theorem for Fourier
2
= (π) + transform:
π s s2
F [f (x) ∗ g (x)] = F [f (x)] .F [g (x)]
= F (s) .G (s)
25. If the Fourier
r  sine transform of
2 s Parseval’s Identity for Fourier transform:
e−x is , prove that
π s2 + 1 Suppose F [f (x)] = F (s) &F [g(x)] = G (s) ,
Z∞
x2 dx π then
2
= using Parseval’s identity.
(x2 + 1) 4 1)If f (x) 6= g(x),
0
Soln : Given Z∞ Z∞
F [f (x)] .F [g(x)] ds = f (x).g(x) dx
−∞ −∞
−x

FS [f (x)] = FS e 2) If f (x) = g(x),
= FS (s) Z∞ Z∞
r   2
2 s |F [f (x)]| ds = |f (x)|2 dx
=
π s2 +1 −∞ −∞
92 Part-B : Unit IV - FOURIER TRANSFORMS (F.T.)

4.2 Part-B

4.2.1 Examples under Fourier Transform Pair

(
1 − |x| for |x| < 1
Example 4.1. Find the Fourier transform of f (x) = . Hence show that
0 otherwise
Z∞
sin2 x π
dx =
x2 2
0

(
1 − |x| for − 1 < x < 1
Solution : The given function can be written as f (x) = .
0 otherwise
Z∞
1
F (s) = F [f (x)] = √ f (x)eisx dx

−∞
Z1
1
=√ (1 − |x|)(cos sx + isin sx)dx

−1
Z1 Z1
1
=√ (1 − |x|) cos sxdx + i (1 − |x|) sin sxdx

−1 −1
Z1
2
=√ (1 − |x|) cos sxdx

0

[∵ (1 − |x|) sin sx is odd]


r Z1
2
= (1 − x) cos sxdx
π
r h0  i1
2 sin sx −cossx
= (1 − x) − (−1)
π s s2 0
r h i
2 sin sx cos sx 1
= (1 − x) −
π s s2 0
r h   i
2 cos s 1
= 0− − 0−
r  π s2 s2
2 1 − cos s

F (s) =
π s2
By inversion formula for Fourier transform
MA8353 Transforms and Partial Differential Equations by K A Niranjan Kumar 93

Z∞
1
f (x) = √ F (s)e−isx ds

−∞
Z∞ r  1 − cos s 
1 2
=√ (cos sx − i sin sx)ds
2π π s2
−∞
Z∞ h    i
1 1 − cos s 1 − cos s
= 2
cos sx − i sin sx ds
π s s2
−∞
Z∞  1 − cos s 
2
f (x) = cos sxds
π s2
0
h  1 − cos s   1 − cos s  i
∵ 2
cos sx is even and 2
sin sx is odd
s s
Z∞  
1 − cos s π
cos sxds = f (x)
s2 2
0
Put x = 0,
Z∞  
1 − cos s π
2
ds = .1
s 2
0
Z∞
2 sin2 2s π
ds =
s2 2
0
Z∞
sin2 2s ds π
 =
s 2 2 2
0 2
s ds
Put = t ⇒ = dt
2 2
Z∞
sin2 t π
∴ 2
dt =
t 2
0

(
1 − x2 if |x| < 1
Example 4.2. Find the Fourier transform of f (x) = and hence evaluate
0 if |x| > 1
Z∞  
sin x − x cos x x
cos dx
x3 2
0

Solution : The given function can be written as

(
1 − x2 if − 1 < x < 1
f (x) =
0 otherwise
94 Part-B : Unit IV - FOURIER TRANSFORMS (F.T.)

Z∞
1
F (s) = F [f (x)] = √ f (x)eisx dx

−∞
Z1
1
=√ (1 − x2 )(cossx + isinsx)dx

−1
Z1
1  
=√ (1 − x2 ) cos sx + i(1 − x2 ) sin sx dx

−1
Z1
2
=√ (1 − x2 ) cos sxdx

0
   
∵ 1 − x2 cos sx is even and 1 − x2 sin sx is odd
r h  − cos sx   − sin sx i1
2 2 sin sx
= (1−x ) −(−2x) +(−2)
π s s2 s3 0
r h  i
2 sin sx cos sx sin sx
= (1 − x2 ) − 2x 2 + (2)
π s s s3
r h i
2 −2 cos s 2 sin s
= +
π s2 s3
r  
2 sin s − s cos s
F (s) =2
π s3
By inversion formula for Fourier transform
Z∞
1
f (x) = √ F (s)e−isx ds

−∞
Z∞ r  
1 2 sin s − s cos s
=√ 2. e−isx ds
2π π s3
−∞
Z∞ h    i
2 sin s − s cos s sin s − s cos s
= 3
cos sx − i sin sx ds
π s s3
−∞
Z∞  sin s − s cos s 
4
f (x) = cos sxds
π s3
h  sin s − s 0cos s   sin s − s cos s  i
∵ cos sx is even and sin sx is odd
s3 s3
Z 


sin s − s cos s π
∴ 3
cos sxds = f (x)
s 4
0
1
Put x =
2
MA8353 Transforms and Partial Differential Equations by K A Niranjan Kumar 95

Z∞    
sin s − s cos s s π 1
cos ds = 1 −
s3 2 4 4
0

=
16

2 2 x2
Example 4.3. Find the Fourier transform of e−a x
. Hence prove that e− 2 is self reciprocal with
respect to the Fourier transform.

2 2
Solution : Given f (x) = e−a x
.
The Fourier transform of f (x) is given by
Z∞
1
F (s) = F [f (x)] = √ f (x)eisx dx

−∞
Z∞
1 2 2
=√ e−a x
eisx dx

−∞
Z∞
1 2 2
=√ e−(a x −isx)
dx

−∞
Z∞
1 2 is
=√ e−[(ax) −2(ax)( 2a )] dx

−∞
Z∞
is 2 is 2
h i
1 is
− (ax)2 −2(ax)( 2a )+( 2a ) −( 2a )
=√ e dx

−∞
Z∞
is 2 s2
h i
1 − (ax− 2a )+
=√ e 4a2 dx

−∞
Z∞
1 is 2 s2
e−(ax− 2a ) e

=√ 4a2 dx

−∞
Z∞
1 − s2 is 2
= √ e 4a2 e−(ax− 2a ) dx

−∞
is dt
Put ax − = t ⇒ dx =
2a a
Z∞
1 − s22 2 dt
∴ F (s) = √ e 4a e−t
2π a
−∞

2
Z∞
1 −s 2
= √ e 4a2 e−t dt
a 2π
−∞
96 Part-B : Unit IV - FOURIER TRANSFORMS (F.T.)

2
Z∞
1 −s 2
= √ e 4a2 2 e−t dt
a 2π
0
2
 √ 
1 − s2 π
= √ e 2 4a
a 2π 2
1 − s22
∴ F (s) = √ e 4a
a 2
1 − s2
F [f (x)] = √ e 4a2
a 2
 2 2 1 − s2
h i
−a2 x2
F e−a x = √ e 4a2 ∵ f (x) = e
a 2
1
Setting a = √
  2
x2 s2
F e− 2 =e− 2

x2
∴ f (x) =e− 2 is self reciprocal.

4.3 Parseval’s Identity for Fourier transform:


(
a2 − x2 , |x| < a
Example 4.4. Show that the Fourier transform of f (x) = as
0, |x| > a
r   Z∞  
2 sin as − as cos as sin s − s cos s π
2 . Hence deduce (i) ds =
π s3 s 3 4
0
Z∞  ( )
sin s − s cos s
2 π x = 0, a = 1 in (i),
(ii) ds = .
s3 15 Parseval’s with a = 1 in (ii)
0

(
a − |x| , |x| < a
Example 4.5. Find the Fourier transform of f (x) = and hence deduce that (i)
0, |x| > a
Z∞  2 Z∞  4
sin t π sin t π
dt = . (ii) dt = .
t 2 t 3
0 0  
r   Z∞  as 2

 2 1 as 2 sin 

 F (s) = 2 sin2 , f (x) = 2
e−isx
ds, 
π s2 2 π s

 −∞ 

 
x = 0, a = 2 in (i), Parseval’s with a = 2 in (ii)

4.3.1 Examples under Fourier Sine & Cosine Transform:

Example 4.6. Find the Fourier sine and Cosine Transform of e−ax , x ≥ 0.
MA8353 Transforms and Partial Differential Equations by K A Niranjan Kumar 97

Solution : The Fourier sine transform of f (x) is given by

r Z∞
2
FS (s) = FS [f (x)] = f (x) sin sxdx
π
0
r Z∞
2
= e−ax sin sxdx
π
r 0 ∞
2 e−ax
= (−a sin sx − s cos sx)
π a2 + s 2 0
r
2 s
=
π a + s2
2

The Fourier Cosine transform of f (x) is given by

r Z∞
2
Fc (s) = Fc [f (x)] = f (x) cos sxdx
π
0
r Z∞
2
= e−x cos sxdx
π
r 0 ∞
2 e−x
= (−a cos sx + s sin sx)
π a2 + s 2 0
r
2 a
=
π a + s2
2

4.3.2 Examples under Convolution & Parseval’s identity:

Z∞  
dx π
Example 4.7. Evaluate .
(x + a ) (x2 + b2 )
2 2 2ab (a + b)
0

Z∞  
x2 dx π
Example 4.8. Evaluate .
(x2 + a2 ) (x2 + b2 ) 2 (a + b)
0

Z∞ n π o
dx
Example 4.9. Evaluate 2
.
(x2 + a2 ) 4a3
0

Z∞ nπo
x2 dx
Example 4.10. Evaluate 2
.
(x2 + a2 ) 4a
0
98 Part-B : Unit IV - FOURIER TRANSFORMS (F.T.)

4.4 Assignment IV[Fourier Transforms]




 0, x < 0
1. Find the Fourier integral representation of f (x) defined as f (x) = 1 .
 2, x =0

e−x , x>0
(
1 − x2 ; if |x| < 1
2. Find the Fourier transform of the function f (x) defined by f (x) = . Hence
0; if |x| ≥ 1
prove that
Z∞    
sin s − s cos s s 3π
(i) cos ds =
s3 2 16
0
Z∞  2
sin s − s cos s π
(ii) ds = .
s3 15
0
( Z∞ 
1 − |x| ; if |x| < 1, sin t

3. Find F.T. of f (x) = and hence find the value of dt and
0; if |x| > 1, t
0
Z∞  
sin t 4
dt
t
0
2 2 2
4. Find the Fourier transform of e−a x
. Hence prove e−x /2
is self reciprocal.


 x; 0 < x < 1
5. Find the Fourier sine and cosine transform of f (x) = 2 − x; 1 < x < 2 .


0; x > 2
2
6. Prove that e−x /2
is self reciprocal under Fourier cosine transform.
1
7. Find the Fourier sine and cosine transform of xn−1 and hence prove √ is self reciprocal under
x
Fourier sine and cosine transforms.

8. Find the Fourier sine transform of e−ax and hence evaluate Fourier cosine transforms of xe−ax
and e−ax sin ax.
Z∞
x2
9. Find F.S.T. and F.C.T. of e−ax , a > 0. Hence evaluate 2
dx and
(a2 + x2 )
0
Z∞
dx
(x2 + a2 ) (x2 + b2 )
0

10. State and prove convolution theorem and Parseval’s identity for Fourier transforms.
5 Z-Transforms(Z.T.)

5.1 Part-A

1. Find Z [an ]. z
3. Prove Z [n] = .
(z − 1)2
Soln : We know that Soln : We know that

X

X Z [x (n)] = x (n) z −n
Z [x (n)] = x (n) z −n
n=0
n=0 ∞
∞ X
X Z [n] = nz −n
∴ Z [an ] = an z −n
n=0
n=0 ∞ 
∞  X n
 1 2 3
X a
n
= = 0+ + 2 + 3 + . . .
= z n z z z
z n=0
n=0     2 
a
 a 2 1 1 1
=1+ + + ··· = 1+2 +3 +...
z z z z z
h i h i−2 h i2
a −1 1 z−1 1 z z
= 1− = = =
z z z z z−1 (z − 1)2
h z − a i−1
=
 
1
h zz i 4. Find Z .
n (n + 1)
∴ Z [an ] = 1 A B
z−a
Soln : Let = + (1)
n (n + 1) n n+1
2. Find Z [u (n − 1)]. 1 = A (n + 1) + B (n)
Soln : Put n = 0, we get A = 1

X Put n = −1, we get B = −1
Z [u (n − 1)] = 1.z −n
1 1 1
n=0 (1) ⇒ = −
1 1
  n (n + 1) n n+1
= + 2 +···
  h1 i
z z 1 1
 1 2  ∴Z =Z −
1 1 n (n + 1) n n+1
= 1+ + + ··· h1i h 1 i
z z z =Z −Z
1
h 1 −1
i n n+1
= 1− z z
z z = log − zlog
h i h z i z−1 z−1
1 z − 1 −1 1 z
= = = (1 − z) log
z z z z−1 z−1
1
=
z−1
100 Part-A : Unit V - Z−TRANSFORMS AND DIFFERENCE EQUATIONS (Z.T.)

d Soln : We know that


5. Prove that Z [n f (n)] = −z F (z) ..
dx ∞
X
Soln : We know that Z [x (n)] = x (n) z −n

X
Z [x (n)] = x (n) z −n n=0
n=0 h1i X∞ 
1

X∞ Z = z −n
d −n−1 n! n!
[F (z)] = (−n) f (n) z n=0
dz ∞    n
X
n=0 1 1
X∞ =
z −n n! z
=− nf (n) n=0
z
1
1 1
 1 2
n=0
∞ =1+ + +···
d X 1! z 2! z
z [F (z)] = − nf (n) z −n 1
 
1 2
dz z z
n=0 = 1+ + + ···
1! 2!
= −Z [nf (n)]
= e1/z
d
Z [nf (n)] = −z [F (z)] h1i
dz
  9. Find Z .
6. Find Z e−t t . 2 3n
Soln
 : We know
 that z
Soln : We know that Z [an ] =
Z e−at f (t) = Z [f (t)]z→zeaT z−
    h1i a n 
1
Z e−t t2 = Z t2 z→zeT Z n =Z
  3 3
T 2 z (z + 1) z
= =
(z − 1)3 z→zeT z − 31

T 2 zeT zeT +1 3z
= =
(zeT − 1)
3 3z − 1
 
7. Find Z [1]. 10. Find Z e3t−5 .
Soln : We know that Soln :
   
Z e3t−5 = Z e3t e−5

X  
Z [x (n)] = x (n) z −n = e−5 Z e3t
 
n=0 = e−5 Z e3nT
X∞ ∞  n
X h n i
1
Z [1] = 1.z −n
= = e−5 Z e3T
z
n=0 n=0
 2 e−5 z
1 1 =
=1+ + + ··· z − e3T
z z  
h i −1
1 11. Find Z 4 (3)n + 2 (−1)n .
= 1−
z Soln : We know that
h z − 1 i−1
= z
h zz i Z [an ] =
z−a
=    
z−1 Z 4.3n + 2 (−1)n = 4Z [3n ] + 2Z (−1)n
h1i 4z 2z
8. Show that Z = e1/z . = +
n! z−3 z+1
MA8353 Transforms and Partial Differential Equations by K A Niranjan Kumar 101

 
12. Find Z eat sin bt . lim F (z) = lim f (0)
z→∞ n→0
Soln : We know that
15. Define unit step sequence.
 −at

Z e f (t) = Z [f (t)]z→zeaT Soln : The unit step sequence u(n) has
 
Z e−at sin bt = [Z (sin bt)]z→zeaT values (
  1 for n > 0
z sin bT u (n) =
= 2
z − 2z cos bT + 1 0 for n < 0
z→zeaT
zeaT sin bT 16. State the Damping rule.
=
z 2 e2aT − 2zeaT cos bT + 1 Soln : The geometric factor a−n when
13. Find Z [an t]. |a| < 1, damps the function un . Hence we
Soln : We know that use the name damping rule.

hz i If Z (un ) = U (z), then Z a−n un = U (az)
n
Z [a f (t)] = F h
2z
i
a 17. If f (z) = . Find lim f (t) and f (0).
z − e−T t→∞
Z [an t] = [Z (t)]z→z/a Soln : By initial value theorem
 
Tz
= f (0) = lim F (z)
(z − 1)2 z→z/a
z→∞
z 2z
T a = lim
= 2 z→∞ z − e−T
z
a −1 2z
  = lim  
T az z→∞
z 1− z e−T
=
(z − a)2
=2
14. Find State and prove initial value theorem
in Z-transform. By final value theorem
Soln : lim f (t) = lim (z − 1) F (z)
t→∞ z→1
Statement : If Z [f (n)] = F (z) ,, then
2z
lim f (0) = lim F (z) = lim (z − 1)
n→0 z→∞
z→1 z − e−T
Proof : =0

X
−n
F [z] = f (n)z
5z
n=0 18. If f [z] = . Find find f (0).
(z − 2) (z − 3)
= f (0)z −0 + f (1)z −1 + f (2)z −2 + · · · Soln : By initial value theorem :
f (1) f (2)
= f (0) + + 2 + ··· f (0) = lim F (z)
z z z→∞
By applying limits, 5z ∞
  = lim =
f (1) f (2) z→∞ (z − 2) (z − 3) ∞
lim F (z) = lim f (0) +
+ 2 + ···
n→0 z→∞ z z 5
h i = lim
1 1 z→∞ (z − 2) + (z − 3)
= f (0) ∵ lim = lim 2 = 0
z→∞ z z→∞ z [ by L’Hospital’s rule]
We can write f (0) as lim f (0) 5
n→0 = lim =0
z→∞ 2z − 5
102 Part-A : Unit V - Z−TRANSFORMS AND DIFFERENCE EQUATIONS (Z.T.)

h  z i hz i
−1 n
19. Find Z log . Z [a fn ] = F
z+1 a
Soln : h1i
But, Z = e1/z
 z  n!
 n
Let F (z) = log a
h z
i
 z+ 1   ∴Z = ea/z Replacing z by
n! a
1/y 1
= log by z =  
(1/y) + 1 y
! 22. Find Z e−iat by z-transform.
1
= log
y Soln :
1+y
y    
  Z e−iat = Z e−iat · 1
1
= log
1+y = {Z(1)}z→zeiaT [By shifting thm.]
  h z i h z
i
= log (1 + y)−1 = ∵ Z (1) =
z − 1 z→zeiaT z−1
= − log (1 + y) ze iaT
= iaT
1 1 (−1)n −n ze − 1
= −y + y 2 − y 3 + ... + z f (n)
2 3 n where T is the sampling period
= Z −1 [f (n)]
( 23. Find the Z-transform of (n + 1)(n + 2).
0, for n = 0
= (−1)n Soln :
n , otherwise
Z[(n + 1)(n + 2)]

20. Form yn = a2n + b (−2)n , derive a = Z[n2 + 3n + 2]


difference equation by eliminating the = Z[n2 ] + 3Z[n] + Z[2]
constants. z (z + 1) 3z z
= + +
Soln : Given yn = a2n + b (−2)n (z − 1) 3
(z − 1) 2 z−1
n+1 n+1 3 2
z + 2z − z
yn+1 = a2 + b (−2) =
n n (z − 1)3
= 2a2 − 2b (−2)
yn+2 = a2n+2 + b (−2)n+2 24. Find the Z-transform of (n + 2).

= 4a2n + 4b (−2)n Soln : Z (n + 2) = Z (n) + Z (2)


Eliminating a and b, we get
z 2z
= +
yn 1 1 (z − 1) 2 (z − 1)


yn+1 2 −2 = 0 25. State final value theorem in Z-transform.

yn+2 4 4
Soln : If Z [f (n)] = F (z), then
⇒ yn+2 − 4yn = 0 lim f (0) = lim (z − 1) F (z)
n→∞ z→1
which is the desired difference equation.
26. Express Zf (n + 1) in terms of f (z).
  Soln : Using the formula of Z-transform of
an
21. Find Z by Z-transform. fn+1 .
n!  
Soln : By of change of scale property We get Zf (n + 1) = Z f − f0 .
MA8353 Transforms and Partial Differential Equations by K A Niranjan Kumar 103

 
27. Find the value of Zf (n) when f (n) = nan . (z − a) − z
= −z
Soln : We know that (z − a)2
 
−a
= −z
d (z − a)2
Z [nfn ] = −z [Z (fn )] az
dz ∴ Z [nan ] =
d
= −z [Z [an ]] [Here fn = an ] (z −a)2
dz
d
h z i
= −z
dz z −a

5.2 Part-B

Table of Z− transform of standard functions.

S.No. x(n) Z[x(n)]


z
1. 1
z−1
kz
2. k
z−1
kz
3. −k
z+1
z
4. an
z−a
z
5. n
(z − 1)2
az
6. nan
(z − a)2
z2 + z
7. n2
(z − 1)3
az(z + a)
8. n 2 an
(z − a)3
d 
9. np −z Z[np−1 ]
dz
1
 z 
10. log
n z−1
1 z
11. z log
n+1 z−1
1
1
12. ez
n!
a
an
13. ez
n!
104 Part-B : Unit V - Z−TRANSFORMS AND DIFFERENCE EQUATIONS (Z.T.)

14. δ(n) 1
1
15. δ(n − k)
zk
 a k
16. an δ(n − k)
z
z
17. u(n)
z−1
1
18. u(n − 1)
z−1
z(z − cos θ)
19. cos nθ 2
z − 2z cos θ + 1
z(z − a cos θ)
20. an cos nθ
z − 2az cos θ + a2
2

z sin θ
21. sin nθ 2
z − 2z cos θ + 1
az sin θ
22. an sin nθ
z 2 − 2az cos θ + a2
z(z − cos hθ)
23. cos hnθ 2
z − 2z cos hθ + 1
z(z − a cos hθ)
24. an cos hnθ
z − 2az cos hθ + a2
2

z sin hθ
25. sin hnθ
z 2 − 2z cos hθ + 1
az sin hθ
26. an sin hnθ
z − 2az cos hθ + a2
2

5.3 Inverse Z-transform

Definition. If Z[x(n)] = X(z), then Z −1 [X(z)] = x(n).


z
Example. We know that Z[1] = .
h z i z−1
∴ Z −1 = 1.
z−1
The following table gives the inverse Z−transform of standard functions.

S.No. X(z) x(n)


z
1. 1
z−1
z
2. (−1)n
z+1
z
3. an
z−a
MA8353 Transforms and Partial Differential Equations by K A Niranjan Kumar 105

z
4. n
(z − 1)2
z2 + z
5. n2
(z − 1)3
az
6. nan
(z − a)2
z2 nπ
7. an cos
z 2 + a2 2
az nπ
8. an sin
z 2 + a2 2
z2
9. (n + 1)an
(z − a)2
1
10. an−1
z−a
1
11. (−a)n−1
z+a
1
12. (n − 1)an−2
(z − a)2
1 1
13. (n − 1)(n − 2)an−3
(z − a)2 2
z
14. (−a)n
z+a
z3 1
15. (n + 1)(n + 2)an
(z − a)3 2!

5.3.1 Evaluation of inverse Z-transform using the method of Convolution

Statement. If Z −1 [X(z)] = x(n) and Z −1 [Y (z)] = y(n), then

n
X
−1
Z [X(z) × Y (z)] = x(n) ∗ y(n) = x(k)y(n − k)
k=0

 
−1 z2
Example 5.1. Use convolution theorem to evaluate Z [Dec 2011]
(z − a)(z − b)
106 Part-B : Unit V - Z−TRANSFORMS AND DIFFERENCE EQUATIONS (Z.T.)

Solution.

  h i
−1 z2 z z
Z = Z −1
(z − a) (z − b) z −az −b
h z i h z i
−1 −1
=Z ∗Z
z−a z−b
= an ∗ b n
n
X
= ak bn−k
k=0
Xn
= ak bn b−k
k=0
n  k
X
n a
=b
b
k=0
     
n a a 2 a n
=b 1+ + + ··· +
b b b
  n+1 
a
−1 h a
i
 
= bn  b a  G.P. with C.R.
−1 b
b
an+1 − bn+1 b an+1 − bn+1
= bn × = .
bn+1 a−b a−b

 
−1 z2
Example 5.2. Using convolution theorem, find Z .
(z − 1)(z − 3)
MA8353 Transforms and Partial Differential Equations by K A Niranjan Kumar 107

[Dec 2013, May 2011]


  h i
−1 z2 z z
Solution. Z = Z −1
(z − 1) (z − 3) z −1z −3
h z i h z i
−1 −1
=Z ∗Z
z−1 z−3
= 1n ∗ 3n
n
X
= 1k 3n−k
k=0
Xn
3n
=
3k
k=0
n  k
X
n 1
=3
3
k=0  2  1 n 
n 1 1
=3 1+ + + ··· +
3 3 3
  1 n+1 
1−
n 3 
=3  
1
1−
3
3 n+1 −1 3 3n+1 − 1
= 3n n+1 × = .
3 2 2
z2
Example 5.3. Using convolution theorem, find the inverse Z−transform of . [Dec 2012]
(z + a)2
  h z i
−1 z2 −1 z
Solution. Z =Z
(z + a)2 z +az +a
h z i h z i
= Z −1 ∗ Z −1
z+a z+a
= (−a)n ∗ (−a)n
n
X
= (−a)k (−a)n−k
k=0
Xn
(−a)n
= (−a)k
(−a)k
k=0
n
X
n
= (−a) 1
k=0
n
= (−a) [1 + 1 + · · · + 1 (n + 1) terms]
= (n + 1) (−a)n .

8z 2
Example 5.4. Using convolution theorem, find the inverse Z−transform of [Jun
(2z − 1) (4z − 1)
108 Part-B : Unit V - Z−TRANSFORMS AND DIFFERENCE EQUATIONS (Z.T.)

2012]

 
 
8z 2  8z 2 
Solution. Z −1 = Z −1     
(2z − 1) (4z − 1) 1 1 
8 z− z−
2 4
 
 z2 
= Z −1    
1 1 
z− z−
2 4
 
z z 
= Z −1  ×
1 1
z− z−
 2 4 
z  z 
= Z −1  ∗ Z −1 
1 1
z− z−
 1 n  12n 4
= ∗
2 4
Xn  k  n−k
1 1
=
2 4
k=0
Xn  k  1 n  1 −k
1
=
2 4 4
k=0
 1 k
 1 n X
n
= 2
4 1
k=0
4
 1 n X
n
= 2k
4
k=0
 1 n 2n+1 − 1
=
4 2−1
1 
= n 2n+1 − 1 .
4
MA8353 Transforms and Partial Differential Equations by K A Niranjan Kumar 109

 z 3
Example 5.5. Using convolution theorem, find the inverse Z−transform of . [May 2010]
z−4
 3   
−1 z −1 z z
Solution. Z =Z
z−4 z − 4 (z − 4)2
h z i  
−1 z2
=Z ∗ Z −1
z−4 (z − 4)2
= 4n ∗ (n + 1) 4n
n
X
= (k + 1) 4k × 4n−k
k=0
n
X
n
=4 (k + 1)
k=0
n
= 4 [1 + 2 + 3 + · · · (n + 1)]
(n + 1)(n + 2)
= 4n .
2

5.3.2 Evaluation of inverse Z-transform using partial fractions method


X(z)
While finding the inverse Z−transform, resolve into partial fractions which will result in
z
the correct procedure for finding the same.
" #
z z2
−z+2
Example 5.6. Find Z −1 . [May 2010]
(z + 1)(z − 1)2

z z2 − z + 2
Solution. Let X(z) = .
(z + 1)(z − 1)2
X(z) z2 − z + 2
= .
z (z + 1)(z − 1)2

z2 − z + 2 A B C
Let 2
= + +
(z + 1)(z − 1) z + 1 z − 1 (z − 1)2
A(z − 1)2 + B(z + 1)(z − 1) + C(z + 1)
=
(z + 1)(z − 1)2
∴ z 2 − z + 2 = A(z − 1)2 + B(z + 1)(z − 1) + C(z + 1).
When z = −1, 1 + 1 + 2 = A(−1 − 1)2
4A = 4
A = 1.
When z = 1, C(1 + 1) = 1 − 1 + 2
2C = 2
C = 1.
110 Part-B : Unit V - Z−TRANSFORMS AND DIFFERENCE EQUATIONS (Z.T.)

Equating the coefficients of z 2 we get

A+B =1
1+B =1
B = 0.
z2 − z + 2 1 1
∴ = +
(z + 1)(z − 1)2 z + 1 (z − 1)2
X(z) 1 1
= +
z z + 1 (z − 1)2
z z
∴ X(z) = + .
z + 1 (z − 1)2
h z i  
−1 −1 −1 z
Z [X(z)] = Z +Z
z+1 (z − 1)2
= (−1)n + n.
10z
Example 5.7. Find the inverse Z−transform of . [Dec 2009]
z2 − 3z + 2
10z
Solution. Let X(z) = .
z 2 − 3z + 2
X(z) 10
= .
z (z − 1)(z − 2)

10 A B
Let = +
(z − 1)(z − 2) z−1 z−2
10 = A(z − 2) + B(z − 1).
When z = 1, −A = 10
∴ A = −10.
When z = 2, B = 10.
X(z) 10 10
=− + .
z z−1 z−2
z z
X(z) = −10 + 10 .
z − 1h zi − 2 h z i
−1 −1 z −1
Z [X(z)] = −10Z + 10Z
z−1 z−2
= −10 × 1 + 10 × 2n
= 10 [2n − 1] .

z2 + z
Example 5.8. Find the inverse Z−transform of using partial fractions. [Nov 2014]
(z − 1)(z 2 + 1)
z2 + z
Solution. Let X(z) = .
(z − 1)(z 2 + 1)
X(z) z+1
= .
z (z − 1)(z 2 + 1)
MA8353 Transforms and Partial Differential Equations by K A Niranjan Kumar 111

z+1 A Bz + c
Let 2
= + 2 .
(z − 1)(z + 1) z−1 z +1
∴ z + 1 = A(z 2 + 1) + (Bz + c)(z − 1)
When z = 1, 2A = 2 ⇒ A = 1
Equating the coefficients of z 2 we get
A + B = 0 ⇒ B = −A ⇒ B = −1.
Equating the constant terms we get
A − C = 1 ⇒ C = A − 1 = 0 ⇒ C = 0.

X(z) 1 z
∴ = − 2 .
z z−1 z +1
z z2
X(z) = − 2 .
z−1 z +1  2 
h z i z nπ
Z −1 [X(z)] = Z −1 − Z −1 2 = 1 − cos .
z−1 z +1 2

5.3.3 Evaluation of inverse Z-transform by Residue method

By inversion
Z integral method, the inverse Z−transform of X(z) is given by
1
x(n) = X(z)z n−1 dz, where c is the closed contour containing all the isolated singularities of
2πi
c
X(z).
By Cauchy’s residue theorem,
x(n) =sum of the residues of X(z)z n−1 at the isolated singularities.
Method of evaluation of the residues

1. If z = a is a simple pole, then R(a) = lim (z − a)X(z)z n−1 .


z→a

1 dm−1  m n−1

2. If z = a is a pole of order m, then R(a) = lim (z − a) X(z)z .
z→a (m − 1)! dz m−1

z
Example 5.9. Using inversion integral method, find the inverse Z−transform of .
(z − 1)(z − 2)

z
Solution. Let X(z) = .
(z − 1)(z − 2)
z = 1 and z = 2 are simple poles .
112 Part-B : Unit V - Z−TRANSFORMS AND DIFFERENCE EQUATIONS (Z.T.)

Let R(1) and R(2) be the residues of X(z)z n−1 at z = 1 and z = 2 respectively.

Now R(1) = lim (z − 1)X(z)z n−1


z→1
z
= lim (z − 1) z n−1
z→1 (z − 1)(z − 2)
z
= lim z n−1
z→1 z − 2
zn 1
= lim = = −1.
z→1 z − 2 −1
R(2) = lim (z − 2)X(z)z n−1
z→2
z
= lim (z − 2) z n−1
z→2 (z − 1)(z − 2)
zn
= lim = 2n .
z→2 z − 1

By the inversion integral method,


Z −1 [X(z)] = x(n) = sum of the Residues
= R(1) + R(2)
= −1 + 2n = 2n − 1.

z
Example 5.10. Find the inverse Z−transform of . [Dec. 2013]
(z + 1)2

z
Solution. Let X(z) = .
(z + 1)2
z = −1 is a pole of order 2.
Residue of X(z)z n−1 at z = −1 is
1 d 
R(−1) = lim (z + 1)2 X(z)z n−1
z→−1 1! dz
 
d ✘
✘ z
= lim (z✘+✘1)2 ×
✘ ✘
✘ 2 ×z
n−1
z→−1 dz (z✘+✘1)

 
d n
= lim (z )
z→−1 dz

= lim nz n−1
z→−1

= n(−1)n−1 .
By inversion integral method
Z −1 [X(z)] = x(n) = sum of the Residues
= R(−1) = n(−1)n−1 .
MA8353 Transforms and Partial Differential Equations by K A Niranjan Kumar 113

 
−1 2z 2 + 4z
Example 5.11. Find Z using residue theorem.
(z − 2)3

2z 2 + 4z
Solution. Let X(z) = .
(z − 2)3
z = 2 is a pole of order 3.
Residue of X(z)z n−1 at z = 2 is
1 d2 
R(2) = lim 2 (z − 2)3 X(z)z n−1
2! z→2 dz  
1 d2 3 2z 2 + 4z n−1
= lim 2 (z − 2) × ×z
2 z→2 dz (z − 2)3
1 d2 
= lim 2 (2z 2 + 4z)z n−1
2 z→2 dz
1 d2 
= lim 2 2z n+1 + 4z n
2 z→2 dz
1 d 
= lim 2(n + 1)z n + 4nz n−1
2 z→2 dz
1 
= lim 2n(n + 1)z n−1 + 4n(n − 1)z n−2
2 z→2
1
= 2n(n + 1)2n−1 + 4n(n − 1)2n−2
2
1
= n(n + 1)2n + 22 n(n − 1)2n−2
2
1
= [n(n + 1)2n + n(n − 1)2n ]
2
1
= × 2n × n(n + 1 + n − 1)
2
1
= × 2n × n × 2n = n2 × 2n .
2
By the residue method
Z −1 [X(z)] = x(n) = sum of the Residues = R(2) = n2 × 2n .

2z
Example 5.12. Find the inverse Z−transform of by inversion integral method.
(z − 1)(z 2 + 1)

2z
Solution. Let X(z) = .
(z − 1)(z 2 + 1)
The poles are given by

(z − 1)(z 2 + 1) = 0
(z − 1)(z + i)(z − i) = 0
z = 1, z = i, z = −i.
All the poles are simple poles.
114 Part-B : Unit V - Z−TRANSFORMS AND DIFFERENCE EQUATIONS (Z.T.)

Let R(1), R(i) and R(−i) be the residues of X(z)z n−1 at z = 1, z = i and z = −i
respectively.

R(1) = lim (z − 1)X(z)z n−1


z→1
2z
= lim (z − 1) 2
z n−1
z→1 (z − 1)(z + 1)
2z n 2
= lim 2 = = 1.
z→1 z + 1 2
R(i) = lim (z − i)X(z)z n−1
z→i
2z
= lim (z − i) z n−1
z→i (z − 1)(z − i)(z + i)
2z n
= lim
z→i (z − 1)(z + i)
2in in−1
= =
(i − 1)2i i−1
R(−i) = lim (z + i)X(z)z n−1
z→−i
2z
= lim (z + i) z n−1
z→−i (z − 1)(z + i)(z − i)
2z n
= lim
z→−i (z − 1)(z − i)
2(−i)n (−i)n+1
= = .
(−i − 1)(−2i) 1+i
By the inversion integral method,
Z −1 [X(z)] = x(n) = sum of the Residues
= R(1) + R(i) + R(−i)
in−1 (−i)n+1
=1+ + .
i−1 1+i
h z
i
Example 5.13. Find Z −1 2
by residue method.
z − 2z + 2
z
Solution. Let X(z) = .
z2 − 2z + 2
The poles are given by
z 2 − 2z + 2 = 0 ⇒ (z − 1)2 + 2 − 1 = 0
(z − 1)2 + 1 = 0
(z − 1)2 = −1
z − 1 = ±i
z = 1 ± i.

1 + i and 1 − i are simple poles.


MA8353 Transforms and Partial Differential Equations by K A Niranjan Kumar 115

Let R(1 + i) and R(1 − i) be the residues of X(z)z n−1 at 1 + i and 1 − i respectively.

R(1 + i) = lim (z − 1 − i)X(z)z n−1


z→(1+i)
z
= lim (z − 1 − i) z n−1
z→1+i (z − 1 − i)(z − 1 + i)
zn
= lim
z→1+i (z − 1 + i)
(1 + i)n (1 + i)n
= =
1+i−1+i 2i
R(1 − i) = lim (z − 1 + i)X(z)z n−1
z→(1−i)
z
= lim (z − 1 + i) z n−1
z→1−i (z − 1 − i)(z − 1 + i)
zn
= lim
z→1−i (z − 1 − i)
(1 − i)n (1 − i)n
= =
1−i−1−i −2i
By the inversion integral method,
Z −1 [X(z)] = x(n) = sum of the Residues
= R(1 + i) + R(1 − i)
(1 + i)n (1 − i)n 1
= − = [(1 + i)n − (1 − i)n ] .
2i −2i 2i
Let 1 + i = r(cos θ + i sin θ).
r cos θ = 1,r sin θ = 1.
r2 = 1 + 1 = 2

r = 2.

Substituting r = 2 we get,
1 1
cos θ = √ , sin θ = √
2 2
π
∴θ= .
4
√  π π

Hence ,1 + i = 2 cos + i sin
4 4 
n
√ n π π n
(1 + i) = ( 2) cos + i sin
4 4 
√ n nπ nπ
= ( 2) cos + i sin .
4 4
Changing i into − i we get
√  nπ nπ

(1 − i)n = ( 2)n cos − i sin
h nπ 4 4 i
n n
√ n nπ nπ nπ
(1 + i) − (1 − i) = ( 2) cos + i sin − cos + i sin .
4 4 4 4
√ nπ
= ( 2)n × 2i sin
4
n
−1 1 √ n nπ nπ
∴ Z [X(z)] = ( 2) × 2i sin = 2 2 sin .
2i 4 4
116 Part-B : Unit V - Z−TRANSFORMS AND DIFFERENCE EQUATIONS (Z.T.)

5.3.4 Solution of difference equations using Z−transform

Example 5.14. Using Z−transform, solve the difference equation yn+2 + 2yn+1 + yn = n given that
y0 = 0 = y1 . [Dec 2013]

Solution. The given difference equation is


yn+2 + 2yn+1 + yn = n.
Taking Z − transform on both sides we get
Z [yn+2 ] + 2Z [yn+1 ] + Z [yn ] = Z[n]
h y1
i z
z 2 Z [yn ] − y0 − + 2z [Z [yn ] − y0 ] + Z [yn ] =
z (z − 1)2
z
z 2 [Z [yn ] − 0 − 0] + 2z [Z [yn ] − 0] + Z [yn ] =
(z − 1)2
 2  z
Z [yn ] z + 2z + 1 =
(z − 1)2
z z
Z [yn ] = =
2 2
(z − 1) (z + 2z + 1) (z − 1)2 (z + 1)2
 
z
⇒ yn = Z −1 .
(z − 1) (z + 1)2
2

z
Let Y (z) = .
(z − 1)2 (z + 1)2
The poles are given by
(z − 1)2 (z + 1)2 = 0.
z = 1, −1 which are of order 2.
Let R(1) and R(−1) be the Residues of Y (z)z n−1 at z = 1 and z = −1 respectively.

1 d  
R(1) = lim (z − 1)2 Y (z)z n−1
1! z→1 dz
 
d z
= lim (z − 1)2 z n−1
z→1 dz (z − 1)2 (z + 1)2
 
d zn
= lim
z→1 dz (z + 1)2
 
(z + 1)2 × n × z n−1 − z n × 2(z + 1)
= lim
z→1 (z + 1)4
4n1n−1 − 1n × 4 4n − 4
= =
16 16
n−1
R(1) = .
4
MA8353 Transforms and Partial Differential Equations by K A Niranjan Kumar 117

1 d  
R(−1) = lim (z − 1)2 Y (z)z n−1
1! z→−1 dz
 
d 2 z n−1
= lim (z − 1) z
z→−1 dz (z − 1)2 (z + 1)2
 
d zn
= lim
z→−1 dz (z − 1)2
 
(z − 1)2 × n × z n−1 − z n × 2(z − 1)
= lim
z→−1 (z − 1)4
4n(−1)n−1 + 4(−1)n−1 n+1
= = 4(−1)n−1
16 16
n+1 n−1
R(−1) = (−1) .
4

By the inversion integral method,

yn = Z −1 [Y (z)] = Sum of the residues


n−1 n+1
= + (−1)n−1
4 4
1 
= n − 1 + (n + 1)(−1)n−1 .
4

Example 5.15. Solve yn+2 + 6yn+1 + 9yn = 2n given that y 0 = y1 = 0.


[Dec 2012, Dec 2009]
Solution. The given difference equation is
yn+2 + 6yn+1 + 9yn = 2n .
Taking Z−transform on both sides we get

Z [yn+2 ] + 6Z [yn+1 ] + 9Z [yn ] = Z [2n ]


h y1
i z
z 2 Z [yn ] − y0 − + 6z [Z [yn ] − y0 ] + 9Z [yn ] =
z z−2
z
z 2 [Z [yn ]] + 6z [Z [yn ]] + 9Z [yn ] =
z−2
 2  z
Z [yn ] z + 6z + 9 =
z−2
2 z
Z [yn ] (z + 3) =
z−2
z
Z [yn ] =
(z − 2)(z + 3)2
 
−1 z
yn = Z .
(z − 2)(z + 3)2

z
Let Y (z) = .
(z − 2)(z + 3)2
The poles are given by z = 2, z = −3.
z = 2 is a simple pole and z = −3 is a pole of order 2.
118 Part-B : Unit V - Z−TRANSFORMS AND DIFFERENCE EQUATIONS (Z.T.)

Residue of Y (z)z n−1 at z = 2 is

R(2) = lim (z − 2)Y (z)z n−1


z→2
z
= lim (z − 2) z n−1
z→2 (z − 2)(z + 3)2
zn
= lim
z→2 (z + 3)2
2n
= .
25
Residue of Y (z)z n−1 at z = −3 is
1 d  
R(−3) = lim (z + 3)2 Y (z)z n−1
1! z→−3 dz
 
d z
= lim (z + 3)2 z n−1
z→−3 dz (z − 2)(z + 3)2
 
d zn
= lim
z→−3 dz (z − 2)
 
(z − 2) × n × z n−1 − z n × 1
= lim
z→−3 (z − 2)2
−5n(−3)n−1 − (−3)n
=
25
(3 − 5n)(−3)n−1
R(−3) = .
25
By residue theorem

yn = Z −1 [Y (z)]
= Sum of the residues
= R(2) + R(−3)
2n 3 − 5n
= + (−3)n−1 .
25 25
Example 5.16. Solve y(k + 2) + y(k) = 1, y(0) = y(1) = 0. [Jun 2012]
Solution. The given difference equation is
y(k + 2) + y(k) = 1.
Taking Z− transform both sides we get

Z [y(k + 2)] + Z [y(k)] = Z [1]


 
2 y(1) z
z Z [y(k)] − y(0) − + Z [y(k)] =
z z−1
z
z 2 Z [y(k)] + Z [y(k)] =
z−1
 z
Z[y(k)] z 2 + 1 =
z−1
z
Z[y(k)] = .
(z − 1) (z 2 + 1)
MA8353 Transforms and Partial Differential Equations by K A Niranjan Kumar 119

z
Let Y (z) = .
(z − 1) (z 2 + 1)
The poles are given by

(z − 1) z 2 + 1 = 0
z = 1, z 2 = −1.
z = ±i.
The poles z = 1, z = i, z = −i which are simple.
Residue of Y (z)z n−1 at z = 1 is

R(1) = lim (z − 1)Y (z)z n−1


z→1
z
= lim (z − 1) 2
z n−1
z→1 (z − 1) (z + 1)
z n 1n 1
= lim 2 = = .
z→1 z + 1 2 2
Residue of Y (z)z n−1 at z = i is

R(i) = lim (z − i)Y (z)z n−1


z→i
z
= lim (z − i) z n−1
z→i (z − 1)(z − i)(z + i)
zn
= lim
z→i (z − 1)(z + i)
in
=
(i − 1) × 2i
1 in 1 in 1 (1 − i)in 1
= =− =− = − (1 − i)in .
2 −1 − i 21+i 2 2 4
Residue of Y (z)z n−1 at z = −i is

R(i) = lim (z + i)Y (z)z n−1


z→−i
z
= lim (z + i) z n−1
z→−i (z − 1)(z − i)(z + i)
zn
= lim
z→−i (z − 1)(z − i)
(−i)n 1 in 1 in 1 (1 + i)in 1
= = =− =− = − (1 + i)in .
(−i − 1) × (−2i) 2 −1 + i 21−i 2 2 4
By residue theorem

y(k) = Sum of the residues


1 1 1
= − (1 − i)in − (1 + i)in
2 4 4
1 1 n 1 1 1 1
= − i (1 − i + 1 + i) = − in × 2 = − in
2 4 2 4 2 2
1
y(k) = (1 − in ) .
2
120 Part-B : Unit V - Z−TRANSFORMS AND DIFFERENCE EQUATIONS (Z.T.)

Example 5.17. Solve the difference equation yn+3 − 3yn+1 + 2yn = 0, given that
y0 = 4, y1 = 0, y2 = 8.
[Dec 2012, May 2011]
Solution. The given difference equation is
yn+3 − 3yn+1 + 2yn = 0.
Taking Z−transform on both sides we get

Z [yn+3 ] − 3Z [yn+1 ] + 2Z [yn ] = 0


h y1 y2
i
z3 Z [yn ] − y0 − − 2 − 3z [Z [yn ] − y0 ] + 2Z [yn ] = 0
z z
h 8
i
z 3 Z [yn ] − 4 − 2 − 3z [Z [yn ] − 4] + 2Z [yn ] = 0
z
z 3 Z [yn ] − 4z 3 − 8z − 3zZ [yn ] + 12z + 2Z [yn ] = 0
 
Z [yn ] z 3 − 3z + 2 = 4z 3 − 4z
4z 3 − 4z
Z [yn ] =
z 3 − 3z + 2 
4z z 2 − 1
=
(z + 1)(z + 2)(z − 1)
4z(z + 1)(z − 1)
=
(z + 1)(z + 2)(z − 1)
4z
Z [yn ] =
h 4z i h z i z+2
−1 −1
∴ yn = Z = 4Z = 4(−2)n
z+2 z+2
which is the required solution.

Example 5.18. Solve yn+2 + 4yn+1 + 3yn = 2n with y0 = 0, y1 = 1. [Dec 2010]


Solution. From the previous problem we get
z
Z [yn ] (z + 3)(z + 1) = +z
z−2
h 1 i
=z +1
z−2
h1 + z − 2i
=z
z−2
(z − 1)
=z
z−2
z(z − 1)
Z [yn ] =
(z + 1)(z − 2)(z + 3)
 
z(z − 1)
yn = Z −1 .
(z + 1)(z − 2)(z + 3)

z(z − 1)
Let Y (z) =
(z + 1)(z − 2)(z + 3)
The poles are given by z = −1, z = 2, z = −3, all of them are simple.
MA8353 Transforms and Partial Differential Equations by K A Niranjan Kumar 121

Residue of Y (z)z n−1 at z = 2 is

R(−1) = lim (z + 1)Y (z)z n−1


z→−1
z(z − 1)
= lim (z + 1) z n−1
z→−1 (z + 1)(z − 2)(z + 3)
(−1)(−2)(−1)n−1
=
(−3)2
2(−1) n−1 1
= = − (−1)n−1 .
−3 × 2 3

Residue of Y (z)z n−1 at z = 2 is

R(2) = lim (z − 2)Y (z)z n−1


z→2
(z − 2)z(z − 1)
= lim z n−1
z→2 (z + 1)(z − 2)(z + 3)
2 2n
= (2)n−1 = .
3×5 15

Residue of Y (z)z n−1 at z = −3 is

R(−3) = lim (z + 3)Y (z)z n−1


z→−3
(z + 3)z(z − 1)
= lim z n−1
z→−3 (z + 1)(z − 2)(z + 3)
z(z − 1)
= lim z n−1
z→−3 (z + 1)(z − 2)
(−3)(−4)
= (−3)n−1
(−2)(−5)
(−3)n
= −2 .
5

By the method of residues, the solution is given by

yn = Sum of the residues


= R(−1) + R(2) + R(−3)
1 1 2
= − (−1)n−1 + 2n − (−3)n .
3 15 5

Example 5.19. Solve Un+2 − 2Un+1 + Un = 2n given that U0 = 2, U1 = 1. [May 2010]


Solution. The given difference equation is
Un+2 − 2Un+1 + Un = 2n .
122 Part-B : Unit V - Z−TRANSFORMS AND DIFFERENCE EQUATIONS (Z.T.)

Taking Z−transform on both sides we get

Z [Un+2 ] − 2Z [Un+1 ] + Z [Un ] = Z [2n ]


 
2 U1 z
z Z [Un ] − U0 − − 2z [Z [Un ] − U0 ] + Z [Un ] =
z z−2
h 1
i z
z 2 Z [Un ] − 2 − − 2z [Z [Un ] − 2] + Z [Un ] =
z z−2
z
z 2 Z [un ] − 2z 2 − z − 2zZ [Un ] + 4z + Z [Un ] =
z−2
 2  2 z
Z [Un ] z − 2z + 1 − 2z + 3z =
z−2
2 z
Z [Un ] (z − 1) = + 2z 2 − 3z
z−2
h 1 i
=z + 2z − 3
z − 2 
1 + (2z − 3)(z − 2)
=z
z−2
 
1 + 2z 2 − 4z − 3z + 6
=z
z−2
" #
z 2z 2 − 7z + 7
=
z−2
" #
z 2z 2 − 7z + 7
Z [Un ] =
(z − 2)(z − 1)2
" #
z 2z 2 − 7z + 7
Un = Z −1 .
(z − 2)(z − 1)2


z 2z 2 − 7z + 7
Let U (z) = .
(z − 2)(z − 1)2
The poles are given by z = 2,and z = 1.
z = 2 is a simple pole and z = 1 is a pole of order 2.
Residue of U (z)z n−1 at z = 2 is

R(2) = lim (z − 2)U (z)z n−1


z→2

z 2z 2 − 7z + 7 n−1
= lim (z − 2) z
z→2 (z − 2)(z − 1)2
2(8 − 14 + 7) n−1
= 2
1
= 2n .
MA8353 Transforms and Partial Differential Equations by K A Niranjan Kumar 123

Residue of U (z)z n−1 at z = 1 is


1 d  
R(1) = lim (z − 1)2 U (z)z n−1
1! z→1 dz
( )

d 2
z 2z 2 − 7z + 7 n−1
= lim (z − 1) z
z→1 dz (z − 2)(z − 1)2
 
d 2z n+2 − 7z n+1 + 7z n
= lim
z→1 dz z−2
"   #
(z − 2) 2(n + 2)z n+1 − 7(n + 1)z n + 7nz n−1 − 2z n+2 − 7z n+1 + 7z n × 1
= lim
z→1 (z − 2)2
−(2n + 4 − ✚✚− 7 +✚
7n ✚ − (2 − 7 + 7)
7n)
=
1
= −2n + 3 − 2
= 1 − 2n.

By residue method, the solution is given by

Un = Sum of the residues


= R(2) + R(1)
= 2n + 1 − 2n.

5.4 Assignment V[Z-Transform and difference equations]

1. Find the Z-transform of cos nθ and sin nθ. Hence deduce the Z-transform of
an cos nθ and an sin nθ.

2. Find Z (nan sin nθ).


!
z z2 − z + 2
3. Find z −1 by using method of partial fraction.
(z + 1) (z − 1)2
" #  
z z2 − z + 2 z
4. Find Z −1 and Z −1
(z + 1)(z − 1)2 (z − 1)(z − 2)
 
−1 z2
5. Using convolution theorem, find the Z
(z − 4) (z − 3)

12z 2
6. Using convolution theorem find the inverse Z-transform of
(3z − 1) (4z + 1)
z(z + 1)
7. Find the inverse Z-transform of by residue method.
(z − 1)3
8. Derive the difference equation from yn = (A + Bn )(−3)n .
124 Part-B : Unit V - Z−TRANSFORMS AND DIFFERENCE EQUATIONS (Z.T.)

9. Solve the difference equation using Z-transform y(n+3) − 3y(n+2) + 2y(n) = 0 given that
y0 = 4, y1 = 0, y2 = 8.

10. Solve y(n+2) + 3y(n+1) + 9y(n) = 2n given that y0 = y1 = 0.

You might also like